resolução de problemas

306

Upload: dionisio-sa

Post on 07-Aug-2015

1.413 views

Category:

Documents


7 download

TRANSCRIPT

Page 1: Resolução de problemas

Mestrado Profissional

em Matemática em Rede Nacional

Iniciação à Matemática

Autores:

Krerley Oliveira Adán J. Corcho

Unidade I:

Capítulos I e II

Page 2: Resolução de problemas

.

Dedicamos este livro as nossas esposas e lhos, que compreenderam

os sábados sacricados em função de escrevê-lo e a nossos pais, por

tudo o que eles representam.

Tente! E não diga que a vitória está perdida. Se é de batalhas que se

vive a vida. Tente outra vez! (Raul Seixas)

Page 3: Resolução de problemas

vi

Page 4: Resolução de problemas

Sumário

Prefácio xi

1 Primeiros Passos 1

1.1 Organizando as Ideias . . . . . . . . . . . . . . . . . . 1

1.2 Verdadeiro ou Falso? . . . . . . . . . . . . . . . . . . . 5

1.3 Teoremas e Demonstrações . . . . . . . . . . . . . . . . 9

1.3.1 Métodos de Demonstração . . . . . . . . . . . . 10

1.4 Algumas Dicas para Resolver Problemas . . . . . . . . 15

1.5 Soluções dos Problemas da Seção 1.4 . . . . . . . . . . 18

1.6 Exercícios . . . . . . . . . . . . . . . . . . . . . . . . . 26

2 Equações e Inequações 31

2.1 Equações do Primeiro Grau . . . . . . . . . . . . . . . 33

2.1.1 Problemas Resolvidos . . . . . . . . . . . . . . . 37

2.2 Sistemas de Equações do Primeiro Grau . . . . . . . . 42

2.2.1 Problemas Resolvidos . . . . . . . . . . . . . . . 46

2.3 Equação do Segundo Grau . . . . . . . . . . . . . . . . 49

2.3.1 Completando Quadrados . . . . . . . . . . . . . 50

2.3.2 Relação entre Coecientes e Raízes . . . . . . . 55

2.3.3 Equações Biquadradas . . . . . . . . . . . . . . 59

2.3.4 O Método de Vièti . . . . . . . . . . . . . . . . 60

vii

Page 5: Resolução de problemas

viii SUMÁRIO

2.4 Inequações . . . . . . . . . . . . . . . . . . . . . . . . . 62

2.5 Inequação do Primeiro Grau . . . . . . . . . . . . . . . 63

2.6 Inequação do Segundo Grau . . . . . . . . . . . . . . . 69

2.6.1 Máximos e Mínimos das Funções Quadráticas . 75

2.7 Miscelânea . . . . . . . . . . . . . . . . . . . . . . . . . 77

2.7.1 Equações Modulares . . . . . . . . . . . . . . . 77

2.7.2 Um Sistema de Equações Não lineares . . . . . 80

2.8 Exercícios . . . . . . . . . . . . . . . . . . . . . . . . . 81

3 Divisibilidade 89

3.1 Conceitos Fundamentais e Divisão Euclidiana . . . . . 90

3.2 Bases Numéricas . . . . . . . . . . . . . . . . . . . . . 99

3.3 Máximo Divisor Comum e Mínimo Múltiplo Comum . 106

3.3.1 Máximo Divisor Comum . . . . . . . . . . . . . 106

3.3.2 Algoritmo de Euclides . . . . . . . . . . . . . . 111

3.3.3 Mínimo Múltiplo Comum . . . . . . . . . . . . 115

3.3.4 Equações Diofantinas Lineares . . . . . . . . . . 120

3.4 Números Primos e Compostos . . . . . . . . . . . . . . 123

3.5 Procurando Primos . . . . . . . . . . . . . . . . . . . . 127

3.5.1 O Crivo de Eratóstenes . . . . . . . . . . . . . . 127

3.5.2 Primos de Mersenne . . . . . . . . . . . . . . . 129

3.5.3 O Teorema Fundamental da Aritmética . . . . . 133

3.6 Exercícios . . . . . . . . . . . . . . . . . . . . . . . . . 139

4 O Princípio da Casa dos Pombos 143

4.1 Primeiros Exemplos . . . . . . . . . . . . . . . . . . . . 145

4.2 Uma Versão mais Geral . . . . . . . . . . . . . . . . . . 146

4.3 Aplicações na Teoria dos Números . . . . . . . . . . . . 149

4.4 Aplicações Geométricas . . . . . . . . . . . . . . . . . . 151

Page 6: Resolução de problemas

SUMÁRIO ix

4.5 Miscelânea . . . . . . . . . . . . . . . . . . . . . . . . . 153

4.6 Exercícios . . . . . . . . . . . . . . . . . . . . . . . . . 157

5 Contagem 161

5.1 Princípio Aditivo da Contagem . . . . . . . . . . . . . 162

5.2 Princípio Multiplicativo de Contagem . . . . . . . . . . 170

5.3 Uso Simultâneo dos Princípios Aditivo e Multiplicativo 178

5.4 Permutações Simples . . . . . . . . . . . . . . . . . . . 181

5.5 Arranjos Simples . . . . . . . . . . . . . . . . . . . . . 184

5.6 Combinações Simples . . . . . . . . . . . . . . . . . . . 188

5.7 O Binômio de Newton . . . . . . . . . . . . . . . . . . 193

5.8 Contagem e Probabilidades . . . . . . . . . . . . . . . 195

5.9 Exercícios Propostos . . . . . . . . . . . . . . . . . . . 197

6 Indução Matemática 203

6.1 Formulação Matemática . . . . . . . . . . . . . . . . . 204

6.2 Aplicações . . . . . . . . . . . . . . . . . . . . . . . . . 206

6.2.1 Demonstrando Identidades . . . . . . . . . . . . 206

6.2.2 Demonstrando Desigualdades . . . . . . . . . . 210

6.2.3 Indução e Problemas de Divisibilidade . . . . . 212

6.3 Indução na Geometria . . . . . . . . . . . . . . . . . . 215

6.4 Miscelânea . . . . . . . . . . . . . . . . . . . . . . . . . 220

6.4.1 Cuidados ao Usar o Princípio da Indução . . . . 222

6.5 Indução e Recorrências . . . . . . . . . . . . . . . . . . 222

6.6 Exercícios . . . . . . . . . . . . . . . . . . . . . . . . . 229

7 Desigualdades 233

7.1 Desigualdade Triangular . . . . . . . . . . . . . . . . . 234

7.2 Desigualdade das Médias . . . . . . . . . . . . . . . . . 238

Page 7: Resolução de problemas

x SUMÁRIO

7.3 Desigualdade de Cauchy-Schwarz . . . . . . . . . . . . 245

7.4 Desigualdade de Jensen . . . . . . . . . . . . . . . . . . 246

7.5 Exercícios . . . . . . . . . . . . . . . . . . . . . . . . . 250

8 Polinômios 255

8.1 Operações com Polinômios . . . . . . . . . . . . . . . . 255

8.2 Algoritmo de Euclides . . . . . . . . . . . . . . . . . . 263

8.3 Sempre Existem Raízes de um Polinômio? . . . . . . . . 268

8.3.1 Números Complexos e Raízes de Polinômios . . 269

8.4 Exercícios . . . . . . . . . . . . . . . . . . . . . . . . . 272

A Apêndice: Funções 279

Referências 285

Page 8: Resolução de problemas

PrefácioImaginação é mais importante que onhe imento. Albert EinsteinLeo, vo ê tem uma religião? Assim, uma religião, omo judaísmo,ou ristianismo, ou Matemáti a...?Alon Peres, 6 anos, lho do Matemáti o Yuval PeresNeste livro pretendemos oferecer ao leitor uma introdução à Mate-

mática Elementar. Juntando as experiências didáticas vividas pelos

autores individualmente no Brasil e em Cuba, e mais alguns anos

juntos como treinadores de projetos de introdução à Matemática no

estado de Alagoas, esperamos tornar para o leitor a Matemática mais

interessante, mostrando um pouco do imenso brilho e beleza que ela

esconde.

O livro foi escrito em capítulos, cada um deles detalhando um

tema central e trazendo alguns teoremas fundamentais. Com muitos

exemplos e aplicações dos conceitos introduzidos, pretendemos mos-

trar ao leitor a importância do assunto abordado. A organização dos

exemplos tenta seguir uma linha em ordem crescente de diculdade e,

para o melhor aproveitamento do livro, o trabalho com os exercícios

é parte fundamental. Ler o enunciado e resolver o maior número pos-

xi

Page 9: Resolução de problemas

xii Prefácio

sível de exercícios é imperativo. Como já disse o Prof. Elon Lima,

Matemática não se aprende passivamente.

Os exemplos e aplicações dos conceitos, bem como os teoremas,

devem ser lidos com cuidado e muita atenção. Para os estudantes

que desejem treinar para olimpíadas de Matemática, sugerimos que

formem grupos de estudo para trabalhar os temas individualmente,

sob a orientação de um professor. Acreditamos que o texto pode ser

utilizado em uma disciplina elementar num curso de licenciatura ou

bacharelado em Matemática.

O primeiro capítulo é para introduzir o leitor no espírito do livro

e dar uma amostra do tipo de problemas e material que seguirá nos

demais capítulos. São propostos alguns problemas, muitos deles com

soluções, e discutimos alguns métodos importantes para uso no dia a

dia dos estudantes. Nesta discussão incluímos o estudo de proposições

matemáticas, provas por contraposição, o método de redução ao absur-

do e algumas outras regras básicas e cuidados que devemos ter ao

resolver problemas em Matemática.

Em seguida, estudamos as equações do primeiro e do segundo grau.

Estudamos os métodos de resolução dessas equações, sistemas de equa-

ções, relações entre raízes e coecientes, bem como alguns problemas

interessantes que podem ser solucionados via essas equações. Em se-

guida, estudamos inequações do primeiro e do segundo grau.

O capítulo seguinte trata do conceito de divisibilidade. Tentamos

introduzir o leitor nos principais aspectos básicos, incluindo-se a divisi-

bilidade com resto, máximo divisor comum e mínimo múltiplo comum,

números primos e compostos, e um pouco de equações diofantinas li-

neares.

Um capítulo útil para o estudante que deseja participar de Olim-

Page 10: Resolução de problemas

Prefácio xiii

píadas de Matemática é o que trata do princípio da casa dos pombos.

Este capítulo é um belo exemplo de como algo aparentemente ingênuo

pode gerar consequências interessantes. Alguns dos exemplos estão

conectados com os capítulos anteriores e aparentemente aplicam o

princípio de modo inusitado, em problemas de geometria, teoria dos

números e em áreas diversas.

No capítulo de contagem, começamos com noções úteis sobre con-

juntos e princípios básicos para contar os elementos de um conjunto.

Nesse capítulo, estamos mais preocupados com as aplicações imediatas

do assunto, sugerindo alguns problemas para o estudante iniciante.

Seguimos discutindo os tipos de agrupamento de elementos e suas

consequências. Obtemos o binômio de Newton e introduzimos a no-

ção de probabilidade de um conjunto, resolvendo alguns problemas

relacionados.

Em seguida, estudante se depara com uma arma poderosa do mate-

mático. O método da indução nita é estudado procurando conectar

esta noção com os capítulos anteriores, reobtendo com o auxílio do

método da indução algumas coisas que já foram deduzidas por outros

métodos. Vários exemplos e problemas são resolvidos, alguns deles de

modo surpreendente e inesperado.

No próximo capítulo, introduzimos algumas desigualdades popula-

res para o uso do estudante. Algumas dessas desigualdades são muito

importantes no estudo mais profundo da Matemática e não apare-

cem em cursos introdutórios, apesar de suas provas e aplicações serem

elementares. Todas as desigualdades aparecem com demonstrações,

em muito dos casos utilizando-se álgebra elementar e o método de

indução nita. São apresentados vários exemplos que mostram a uti-

lidade dessas desigualdades em alguns problemas práticos. Para xar

Page 11: Resolução de problemas

xiv Prefácio

o conhecimento, propomos vários exercícios complementares. Alguns

deles, cuja solução é mais elaborada, são sugeridos. No último ca-

pítulo, estudamos um pouco as propriedades gerais dos polinômios.

Para complementar a formação do leitor menos experiente, incluímos

um apêndice sobre funções.

Somos gratos a muitas pessoas que colaboraram com a elaboração

deste livro com sugestões e correções em versões iniciais. Entre eles,

citamos: Carlos Gustavo Moreira, Ali Tahzibi, Feliciano Vitório, Edu-

ardo Teixeira, Chico Potiguar e vários de nossos alunos de Iniciação

Cientíca e mestrado, que por várias ocasiões deram sugestões para

a melhoria do texto. Um agradecimento especial vai para Fernando

Echaiz, que nos ajudou ativamente nas notas do Capítulo 5 que origi-

naram este texto. Finalmente, agradecemos aos revisores pela leitura

cuidadosa e ao comitê editorial da SBM, na pessoa da profa. Helena

Lopes, pelo excelente trabalho de editoração.

Maceió, Abril de 2010

Krerley Oliveira

Adán J. Corcho

Page 12: Resolução de problemas

1

Primeiros PassosRedu tio ad absurdum, que Eu lides gostava tanto, é uma das maisnas armas do matemáti o. É muito mais no que um movimentode xadrez: o jogador de xadrez pode ofere er o sa rifí io de umapeça, mas o matemáti o ofere e o jogo inteiro. G. H. HardyNeste capítulo, discutiremos algumas ideias gerais e convenções

que servirão como base para os diferentes métodos de resolução de

problemas que trataremos nos capítulos seguintes. Alguns dos exem-

plos que abordamos serão úteis para orientar quanto ao cuidado que

devemos ter quando discutimos problemas em Matemática.

1.1 Organizando as Ideias

Para resolver problemas matemáticos precisamos ter bem claro o que

devemos provar e o que estamos assumindo como verdade. É sobre

isso que falaremos agora. Começaremos observando as seguintes ar-

mações:

1

Page 13: Resolução de problemas

2 1 Primeiros Passos

(a) A soma de dois números pares é sempre um número par.

(b) Todo brasileiro é carioca.

(c) A terra é um planeta.

(d) Se c é o comprimento da diagonal de um retângulo de lados a e

b, então c2 = a2 + b2.

(e) Se a < 1, então a2 > a.

Todas as armações acima se encaixam no conceito de proposição,

que damos a seguir.

Uma proposição ou sentença é uma frase armativa em forma de

oração, com sujeito, verbo e predicado, que ou é falsa ou é verdadeira,

sem dar lugar a uma terceira alternativa.

Por exemplo, as proposições (a) e (c) são claramente verdadeiras;

mais adiante nos convenceremos da veracidade da proposição (d). Por

outro lado, as proposições (b) e (e) são falsas. Com efeito, para cons-

tatar a veracidade da sentença (b) teríamos que checar o registro de

nascimento de cada brasileiro e vericar se nasceu no Rio de Janeiro,

mas isto é falso pois o conhecido escritor Graciliano Ramos é um

brasileiro nascido em Alagoas. Analogamente, para convencer-nos de

que a proposição (e) é falsa basta tomar a = 1/2 e checar que (1/2)2 =

1/4 não é maior do que 1/2 como a sentença arma. Em ambos os

casos temos vericado que as proposições (b) e (e) são falsas apre-

sentando casos particulares onde as mesmas deixam de valer. Estes

casos particulares são chamados de contraexemplos e são muito úteis

para vericar a falsidade de algumas proposições.

Notemos que as proposições (d) e (e) são do tipo:

Page 14: Resolução de problemas

1.1 Organizando as Ideias 3

Se P , então Q,

onde P e Q também são sentenças. Por exemplo, na proposição (e)

temos que:

P : c é o comprimento da diagonal de um retângulo de lados a e b,

Q: c2 = a2 + b2,

ou seja, estamos assumindo que P é verdade e usando este fato deve-

mos vericar se P é verdade ou não.

Uma proposição condicional ou implicativa é uma nova proposição

formada a partir de duas proposições P e Q, que é escrita na forma:

Se P , então Q ou P implica Q,

onde para o último caso usamos a notação: P =⇒ Q. Chamaremos

a proposição P de hipótese e a proposição Q de tese. A hipótese

também é chamada de proposição antecedente e a tese, de proposição

consequente.

Por exemplo, na proposição condicional (f) a hipótese é: a < 1 e a

tese é: a2 > a.

A partir de uma de uma proposição condicional podem-se gerar

novas proposições que são de especial interesse para os matemáticos.

Vamos chamar o modo em que apresentamos uma proposição de forma

positiva. Por exemplo, quando enunciamos a proposição

Se como laranja, então gosto de frutas,

assumimos esta armação como sua forma positiva. Vamos descrever

agora como podemos obter novas proposições a partir desta.

Page 15: Resolução de problemas

4 1 Primeiros Passos

Forma recíproca de uma proposição condicional: para cons-

truirmos a forma recíproca, temos que trocar na forma positiva a hi-

pótese pela proposição consequente e vice-versa. Vejamos em nosso

exemplo:

Forma da proposição Hipótese Tese

Positiva como laranja gosto de frutas

Recíproca gosto de frutas como laranja

Assim, a recíproca de proposição de nosso exemplo é então:

Se gosto de frutas, então como laranja

Forma contrapositiva de uma proposição condicional: Para

obtermos a forma contrapositiva a partir da forma positiva de uma

proposição condicional podemos fazer primeiro sua forma recíproca e

em seguida negamos as sentenças antecedente e consequente da recí-

proca ou, também, podemos primeiro negar as sentenças antecedente

e consequente da forma positiva e imediatamente fazer a forma recí-

proca desta última. A forma contrapositiva também é conhecida como

forma contrarrecíproca. Usando novamente nosso exemplo temos que:

Forma da Proposição Hipótese Tese

Positiva como laranja gosto de frutas

Recíproca gosto de frutas como laranja

Contrapositiva não gosto de frutas não como laranja

Portanto, a forma contrapositiva escreve-se assim:

Page 16: Resolução de problemas

1.2 Verdadeiro ou Falso? 5

Se não gosto de fruta, então não como laranja

Em particular, a forma contrapositiva de uma proposição poderá ser,

eventualmente, uma forma indireta muito ecaz de vericar resultados

em Matemática.

1.2 Verdadeiro ou Falso?

Uma das coisas que distingue a Matemática das demais ciências natu-

rais é o fato de que um tema de Matemática é discutido utilizando-se

a lógica pura e, por conta disso, uma proposição em Matemática, uma

vez comprovada sua veracidade, é aceita como verdade irrefutável e

permanecerá assim através dos séculos. Por exemplo, até hoje usamos

o teorema de Tales do mesmo modo que foi usado antes de Cristo e

este fato continuará valendo eternamente.

Vamos ilustrar melhor essa diferença com um exemplo em Geo-

graa. Hoje, todos nós sabemos que a Terra tem aproximadamente

o formato de uma laranja, um pouco achatada nos polos. Porém,

na época de Pitágoras, um dos grandes temores dos navegadores era

encontrar o m do mundo. No pensamento de alguns destes aventu-

reiros, a Terra tinha o formato de um cubo, e uma vez chegando em

um dos seus extremos, o navio despencaria no vazio. Esse é um dos

muitos exemplos de como a concepção da natureza mudou ao longo

do tempo, transformando uma concepção verdadeira num período da

humanidade em algo completamente falso em outra época. Porém,

para nossa felicidade, isso não acontece na Matemática. Uma propo-

sição matemática ou é verdadeira ou é falsa e permanecerá assim para

sempre.

Page 17: Resolução de problemas

6 1 Primeiros Passos

Mas como saber se uma proposição é verdadeira ou falsa? A pri-

meira coisa que devemos fazer é tomar muito cuidado. As aparências

enganam ou, como diziam nossos avós, nem tudo que reluz é ouro.

O leitor, avisado disso, pense agora na seguinte pergunta:

Pergunta 1: Qual é a chance de que pelo menos duas pessoas num

ônibus com 44 passageiros façam aniversário no mesmo dia do ano?

Como já avisamos, o leitor deve ter cuidado ao responder à per-

gunta acima, pois podemos nos enganar muito facilmente. Por exem-

plo, podemos formular o seguinte argumento errado: o ano tem 365

dias e, como estou escolhendo um grupo de 44 (número muito pequeno

com respeito a 365) pessoas ao acaso, é claro que podemos responder

à pergunta com a seguinte armação:

Resposta intuitiva: A chance de que num grupo de 44 pessoas pelo

menos duas delas façam aniversário no mesmo dia do ano é pequena.

À primeira vista a resposta dada pode até parecer verdadeira, mas

com uma análise mais cuidadosa veremos que é completamente falsa.

Na verdade, a chance de que pelo menos duas pessoas do ônibus façam

aniversário no mesmo dia do ano é de cerca de 93%!

Quem não acreditar nisto pode fazer duas coisas: primeiro, ir a

sua sala de aula ou no seu ônibus escolar, que deve ter pelo menos

44 pessoas, e fazer o experimento ao vivo. Muito provavelmente você

deve conseguir duas pessoas que fazem aniversário no mesmo dia do

ano. Se você verica que existem duas pessoas que fazem aniversário

no mesmo dia do ano, não é por acaso, pois a chance de isso acontecer

é muito alta. Mas, cuidado! Isso não é uma prova matemática para

este fato. Para provar que este fato é verdadeiro você deve vericar

que se escolhermos ao acaso um grupo de 44 pessoas então com aproxi-

Page 18: Resolução de problemas

1.2 Verdadeiro ou Falso? 7

madamente 93% de chance, pelo menos duas delas fazem aniversário

no mesmo dia do ano!

Porém, se você faz o experimento e não encontra duas pessoas que

fazem aniversário no mesmo dia do ano (você seria muito azarado!),

não se desespere. Lembre-se de que se trata de algo que acontece com

chance de 93% e que pode não acontecer quando fazemos um teste.

Em qualquer um dos casos, para ter a certeza de que a proposição

é verdadeira o leitor deve demonstrá-la. Faremos isso no nal do

Capítulo

Vamos analisar agora outro fato aparentemente óbvio.

Pergunta 2: Num campeonato de futebol onde cada time joga a

mesma quantidade de jogos, cada vitória vale três pontos, o empate

vale um ponto e a derrota nenhum ponto. Em caso de empate, o

critério de desempate entre as equipes era o seguinte:

• A melhor equipe é aquela que tem mais vitórias.

Os organizadores decidiram passar a adotar o critério a seguir:

• A melhor equipe é aquela que tem mais derrotas.

Você acha que este último critério adotado é justo?

Com respeito a esta pergunta, o leitor deve ter respondido do se-

guinte modo:

Resposta: Um time que perdeu mais é pior que um que perdeu me-

nos; portanto, a mudança de critério é totalmente injusta. Acertamos

a sua resposta?

Na verdade, não houve mudança nenhuma de critério, ou seja,

ambos os critérios nos conduzem ao mesmo ganhador.

Page 19: Resolução de problemas

8 1 Primeiros Passos

Para ver isso rapidamente, lembre-se de que se a equipe A perdeu

mais que a equipe B e ainda assim empataram, então ela deve ter

ganho mais, para que no m do campeonato a equipe A ainda assim

conseguisse empatar com a equipe B. Vamos mostrar isso precisa-

mente. Sejam d1, e1, v1 o número de derrotas, empates e vitórias,

respectivamente, da equipe A. Do mesmo modo, sejam d2, e2, v2 o

número de derrotas, empates e vitórias, respectivamente, da equipe

B. Suponhamos que a equipe A obteve mais vitórias do que a equipe

B, ou seja, que v1 > v2. Como cada equipe jogou o mesmo número de

jogos, temos que

d1 + e1 + v1 = d2 + e2 + v2. (1.1)

Por outro lado, note que o número de pontos obtidos pela equipe A é

e1 + 3v1. Do mesmo modo, o número de pontos obtidos pela equipe

B é igual a e2 + 3v2. Como as duas empataram, temos que:

e1 + 3v1 = e2 + 3v2.

Ou ainda,

3(v1 − v2) = e2 − e1 ou v2 − v1 = −e2 − e1

3.

Como v1−v2 > 0, temos que e2−e1 > 0. Reescrevendo a equação (1.1),

temos que:

d1 − d2 = e2 − e1 + (v2 − v1) = e2 − e1 −e2 − e1

3=

2

3(e2 − e1).

Logo, temos que d1 − d2 > 0, pois e2 − e1 > 0. Isso signica que

A teve mais derrotas que B; logo, qualquer um dos dois critérios de

desempate usado nos leva à equipe vencedora.

Page 20: Resolução de problemas

1.3 Teoremas e Demonstrações 9

Assim, como estes dois exemplos mostram, ao depararmos com um

problema em Matemática, devemos ter cuidado ao tirar conclusões

apressadas para evitar que cometamos algum engano. Pode acontecer

que uma situação que é claramente falsa para um observador menos

atento, se mostre verdadeira quando fazemos uma análise mais crite-

riosa.

1.3 Teoremas e Demonstrações

Agora denimos o que entendemos por demonstração matemática de

uma proposição.

Uma demonstração em Matemática é o processo de raciocínio ló-

gico e dedutivo para checar a veracidade de uma proposição condici-

onal. Nesse processo são usados argumentos válidos, ou seja, aqueles

que concluam armações verdadeiras a partir de fatos que também

são verdadeiros.

Como exemplo de demonstração citamos a argumentação usada

para mostrar na segunda pergunta da seção anterior que os critérios

de desempate eram similares.

Sempre que, via uma demonstração, comprovemos a veracidade de

uma proposição passamos então a chamar esta de teorema. Assim, um

teorema é qualquer armação que possa ser vericada mediante uma

demonstração.

Alguns teoremas se apresentam na forma de uma proposição con-

dicional, isto é, uma sentença do tipo Se P , então Q ou implicativa

da forma P =⇒ Q. Nesse caso, a sentença P é chamada de hipótese

e a sentença Q é denominada de tese. Ou seja, a validade da hipótese

nos implica a veracidade da tese.

Page 21: Resolução de problemas

10 1 Primeiros Passos

Um exemplo de teorema é o famoso teorema de Pitágoras, cujo

enunciado diz o seguinte:

Teorema 1.1 (Teorema de Pitágoras). Num triângulo retângulo a

soma dos quadrados dos catetos é igual ao quadrado da hipotenusa.

Notemos que o teorema de Pitágoras não está enunciado na forma

condicional, mas pode ser reescrito nessa forma como:

Teorema 1.2 (Teorema de Pitágoras). Se T é um triângulo retângulo

de catetos a e b e hipotenusa c, então c2 = a2 + b2.

Observação 1.3. Em geral, é mais comum usar a palavra teorema

apenas para certas proposições que são de grande importância mate-

mática, chamando-se simplesmente de proposição ao resto das propo-

sições verdadeiras que admitem uma demonstração. Para uma discus-

são mais detalhada, recomendamos [8].

1.3.1 Métodos de Demonstração

Quando realizamos uma demonstração não existe um caminho único.

Dependendo do problema em questão podemos usar métodos dife-

rentes. A seguir ilustramos os seguintes três métodos:

• Demonstração direta.

• Demonstração por contraposição.

• Demonstração por redução ao absurdo.

Page 22: Resolução de problemas

1.3 Teoremas e Demonstrações 11

Demonstração Direta

A demonstração direta é aquela em que assumimos a hipótese como

verdadeira e através de uma série de argumentos verdadeiros e dedu-

ções lógicas concluímos a veracidade da tese.

γ

α

βa b

b

a

a

bc

Q

Figura 1.1: Figura auxiliar para a demonstração do teorema de Pitágoras

Um exemplo de demonstração direta é a que daremos a seguir,

para o teorema de Pitágoras enunciado anteriormente no Teorema

1.1. Com efeito, usando a gura acima temos que a área do quadrado

de lado a + b é a soma das quatro áreas dos triângulos retângulos

congruentes pelo critério lado-ângulo-lado (de catetos a e b) mais a

área do quadrilátero Q, o qual é um quadrado visto que cada um dos

seus lados coincide com a hipotenusa c dos triângulos retângulos de

catetos a e b e, além disso, cada um dos seus ângulos internos mede

γ = 180o − (α + β) = 180 − 90 = 90 (veja a Figura 1.1).

Portanto,

(a+ b)2 = 4 · ab2

+ c2,

de onde

a2 + 2ab+ b2 = 2ab+ c2,

e consequentemente

a2 + b2 = c2,

Page 23: Resolução de problemas

12 1 Primeiros Passos

como queríamos.

Demonstração por Contraposição

Este método é baseado no fato de que a veracidade de forma positiva

de uma proposição é equivalente à veracidade de sua forma contraposi-

tiva, podendo ser esta última, eventualmente, mais fácil de se provar.

Por exemplo, a armação

Se sou alagoano, então sou brasileiro

é equivalente à armação

Se não sou brasileiro, então não sou alagoano

Por exemplo, provemos a seguinte proposição:

Proposição 1.4. Se N2 é par, então N é par.

• Hipótese: N2 é par.

• Tese: N é par.

Desaamos o leitor a tentar mostrar esta proposição partindo da hipó-

tese e tentando concluir a tese. Note que podemos vericar que nossa

proposição é verdadeira para vários valores de N2 como na tabela a

seguir, mas isso não é uma prova matemática da nossa proposição.

N2 4 16 36 64 100 144

N 2 4 6 8 10 12

Page 24: Resolução de problemas

1.3 Teoremas e Demonstrações 13

Mesmo vericando para um bilhão de valores de N2, sempre nos

restariam números para serem vericados. Como nossas tentativas

de provar a forma positiva dessa proposição estão sendo frustradas,

apelaremos para mostrar a forma contrapositiva da mesma, isto é:

Proposição 1.5. Se N não é par, então N2 não é par.

Neste caso, temos:

• Hipótese: N não é par.

• Tese: N2 não é par.

Demonstração. Como estamos assumindo que N não é par, logo N

tem que ser ímpar, ou seja, existe p, número inteiro, tal queN = 2p+1.

Logo,

N2 = (2p+ 1)(2p+ 1)

= 4p2 + 2p+ 2p+ 1

= 4p2 + 4p+ 1

= 2(2p2 + 2p) + 1

= 2q + 1,

onde q = 2p2 + 2p. Logo, N2 = 2q + 1 é ímpar e concluímos assim

nossa prova.

Demonstração por Redução ao Absurdo

Este método é uma das ferramentas mais poderosas da Matemática.

O nome provém do latim reductio ad absurdum e também é conhecido

como método do terceiro excluído devido ao mesmo estar baseado na

Page 25: Resolução de problemas

14 1 Primeiros Passos

lei do terceiro excluído que diz o seguinte: uma armação que não

pode ser falsa, deverá ser consequentemente verdadeira.

De um modo geral, o roteiro que segue uma demonstração por

redução ao absurdo é o seguinte:

• Assumimos a validade da hipótese.

• Supomos que nossa tese é falsa.

• Usando as duas informações anteriores concluímos, através de

argumentos verdadeiros, uma armação falsa; como tal fato não

poderá ocorrer, então nossa tese deverá ser verdadeira.

Vamos mostrar como o método funciona na prática provando a

seguinte proposição:

Proposição 1.6. Seja x um número positivo, então x+ 1/x ≥ 2.

Destaquemos primeiramente a nossa hipótese e a nossa tese.

• Hipótese: x é um número positivo.

• Tese: x+ 1/x ≥ 2.

Demonstração. Seja x um número positivo e suponhamos que a tese

é falsa, isto é, x + 1x< 2. Usando que x > 0 e multiplicando por este

a desigualdade anterior, obtemos que

x2 + 1 < 2x.

Daí segue-se que x2 − 2x+ 1 < 0 é equivalente a (x− 1)2 < 0, já que

x2 − 2x + 1 = (x − 1)2, o que é impossível. Portanto, x + 1/x ≥ 2,

como desejávamos.

Page 26: Resolução de problemas

1.4 Algumas Dicas para Resolver Problemas 15

1.4 Algumas Dicas para Resolver Proble-

mas

Nesta seção, damos algumas regras gerais que consideramos impor-

tante ter em mente na hora de resolver um problema de Matemática.

Aplicaremos estas regras a alguns problemas interessantes para ilus-

trar a sua importância. Elas são:

R1) Ler bem o enunciado do problema e utilizar todas as informações

disponíveis.

R2) Fazer casos particulares ou casos mais simples de problemas si-

milares, para adquirir familiaridade com o problema.

R3) Mudar a representação do problema, transformando-o em um

problema equivalente.

R4) Usar a imaginação pesquisando caminhos alternativos. Extra-

polar os limites!

A seguir propomos vários problemas onde as regras anteriores são

muito úteis. O leitor deve tentar resolvê-los; mas se não conseguir

achar solução depois de muito tentar poderá então passar para a pró-

xima seção onde os solucionamos.

Problema 1.7. Ao encontrar uma velha amiga (A), durante uma

viagem de trem, um matemático (M) tem a seguinte conversa:

(M) Como vão os três lhos da senhora?

(A) Vão bem, obrigada!

Page 27: Resolução de problemas

16 1 Primeiros Passos

(M) Qual a idade deles mesmo?

(A) Vou lhe dar uma dica. O produto das idades deles é 36.

(M) Só com essa dica é impossível!

(A) A soma das idades deles é igual ao número de janelas deste

vagão.

(M) Ainda não sei!

(A) O mais velho toca piano!

(M) Agora eu sei!

Você é capaz de descobrir as idades dos três lhos da senhora?

Problema 1.8. Numa cesta encontram-se 9 moedas idênticas, sendo

que 8 delas têm o mesmo peso e uma moeda é mais leve que as demais.

Usando duas vezes uma balança de dois pratos, encontrar a moeda

mais leve.

Problema 1.9. Numa pequena ilha existem 5 pessoas de olhos azuis

e 5 pessoas de olhos verdes. Existe um grande tabu nesta ilha que é o

seguinte: se uma pessoa descobre que possui olhos azuis ela se suicida

à meia-noite do dia em que descobriu, pulando do alto da prefeitura.

Por conta disso, ninguém conversa sobre o assunto, olha para espelhos

ou vê seu reexo na água. Todos se cruzam diariamente e conhecem

os olhos de seus amigos. Numa manhã, um estrangeiro chegou à ilha

e reuniu as 10 pessoas para o seguinte pronunciamento:

Nesta ilha, existe uma pessoa de olhos azuis.

Pergunta-se:

Page 28: Resolução de problemas

1.4 Algumas Dicas para Resolver Problemas 17

(a) O que aconteceu com os habitantes da ilha?

(b) Que informação nova o estrangeiro trouxe?

Problema 1.10. Um viajante deseja se hospedar durante 31 dias num

hotel. Entretanto, percebe que está sem dinheiro e que a única coisa

que possui é uma corrente com 31 elos de ouro. Para pagar sua conta,

ele acertou com o gerente pagar um elo por dia, sem atrasar ou a-

diantar o pagamento, durante os 31 dias. O gerente pode dar troco em

elos. Depois ele deseja recuperar a corrente e por isso ele quer pagar

a conta cortando a corrente no menor número de pedaços. Quantos

cortes você conseguiria dar e pagar a conta?

Problema 1.11. Sabendo que em cada jogada o movimento do cavalo

consiste em se deslocar duas casas na horizontal e uma na vertical

ou duas na vertical e uma na horizontal, decidir se é possível sair

da conguração apresentada no tabuleiro (a) e chegar à conguração

apresentada no tabuleiro (b) da Figura 1.2 sem que em algum momento

existam dois cavalos na mesma casa.

(a) (b)Figura 1.2: Cavalos de xadrez

Page 29: Resolução de problemas

18 1 Primeiros Passos

Problema 1.12. Mostre que podemos cobrir os 9 pontos no reticulado

da Figura 1.3 traçando 4 segmentos de reta sem tirar o lápis do papel.

•••

•••

•••

Figura 1.3: Reticulado de 9 pontos

Sugerimos seguir as dicas abaixo para obter sucesso na solução dos

problemas:

• Para os problemas 1.7 e 1.8 use a primeira regra.

• Para os problemas 1.9 e 1.10 use a segunda regra. Por exemplo,

no problema 1.9 fazer primeiro o caso: uma pessoa com olhos

azuis e uma com olhos verdes e depois fazer o caso: duas pessoas

de olhos azuis e duas de olhos verdes; generalize.

• Para os problema 1.11 use a terceira regra.

• Para o problema 1.12 use a quarta regra.

1.5 Soluções dos Problemas da Seção 1.4

A seguir apresentamos soluções para os problemas enunciados na seção

anterior.

Solução do Problema 1.7. É muito importante neste problema tirar

o máximo de informação das dicas da senhora. Vamos à primeira dica:

o produto das idades é 36.

Page 30: Resolução de problemas

1.5 Soluções dos Problemas da Seção 1.4 19

Suponhamos que as idades dos lhos sejam 0 6 x 6 y 6 z 6 36.

Como xyz = 36, temos as seguintes possibilidades para os números x,

y e z:x y z xyz

1 1 36 36

1 2 18 36

1 3 12 36

1 4 9 36

1 6 6 36

2 2 9 36

2 3 6 36

3 3 4 36

A segunda dica dada pela senhora é a soma das idades. Assim,

vamos agora calcular todas as possíveis somas de acordo com as fato-

rações de 36 dadas na tabela anterior:

x y z x+ y + z

1 1 36 38

1 2 18 21

1 3 12 16

1 4 9 14

1 6 613

2 2 913

2 3 6 11

3 3 4 10

Sabemos que após a segunda dica, o matemático ainda não conse-

guiu deduzir as idades das crianças.

Page 31: Resolução de problemas

20 1 Primeiros Passos

Por que ele não conseguiu? Imagine que o número da casa fosse

14. Ora, de acordo com nossa tabela, só existe um terno de números

cujo produto é 36 e a soma é 14, que é o terno (1,4,9). Assim, se o

número da casa fosse 14 o matemático teria dado a resposta após a

segunda dica. Como ele cou em dúvida, olhando a tabela 2, chegamos

à conclusão de que o número da casa só pode ser igual a 13.

Lembremos a última dica: o mais velho toca piano. No início essa

dica parecia inútil, mas agora ela é fundamental para resolvermos o

problema. De fato, como o mais velho toca piano, isso signica que

existe um mais velho, o que descarta o caso (1,6,6). Assim, as idades

são 2, 2, e 9.

Solução do Problema 1.8. Este é o tipo de problema que a primeira

vista pode parecer difícil, mas que quando usamos todas as informa-

ções do seu enunciado se torna fácil. A ideia é dividir as moedas em

três grupos de três moedas cada, que chamaremos grupos A, B e C.

Colocaremos na balança os grupos A e B e deixaremos o grupo C fora.

Podem acontecer duas coisas:

(a) Os pratos cam equilibrados.

(b) Os pratos cam desequilibrados.

No caso (a), temos que os grupos A e B têm o mesmo peso. Logo,

a moeda mais leve deve estar no grupo C. No caso (b), um dos grupos

cou mais leve, o que signica que a moeda mais leve está neste grupo.

Assim, utilizando a balança apenas uma vez conseguiremos descobrir

qual é o grupo em que a moeda mais leve está. Digamos que este grupo

seja o grupo A. Para achar a moeda mais leve, procedemos de modo

semelhante ao que zemos anteriormente: separamos as três moedas

Page 32: Resolução de problemas

1.5 Soluções dos Problemas da Seção 1.4 21

do grupo A colocando uma em cada prato e deixando a terceira de

fora. Podem acontecer duas coisas:

(a) Os pratos cam desequilibrados e assim a moeda mais leve está

no prato mais leve.

(b) Os pratos cam equilibrados, logo a moeda mais leve foi a que

cou fora.

No nal, usamos a balança exatamente duas vezes.

Solução do Problema 1.9. Como em muitos problemas de Mate-

mática, abordar casos mais simples do problema pode ajudar bastante

na solução. Assim, vamos imaginar o seguinte caso mais simples:

na ilha existe somente uma pessoa de olhos azuis e a outra de olhos

verdes. Pensando neste caso, a pessoa que tinha olhos azuis só via as

que tinham olhos verdes. Quando o estrangeiro armou que existia

uma pessoa de olhos azuis, ela descobriu que tinha olhos azuis, pois as

outras pessoas tinham olhos verdes. Assim, à meia-noite ela subiu na

prefeitura e pulou. Com isso, a pessoa que tinha olhos verdes descobriu

que tinha olhos verdes, pois se ela tivesse olhos azuis sua companheira

não se suicidaria no dia anterior.

Vamos agora dar um passo crucial na solução do nosso problema

original, considerando o caso onde existem duas pessoas de olhos azuis

e duas pessoas de olhos verdes na ilha. Vamos chamar as pessoas de

olhos azuis de A e B e as pessoas de olhos verdes de C e D. No dia

em que o estrangeiro fez o seu pronunciamento, nada aconteceu, pois

as pessoas C e D viam as pessoas A e B com olhos azuis e a pessoa

A via a pessoa B com olhos azuis e vice-versa. Já no segundo dia, a

pessoa A teve o seguinte pensamento:

Page 33: Resolução de problemas

22 1 Primeiros Passos

Se eu tivesse olhos verdes, a pessoa B teria descoberto que

tinha olhos azuis ontem, pois ela veria três pessoas de olhos

verdes. Como ela não se suicidou ontem, eu tenho olhos

azuis.

Pensando da mesma forma, a pessoa B descobriu que também tinha

olhos azuis. Por isso, à meia-noite do segundo dia, as pessoas A e B

se suicidaram.

O que aconteceu depois? As pessoas C e D ainda tinham a dúvida

da cor de seus olhos. Para chegar à conclusão de que seus olhos são

verdes, no terceiro dia, a pessoa C pensou assim:

Bem, se eu tivesse olhos azuis, as pessoas A e B veriam

cada uma duas pessoas com olho azul. Logo, elas não te-

riam se suicidado no segundo dia, pois não conseguiriam

deduzir a cor de seus olhos. Logo, tenho olhos verdes.

Ufa!

Do mesmo modo, a pessoa D conseguiu descobrir a cor de seus olhos.

Analisando de modo semelhante, conseguiremos deduzir que no

problema original as cinco pessoas de olhos azuis descobrirão que pos-

suem olhos azuis e juntas se suicidarão no quinto dia após o pronun-

ciamento do estrangeiro.

Agora vamos descobrir a resposta da segunda pergunta do enun-

ciado: que informação nova o estrangeiro trouxe? Aparentemente

nada de novo foi acrescentado pela frase do estrangeiro, pois cada

pessoa estava vendo alguma pessoa com olhos azuis. Mas isso não é

verdade.

Para ver isso e descobrir qual é a nova informação que o estrangeiro

trouxe, vamos voltar ao caso de somente duas pessoas na ilha, uma

Page 34: Resolução de problemas

1.5 Soluções dos Problemas da Seção 1.4 23

de olhos azuis e outra de olhos verdes. Neste caso, a pessoa de olhos

azuis somente vê uma pessoa de olhos verdes. Com a informação de

que existe uma pessoa de olhos azuis ela pode descobrir a cor de seus

olhos. Note que a pessoa de olhos verdes já sabia que existia pelo

menos uma pessoa de olhos azuis. Mas ela não sabia que a pessoa

de olhos azuis tinha conhecimento de que na ilha existia alguém com

olhos azuis. Essa é a nova informação que o estrangeiro trouxe.

Solução do Problema 1.10. Uma primeira solução é cortar a cor-

rente 30 vezes, separando todos os elos. Porém, essa não é a melhor so-

lução, como veremos a seguir. Vamos iniciar nossa análise observando

que para pagar o primeiro dia precisamos dar um corte na corrente.

Assim, o gerente receberá um elo. O pulo do gato do problema vem

agora: para pagar o 2 dia, vamos cortar a corrente de modo a separar

dois elos de uma vez. Assim, daremos dois elos ao gerente e ele de-

volverá um elo de troco. Com este elo pagaremos o terceiro dia. Note

que pagamos três dias fazendo dois cortes na corrente, como mostra a

tabela:

Gerente Viajante

Elos 1, 2 28

Note que o número 2 denota o pedaço que contém 2 elos. Para

pagar o 4 dia, cortaremos a corrente de modo a obter um pedaço

com quatro elos. Entregamos ao gerente este pedaço e recebemos

de troco um elo solto e um pedaço com dois elos. Com o elo solto,

pagamos o 5 dia. Assim, no 5 dia teremos os seguintes grupos de

elos:

Gerente Viajante

Elos 1, 4 2, 24

Page 35: Resolução de problemas

24 1 Primeiros Passos

Assim, pagamos o 6 dia com o pedaço que contém dois elos e

receberemos o elo solto de troco. Finalmente pagaremos o 7 dia com

o elo solto. Note que foi possível pagar 7 dias com apenas três cortes na

corrente. A continuação do procedimento está quase revelada. Para

pagar o 8 dia, cortaremos um pedaço com oito elos. Daremos este

pedaço e receberemos de troco 7 elos, sendo um elo solto, um pedaço

com 4 e um pedaço com dois elos. Repetindo o procedimento anterior,

pagaremos os 7 dias seguintes, pagando até o 15 dia sem precisar de

cortes adicionais. Ou seja, para pagar os 15 primeiros dias, precisamos

de 4 cortes na corrente. Neste momento, a corrente está distribuída

do seguinte modo:

Gerente Viajante

Elos 1, 2, 4, 8 16

Para pagar o 16 dia, entregaremos ao gerente o pedaço com os 16

elos restantes, recebendo 15 elos divididos em pedaços de 1, 2, 4 e 8

elos. Se repetirmos o processo, pagaremos o hotel até o 31 dia sem

precisar de novos cortes. Assim, o mínimo número de cortes é 4.

Solução do Problema 1.11. Para resolver este problema vamos usar

a estratégia de mudar a representação. O que signica isso? Vamos

reescrever o problema com outros ingredientes, porém sem alterar em

nada sua essência. Primeiramente, enumere as casas do tabuleiro com

os números 1, 2, . . . , 9, como na Figura 1.4.

Vamos agora associar ao tabuleiro, um conjunto de nove pontos

também enumerados com os números 1, 2, . . . , 9. Se for possível sair

de uma casa i e chegar à casa j com apenas uma jogada do cavalo,

colocaremos um segmento ligando os pontos i e j. Por exemplo, é

Page 36: Resolução de problemas

1.5 Soluções dos Problemas da Seção 1.4 25

1 2 3

4 5 6

7 8 9

Figura 1.4: Tabuleiro de 9 casas

possível, saindo da casa 1 chegar à casa 6 e a casa 8. Desse modo,

o ponto com número 1 está ligado com o ponto com número 8. Se

analisarmos todas as possíveis ligações entre os pontos obteremos um

esquema com o mostrado na Figura 1.5

•5• •1 86 34927

•••

Figura 1.5: Conexões das casas

•5• •1 86 34927

•••

Figura 1.6: Tabuleiro (a)

Assim, se colocarmos os cavalos como no tabuleiro (a), teremos

a situação descrita na Figura 1.6. Deste modo, ca evidente que não

podemos trocar a posição dos cavalos branco e preto sem que em algum

momento eles ocupem a mesma casa.

Page 37: Resolução de problemas

26 1 Primeiros Passos

1.6 Exercícios

1. Uma sacola contém meias cujas cores são branca, preta, amarela

e azul. Sem olhar para a sacola, qual é a quantidade mínima de

meias que precisamos retirar da mesma para garantir pelo menos

um par de meias da mesma cor?

2. O pai do padre é lho único de meu pai. O que eu sou do padre?

3. Numa mesa há 5 cartas:

Q T 3 4 6

Cada carta tem de um lado um número natural e do outro lado

uma letra. João arma: Qualquer carta que tenha uma vogal

tem um número par do outro lado. Pedro provou que João

mente virando somente uma das cartas. Qual das 5 cartas foi a

que Pedro virou?

4. A polícia prende 4 homens, um dos quais é culpado de um furto.

Eles fazem as seguintes declarações:

• Arnaldo: Bernaldo é o culpável.

• Bernaldo: Cernaldo é o culpável.

• Dernaldo: eu não sou culpável.

• Cernaldo: Bernaldo mente ao dizer que eu sou culpável.

Se se sabe que só uma destas declarações é a verdadeira, quem

é culpável pelo furto?

Page 38: Resolução de problemas

1.6 Exercícios 27

5. Numa cidade existe uma pessoa X que sempre mente terças,

quintas e sábados e é completamente sincera o resto dos dias

da semana. Felipe chega um certo dia na cidade e mantém o

seguinte diálogo com a pessoa X:

Felipe: Que dia é hoje?

X: Sábado.

Felipe: Que dia será amanhã?

X: Quarta-feira.

Em qual dia da semana foi mantido este diálogo?

6. Divida o relógio de parede abaixo em 6 partes iguais de forma tal

que a soma das horas que cam em cada parte seja a mesma.n12 1 234567891011 •

7. João adora Gabriela, que é uma aluna excelente em Matemática.

João armou um plano para dar um beijo nela, e descobriu que

poderá fazer isso apenas dizendo uma frase. Que frase é essa?

8. No plano se colocam 187 rodas dentadas do mesmo diâmetro,

enumeradas de 1 até 187. A roda 1 é acoplada com a roda 2, a 2

com a 3, . . . , a 186 com a 187 e esta última com a roda 1. Pode

tal sistema girar?

Page 39: Resolução de problemas

28 1 Primeiros Passos

9. Um canal, em forma quadrada, de 4 metros de largura rodeia um

castelo. A ponte do castelo está fechada e um intruso quer entrar

no castelo usando duas pranchas de 3,5 metros de comprimento.

Será que o intruso consegue?

10. Os números 1, 2, 3, . . . , 99 são escritos no quadro-negro e é permi-

tido realizar a seguinte operação: apagar dois deles e substituí-

los pela diferença do maior com o menor. Fazemos esta operação

sucessivamente até restar apenas um último número no quadro.

Pode o último número que restou ser o zero?

11. Alguém elege dois números, não necessariamente distintos, no

conjunto de números naturais 2, . . . , 20. O valor da soma destes

números é dado somente a Adriano (A) e o valor do produto dos

números é dado unicamente a Karla (K).

Pelo telefone A diz a K: Não é possível que descubras minha

soma.

Uma hora mais tarde, K lhe diz a A: Ah! sabendo disso, já

sei quanto vale tua soma!

Mais tarde A chama outra vez a K e lhe informa: Poxa,

agora eu também conheço teu produto!

Quais números foram eleitos?

12. É possível cobrir um tabuleiro de xadrez com 31 dominós onde

removemos as casas dos vértices superior esquerdo e inferior di-

reito?

13. Num saco encontram-se 64 moedas leves e 64 moedas pesadas.

Page 40: Resolução de problemas

1.6 Exercícios 29

É possível separar duas moedas de pesos diferentes com 7 pesa-

gens?

14. Quantas vezes precisamos dobrar um papel de 1mm de espessura

para que a altura da pilha chegue da Terra à Lua?

15. Descubra o erro da prova da armação abaixo:

Armação: Três é igual a dois.

Seja x um número diferente de zero. Temos que:

3x− 3x = 2x− 2x.

Colocando x− x em evidência, temos que:

3(x− x) = 2(x− x).

Cancelando x− x em ambos os lados, obtemos que 3 = 2.

Page 41: Resolução de problemas

30 1 Primeiros Passos

Page 42: Resolução de problemas

2

Equações e Inequações

Álgebra é generosa; ela geralmente nos dá mais do que lhe pedimos.D'AlembertNa antiguidade, todo conhecimento matemático era passado de

geração para geração através de receitas. A falta de símbolos e notação

adequada complicava substancialmente a vida de quem precisava usar

a Matemática e de quem apreciava sua beleza. Por exemplo, o uso de

letras (x, y, z etc.) para representar números desconhecidos não tinha

sido inventado ainda. Isso só veio ocorrer por volta dos meados do

século XVI, ou seja, a menos de 500 anos atrás.

Apesar disso, o conhecimento matemático das antigas civilizações

era surpreendente. Os egípcios, babilônios, gregos e vários outros po-

vos tinham o domínio de métodos e técnicas que são empregados hoje,

como soluções de equações do primeiro e segundo graus, inteiros que

são soma de quadrados e vários outros conhecimentos. Especialmente

os gregos, cuja cultura matemática resistiu aos tempos com a preser-

vação de Os Elementos de Euclides, tinham desenvolvido e catalisado

31

Page 43: Resolução de problemas

32 2 Equações e Inequações

muitos dos avanços da época.

Entretanto, todos os resultados tinham uma linguagem através dos

elementos de geometria, mesmo aqueles que só envolviam proprieda-

des sobre os números. Essa diculdade deve-se em parte aos sistemas

de numeração que eram utilizados pelos gregos e, posteriormente, pe-

los romanos, que eram muito pouco práticos para realizar operações

matemáticas.

Por volta de 1.100, viveu na Índia Bhaskara, um dos mais impor-

tantes matemáticos de sua época. Apesar de suas contribuições terem

sido muito profundas na Matemática, incluindo-se aí resultados sobre

equações diofantinas, tudo indica que Bhaskara não foi o primeiro a

descobrir a fórmula, que no Brasil chamamos de fórmula de Bhaskara,

assim como Pitágoras não deve ter sido o primeiro a descobrir o te-

orema que leva o seu nome, já que 3.000 a.c. os babilônios tinham

conhecimento de ternas pitagóricas de números inteiros bem grandes.

Apesar de ter conhecimento de como solucionar uma equação do

segundo grau, a fórmula que Bhaskara usava não era exatamente igual

a que usamos hoje em dia, sendo mais uma receita de como encontrar

as raízes de uma equação. Para encontrar essas raízes, os indianos

usavam a seguinte regra:

Multiplique ambos os membros da equação pelo número que vale

quatro vezes o coeciente do quadrado e some a eles um número igual

ao quadrado do coeciente original da incógnita. A solução desejada

é a raiz quadrada disso.

O uso de letras para representar as quantidades desconhecidas só

veio a se tornar mais popular com os árabes, que também desenvol-

veram um outro sistema de numeração, conhecido como indo-arábico.

Destaca-se também a participação do matemático francês François

Page 44: Resolução de problemas

2.1 Equações do Primeiro Grau 33

Vièti, que aprimorou esse uso dos símbolos algébricos em sua obra In

artem analyticam isagoge e desenvolveu um outro método para resol-

ver a equação do segundo grau.

Na seção seguinte estudaremos com detalhe a equação do primeiro

grau, e como podemos utilizá-la para resolver alguns problemas em

Matemática.

2.1 Equações do Primeiro Grau

Iniciamos nossa discussão resolvendo o seguinte problema:

Exemplo 2.1. Qual é o número cujo dobro somado com sua quinta

parte é igual a 121?

Solução: Vamos utilizar uma letra qualquer, digamos a letra x, para

designar esse número desconhecido. Assim, o dobro de x é 2x e sua

quinta parte é x/5. Logo, usando as informações do enunciado, obte-

mos que:

2x+x

5= 121,

ou ainda,

10x+ x = 605,

onde 11x = 605. Resolvendo, temos que x = 605/11 = 55.

Se você já teve contato com o procedimento de resolução do exem-

plo acima, notou que o principal ingrediente é a equação do primeiro

grau em uma variável.

Denição 2.2. Uma equação do primeiro grau na variável x é uma

expressão da forma

ax+ b = 0,

Page 45: Resolução de problemas

34 2 Equações e Inequações

onde a 6= 0, b ∈ R e x é um número real a ser encontrado.

Por exemplo, as seguintes equações são do primeiro grau:

(a) 2x− 3 = 0.

(b) −4x+ 1 = 0.

(b)3

2x− π = 0.

Para trabalhar com equações e resolvê-las, vamos pensar no mo-

delo da balança de dois pratos. Quando colocamos dois objetos com

o mesmo peso em cada prato da balança, os pratos se equilibram.

Quando os pratos estão equilibrados, podemos adicionar ou retirar a

mesma quantidade de ambos os pratos, que ainda assim eles perma-

necerão equilibrados. Essa é uma das principais propriedades quando

estamos trabalhando com uma equação. Em geral, para resolver uma

equação, utilizamos as seguintes propriedades da igualdade entre dois

números:

Propriedade 1. Se dois números são iguais, ao adicionarmos a

mesma quantidade a cada um destes números, eles ainda permane-

cem iguais. Em outras palavras, escrevendo em termos de letras, se a

e b são dois números iguais, então a+ c é igual a b+ c, ou seja,

a = b =⇒ a+ c = b+ c.

Note que podemos tomar c um número negativo, o que signica

que estamos subtraindo a mesma quantidade dos dois números. Por

exemplo, se x é um número qualquer que satisfaz

5x− 3 = 6,

Page 46: Resolução de problemas

2.1 Equações do Primeiro Grau 35

somando-se 3 a ambos os lados da equação acima, obtemos que x deve

satisfazer:

(5x− 3) + 3 = 6 + 3, ou seja, 5x = 9.

Propriedade 2. Se dois números são iguais, ao multiplicarmos a

mesma quantidade por cada um destes números, eles ainda permane-

cem iguais. Em outras palavras, escrevendo em termos de letras, se a

e b são dois números iguais, então a · c é igual a b · c, ou seja,

a = b =⇒ ac = bc.

Por exemplo, se 5x = 9 podemos multiplicar ambos os lados da

igualdade por 1/5 para obter

x =5x

5=

9

5,

encontrando o número que satisfaz a equação 5x− 3 = 6.

Para nos familiarizarmos um pouco mais com a linguagem das

equações, vamos pensar no seguinte problema:

Exemplo 2.3. Para impressionar Pedro, Lucas propôs a seguinte

brincadeira:

- Escolha um número qualquer.

- Já escolhi, disse Pedro.

- Multiplique este número por 6. A seguir, some 12. Divida o que

você obteve por 3. Subtraia o dobro do número que você escolheu. O

que sobrou é igual a 4!

Pedro realmente cou impressionado com a habilidade de Lucas.

Mas não há nada de mágico nisso. Você consegue explicar o que Lucas

fez?

Page 47: Resolução de problemas

36 2 Equações e Inequações

Solução: Na verdade, Lucas tinha conhecimento de como operar com

equações. Vamos ver o que Lucas fez de perto, passo a passo, utili-

zando a linguagem das equações. Para isso, vamos chamar a quanti-

dade que Pedro escolheu de x:

• Escolha um número: x.

• Multiplique este número por 6: 6x.

• A seguir, some 12: 6x+ 12.

• Divida o que você obteve por 3:6x+ 12

3= 2x+ 4.

• Subtraia o dobro do número que você escolheu: 2x+ 4−2x = 4.

• O que sobrou é igual a 4!

Observação 2.4. Devemos ter cuidado na hora de efetuar divisões em

ambos os lados de uma equação, para não cometer o erro de dividir

os lados de uma igualdade por zero. Por exemplo, podemos dar uma

prova (obviamente) falsa de que 1 = 2, utilizando o seguinte tipo de

argumento: sempre é verdade que

x+ 2x = 2x+ x.

Logo,

x− x = 2x− 2x

Colocando (x− x) em evidência:

1(x− x) = 2(x− x)

Dividindo por (x − x) os dois lados da igualdade acima, temos que

1 = 2. Qual o erro?

Page 48: Resolução de problemas

2.1 Equações do Primeiro Grau 37

Para encontrar a solução da equação ax + b = 0, procedemos do

seguinte modo:

• Somamos −b a ambos os lados da equação, obtendo

ax+ b+ (−b) = 0 + (−b)⇐⇒ ax = −b.

Note que como somamos a mesma quantidade aos dois lados da

equação, ela não se alterou.

• Dividimos os dois lados da equação por a 6= 0. Isso também não

altera a igualdade e nos dá que o valor de x é:

ax

a=−ba⇐⇒ x = − b

a.

Assim, a solução da equação ax+ b = 0 é

x = − b

a.

2.1.1 Problemas Resolvidos

Vamos ver agora alguns problemas que podem ser solucionados utili-

zando as equações do primeiro grau.

Problema 2.5. Se x representa um dígito na base 10 e a soma

x11 + 11x+ 1x1 = 777,

quem é x?

Page 49: Resolução de problemas

38 2 Equações e Inequações

Solução: Para resolver este problema, precisamos nos recordar que se

abc é a escrita de um número qualquer na base 10, então esse número

é igual a 102a+ 10b+ c. Assim, temos que

x11 = 100x+ 11

11x = 110 + x

1x1 = 101 + 10x

Logo, temos a seguinte equação do primeiro grau:

100x+ 11 + 110 + x+ 101 + 10x = 777 ou 111x+ 222 = 777

Logo,

x =777− 222

111= 5.

Problema 2.6. Determine se é possível completar o preenchimento

do tabuleiro abaixo com os números naturais de 1 a 9, sem repetição,

de modo que a soma de qualquer linha seja igual a de qualquer coluna

ou diagonal.

1 6

9

Solução: Primeiro, observe que a soma de todos os números naturais

de 1 a 9 é 45. Assim, se denotamos por s o valor comum da soma dos

elementos de uma linha, somando as três linhas do tabuleiro, temos

que:

45 = 1 + 2 + · · ·+ 9 = 3s,

Onde s deve ser igual a 15. Assim, chamando de x o elemento da

primeira linha que falta ser preenchido,

Page 50: Resolução de problemas

2.1 Equações do Primeiro Grau 39

1 x 6

9

temos que 1 + x + 6 = 15. Logo, x = 8. Assim, observando a coluna

que contém 8 e 9, temos que sua soma é maior que 15. Logo, não é

possível preencher o tabuleiro de modo que todas as linhas e colunas

tenham a mesma soma.

Os quadrados de números com essas propriedades se chamam qua-

drados mágicos. Tente fazer um quadrado mágico. Você já deve ter

percebido que no centro do quadrado não podemos colocar o número

9. De fato, vamos descobrir no exemplo abaixo qual é o número que

deve ser colocado no centro de um quadrado mágico.

Problema 2.7. Descubra os valores de x de modo que seja possível

completar o preenchimento do quadrado mágico abaixo:

x

Solução: Para descobrir x, vamos utilizar o fato de que a soma de

qualquer linha, coluna ou diagonal é igual a 15, já obtido no exemplo

anterior. Se somarmos todas as linhas, colunas e diagonais que contêm

x, teremos que a soma será 4 · 15 = 60, pois existem exatamente uma

linha, uma coluna e duas diagonais que contêm x. Note também que

cada elemento do quadrado mágico será somado exatamente uma vez,

exceto x que será somado quatro vezes. Assim:

1 + 2 + 3 + 4 + · · ·+ 9 + 3x = 60,

onde temos que 45 + 3x = 60 e consequentemente x = 5.

Page 51: Resolução de problemas

40 2 Equações e Inequações

O problema a seguir é um fato curioso que desperta nossa atenção

para como a nossa intuição às vezes é falha.

Problema 2.8. Imagine que você possui um o de cobre extrema-

mente longo, mas tão longo que você consegue dar a volta na Terra

com ele. Para simplicar a nossa vida e nossas contas, vamos supor

que a Terra é uma bola redonda (o que não é exatamente verdade)

sem nenhuma montanha ou depressão e que seu raio é de exatamente

6.378.000 metros.

O o com seus milhões de metros está ajustado à Terra, cando

bem colado ao chão ao longo do equador. Digamos agora que você

acrescente 1 metro ao o e o molde de modo que ele forme um círculo

enorme, cujo raio é um pouco maior que o raio da Terra e tenha o

mesmo centro. Você acha que essa folga será de que tamanho?

Nossa intuição nos leva a acreditar que como aumentamos tão

pouco o o, a folga que ele vai ter será também muito pequena, di-

gamos alguns poucos milímetros. Mas veremos que isso está comple-

tamente errado!

Solução. Utilizaremos para isso a fórmula que diz que o comprimento

C de um círculo de raio r é

C = 2πr,

onde π (lê-se pi) é um número irracional que vale aproximadamente

3, 1416 (veja a observação a seguir).

De fato, o comprimento da Terra CT calculado com essa fórmula é

aproximadamente:

CT = 2πrT ∼= 2× 3, 1415× 6.378.000 = 40.072.974 metros,

Page 52: Resolução de problemas

2.1 Equações do Primeiro Grau 41

onde rT é o raio da Terra.

Se chamamos de x o tamanho da folga obtida em metros e rf o raio

do o, temos que a folga será igual a x = rf −rT . Logo, basta calcularrf . Por um lado, o comprimento do o é igual a CT + 1 = 40.072.975.

Logo,

40.072.975 = 2πrf onde rf =40.072.975

2π.

Fazendo o cálculo acima, temos que rf é aproximadamente igual

a 6.378.000, 16 metros. Assim, x é aproximadamente igual a x =

rf − rT = 0, 16 metros, ou seja, 16 centímetros!

Observação 2.9. Vale observar que a folga obtida aumentando o o

não depende do raio em consideração. Por exemplo, se repetíssemos

esse processo envolvendo a Lua em vez da Terra, obteríamos que ao

aumentar o o em um metro, a folga obtida seria dos mesmos 16

centímetros. Verique isso!

Observação 2.10. De fato, podemos denir (e calcular!) o número

π de várias maneiras práticas. Vamos considerar dois experimentos

(que se você não conhece π deve fazer):

Experimento 1: Pegar um cinto e fazer um círculo com ele. Calcule

o comprimento do cinturão e divida pelo diâmetro do círculo obtido.

Experimento 2: Pegar uma tampa de uma lata e medir o compri-

mento do círculo da tampa e dividir pelo diâmetro da tampa.

Se você efetuou os cálculos acima com capricho, deve ter notado

que o número obtido é aproximadamente o mesmo. Se nossos círculos

fossem perfeitos (eles nunca são: sempre têm algumas imperfeições)

obteríamos o número π. Uma aproximação para π é

π ∼= 3, 1415926535897932384626433832795.

Page 53: Resolução de problemas

42 2 Equações e Inequações

2.2 Sistemas de Equações do Primeiro Grau

Nesta seção iremos discutir situações onde queremos descobrir mais de

uma quantidade, que se relacionam de modo linear, ou seja, através

de equações do primeiro grau. Por exemplo, considere o seguinte pro-

blema:

Exemplo 2.11. João possui 14 reais e deseja gastar esse dinheiro em

chocolates e sanduíches para distribuir com seus 6 amigos, de modo

que cada um que exatamente com um chocolate ou um sanduíche.

Sabendo que cada chocolate custa 2 reais e cada sanduíche custa 3

reais, quantos chocolates e sanduíches João deve comprar?

Para resolver esse problema, vamos chamar de x a quantidade de

chocolates que João deve comprar e y o número de sanduíches. Assim,

como João deseja gastar 14 reais, temos que

2x+ 3y = 14. (2.1)

Como João comprará exatamente 6 guloseimas, uma para cada amigo,

temos que

x+ y = 6. (2.2)

Note que não encontramos uma equação do primeiro grau em uma

variável e sim duas equações do primeiro grau em duas variáveis. Esse

é um caso particular de um sistema de equações do primeiro grau em

várias variáveis.

Denição 2.12. Uma equação do primeiro grau nas variáveis x1, x2,

. . . , xn é uma expressão da forma

a1x1 + a2x2 + · · ·+ anxn + b = 0,

Page 54: Resolução de problemas

2.2 Sistemas de Equações do Primeiro Grau 43

onde os números a1, a2, . . . , an são diferentes de zero e b é um número

real.

Por exemplo,

2x− 3y = 0

é uma equação do primeiro grau nas variáveis x e y. Assim como,

2a− b+c

3= 5

é uma equação do primeiro grau nas variáveis a, b e c.

Dizemos que os números (r1, r2, . . . , rn) formam uma solução da

equação, se substituindo x1 por r1, x2 por r2, . . . , xn por rn, temos

que a equação acima é satisfeita, isto é, a1r1 +a2r2 + · · ·+anrn+b = 0.

Por exemplo, (3, 2) é uma solução da equação 2x− 3y = 0 acima,

pois

2 · 3− 3 · 2 = 0.

Note que a ordem que apresentamos os números importa, pois (2, 3)

não é solução da equação 2x − 3y = 0, já que 2 · 2 − 3 · 3 = −5 6= 0.

Do mesmo modo, (2, 0, 3) é solução da equação 2a− b+c

3= 5, pois

2 · 2− 0 +3

3= 5.

Denição 2.13. Um sistema de equações do primeiro grau em n

variáveis x1, x2, . . ., xn é um conjunto de k equações do primeiro

grau em algumas das variáveis x1, x2, . . . , xn, isto é, tem-se o seguinte

conjunto de equaçõesa11x1 + a12x2 + · · ·+ a1nxn + b1 = 0,

a21x1 + a22x2 + · · ·+ a2nxn + b2 = 0,

· · · · · · · · · · · · · · · · · · · · · · · · · · · · · · · · · · ·ak1x1 + ak2x2 + · · ·+ aknxn + bk = 0,

(2.3)

Page 55: Resolução de problemas

44 2 Equações e Inequações

onde alguns dos elementos aij (1 ≤ i ≤ k, 1 ≤ j ≤ n) podem ser zero.

Porém, em cada uma das equações do sistema algum coeciente aij é

diferente de zero e, além disso, cada variável xj aparece em alguma

equação com coeciente distinto de zero.

Dizemos que os números (r1, r2, . . . , rn) formam uma solução do

sistema de equações (2.3) se (r1, r2, . . . , rn) é solução para todas as

equações simultaneamente.

Quando resolvemos um sistema de equações do primeiro grau, po-

dem acontecer três situações:

(a) o sistema tem uma única solução;

(b) o sistema tem uma innidade de soluções;

(c) o sistema não possui solução.

A seguir ilustramos com exemplos cada uma das situações acima.

Situação (a): Retomamos o sistema proposto no Exemplo 2.11, o

qual se encaixa neste caso.2x+ 3y = 14,

x+ y = 6.

Isolamos o valor de uma das variáveis numa das equações. Por conveni-

ência nos cálculos isolamos o valor de x na segunda equação, obtendo:

x = 6− y.

A seguir, substituímos esse valor na outra equação, obtendo uma equa-

Page 56: Resolução de problemas

2.2 Sistemas de Equações do Primeiro Grau 45

ção do primeiro grau. Resolvendo temos:

2(6− y) + 3y = 14,

12− 2y + 3y = 14,

y = 2.

Assim, y = 2. Imediatamente, encontramos o valor de x = 6− 2 = 4.

Vamos agora resolver alguns problemas semelhantes.

Situação (b): Consideremos os sistema de primeiro grau nas variáveis

x, y e z dado por x+ y − z − 1 = 0,

x− y − 1 = 0.(2.4)

Da segunda equação segue-se que

x = y + 1. (2.5)

Substituindo esta expressão na primeira equação obtemos

(y + 1) + y − z − 1 = 0,

2y − z = 0,

z = 2y. (2.6)

Notemos que as variáveis x e z são resolvidas em função da variável y,

a qual não possui nenhuma restrição, de modo que se y assumir um

valor real t então x e z cam automaticamente determinadas por este

valor t. Isto é, para todo t real, de (2.5) e (2.6) tem-se que

x = t+ 1, y = t, z = 2t

é solução do sistema (2.4) e, portanto, temos innitas soluções para

este.

Page 57: Resolução de problemas

46 2 Equações e Inequações

Situação (c): Consideremos agora o sistema de primeiro grau nas

variáveis x, y e z dado porx+ y + 2z − 1 = 0,

x+ z − 2 = 0,

y + z − 3 = 0.

(2.7)

Neste caso, da segunda e da terceira equação segue-se que

x = 2− z e y = 3− z.

Substituindo estas expressões na primeira equação obtém-se

(2− z) + (3− z) + 2z − 1 = 0⇐⇒ 4 = 0,

o que é uma incompatibilidade. Logo, este sistema não tem solução.

Observação 2.14. Os sistemas de equações de primeiro grau são tam-

bém conhecidos como sistemas de equações lineares. Quando um sis-

tema de equações lineares envolve muitas variáveis não é tão fácil

resolvê-lo se não se organiza com cuidado seu processo de resolução.

Existe uma teoria bem conhecida e amplamente divulgada sobre mé-

todos de resolução para esse tipo de sistemas. Um dos métodos mais

usado e eciente para resolver sistemas lineares é o método de elimi-

nação gaussiana. O leitor interessado pode consultar [7].

2.2.1 Problemas Resolvidos

O problema a seguir foi proposto na primeira fase da Olimpíada Bra-

sileira de Matemática.

Page 58: Resolução de problemas

2.2 Sistemas de Equações do Primeiro Grau 47

Problema 2.15. Passarinhos brincam em volta de uma velha árvore.

Se dois passarinhos pousam em cada galho, um passarinho ca voando.

Se todos os passarinhos pousam, com três em cada galho, um galho ca

vazio. Quantos são os passarinhos?

Solução: Vamos chamar de p o número de passarinhos e g o número

de galhos da árvore. Temos que se dois passarinhos pousam em cada

galho, um passarinho ca voando, ou seja,

2g = p− 1.

Além disso, se todos os passarinhos pousam, com três em um mesmo

galho, um galho ca vazio:

3(g − 1) = p.

Substituindo na equação anterior, temos que 2g = 3g − 3 − 1, onde

segue-se que g = 4 e p = 9.

Problema 2.16. Quanto medem as áreas A1 e A2 na gura abaixo,

sabendo que o quadrado tem lado 1 e as curvas são arcos de círculos

com centros nos vértices V1 e V2 do quadrado, respectivamente.

A1

A2

V2

V

Solução: Aplicando relações de áreas na gura temos queA1 + A2 = π

4,

A1 + 2A2 = 1,

Page 59: Resolução de problemas

48 2 Equações e Inequações

ou seja, chegamos a um sistema de equações do primeiro grau com

duas incógnitas A1 e A2. Da primeira equação temos que

A1 =π

4− A2;

substituindo esta na segunda equação obtemos

π

4− A2 + 2A2 = 1,

de ondeπ

4+ A2 = 1.

Logo, A2 = 1− π4

e A1 = π4−(1− π

4

)= π

2− 1.

Problema 2.17. Carlos e Cláudio são dois irmãos temperamentais

que trabalham carregando e descarregando caminhões de cimento. Para

Carlos e Cláudio tanto faz carregar ou descarregar o caminhão, o tra-

balho realizado por eles é o mesmo. Quando estão de bem, trabalham

juntos e conseguem carregar um caminhão em 15 minutos. Cláudio

é mais forte e trabalha mais rápido conseguindo carregar sozinho um

caminhão em 20 minutos.

(a) Um dia, Cláudio adoeceu e Carlos teve que carregar os caminhões

sozinho. Quanto tempo ele leva para carregar cada um?

(b) Quando os dois brigam, Carlos costuma se vingar descarregando

o caminhão, enquanto Cláudio o carrega com sacos de cimento.

Quanto tempo Cláudio levaria para carregar o caminhão com

Carlos descarregando?

Solução: Vamos chamar de x a quantidade de sacos que Cláudio car-

rega por minuto e y a quantidade de sacos que Carlos carrega por

Page 60: Resolução de problemas

2.3 Equação do Segundo Grau 49

minuto. Como Cláudio carrega mais que Carlos, sabemos que y < x.

Do enunciado, sabemos que os dois juntos carregam um caminhão em

15 minutos. Se um caminhão tem capacidade para c sacos, temos que:

15x+ 15y = c.

Além disso, sabemos que Cláudio sozinho carrega o mesmo caminhão

em 20 minutos. Logo,

20x = c.

Assim, igualando as duas equações, temos que

15x+ 15y = 20x, onde 15y = 20x− 15x = 5x.

Logo, dividindo ambos os lados por 5, temos que 3y = x. Assim, Cláu-

dio carrega três vezes mais sacos que Carlos e a resposta do primeiro

item é 20× 3 minutos, já que 60y = 20× 3y = 20x = c.

Para descobrir quanto tempo os dois levam para carregar o cami-

nhão quando estão brigados, observamos que a cada minuto eles car-

regam x − y sacos, ou seja, 3y − y = 2y sacos. Logo, precisam de 30

minutos, já que 30× 2y = 60y = c.

2.3 Equação do Segundo Grau

Como já mencionamos em nossa introdução, o conhecimento de mé-

todos para solucionar as equações do segundo grau remonta às civi-

lizações da antiguidade, como os babilônios e egípcios. Apesar disso,

a fórmula que conhecemos por fórmula de Bhaskara, em homenagem

ao matemático indiano de mesmo nome e que determina as soluções

de uma equação do segundo grau, só veio a aparecer do modo que

usamos muito mais tarde, com o francês Vièti. Nesta seção iremos

deduzir esta fórmula e aplicá-la a alguns problemas interessantes.

Page 61: Resolução de problemas

50 2 Equações e Inequações

2.3.1 Completando Quadrados

Um modo de resolver uma equação do segundo grau é o método de

completar quadrados. Ele consiste em escrever a equação numa forma

equivalente que nos permita concluir quais são as soluções diretamente.

Vamos ilustrar isso com um exemplo, resolvendo a equação

x2 − 6x− 8 = 0.

Podemos escrever essa equação como:

x2 − 6x = 8.

Somando 9 ao lado esquerdo, obtemos x2− 6x+ 9 que é o mesmo que

(x− 3)2. Assim, somando 9 a ambos os lados da equação, obtemos:

(x− 3)2 = 9 + 8 = 17.

Logo, x− 3 =√

17 ou x− 3 = −√

17. Logo, as soluções são:

x1 = 3 +√

17 e x2 = 3−√

17.

Denição 2.18. A equação do segundo grau com coecientes a, b e c

é uma expressão da forma:

ax2 + bx+ c = 0, (2.8)

onde a 6= 0, b, c ∈ R e x é uma variável real a ser determinada.

Para encontrar as soluções desta equação, vamos proceder do se-

guinte modo: isolando o termo que não contém a variável x do lado

direito da igualdade na equação (2.8)

ax2 + bx = −c

Page 62: Resolução de problemas

2.3 Equação do Segundo Grau 51

e dividindo os dois lados por a, obtemos:

x2 +b

ax =−ca.

Agora vamos acrescentar um número em ambos os lados da equa-

ção acima, de modo que o lado esquerdo da igualdade seja um qua-

drado perfeito. Para isso, observe que é necessário adicionar b2

4a2aos

dois lados da igualdade. Assim, temos que:(x+

b

2a

)2

= x2 + 2b

2ax+

(b

2a

)2

=b2

4a2− c

a=b2 − 4ac

4a2.

Em geral, chamamos a expressão b2−4ac de discriminante da equação

(2.8) e denotamos pela letra maiúscula ∆ (lê-se delta) do alfabeto

grego. Assim, podemos escrever a igualdade anterior como:(x+

b

2a

)2

=b2 − 4ac

4a2=

4a2. (2.9)

Por isso, para que exista algum número real satisfazendo a igual-

dade acima, devemos ter que ∆ ≥ 0, já que o termo da esquerda na

igualdade é maior ou igual a zero. Extraindo a raiz quadrada quando

∆ ≥ 0, temos as soluções:

x+b

2a=

√b2 − 4ac

2ae x+

b

2a= −√b2 − 4ac

2a.

Assim, obtemos as seguintes soluções:

x1 = − b

2a+

√b2 − 4ac

2a=

−b+√∆

2a

e

Page 63: Resolução de problemas

52 2 Equações e Inequações

x2 = − b

2a−

√b2 − 4ac

4a2=

−b−√∆

2a.

Em resumo,

• Se ∆ > 0 existem duas soluções reais.

• Se ∆ = 0 só existe uma solução real (x1 = x2 = −b/2a).

• Se ∆ < 0 não existe solução real.

A seguir apresentamos alguns exemplos.

Exemplo 2.19. Encontre as soluções da equação 2x2 − 4x+ 2 = 0.

Solução: Observe que a = 2, b = −4 e c = 2. Logo,

∆ = b2 − 4ac = (−4)2 − 4 · 2 · 2 = 0.

Assim, a única solução é x = − b

2a=

4

4= 1.

Exemplo 2.20. Encontre as raízes da seguinte equação do segundo

grau:

x2 − x− 1 = 0.

Solução: Basta aplicarmos diretamente a fórmula que acabamos de

deduzir. Como a = 1, b = −1 e c = −1, calculando ∆ temos:

∆ = b2 − 4ac = (−1)2 − 4 · 1 · (−1) = 5.

Logo, as soluções são

x1 =−b+

√∆

2a=

1 +√

5

2e x2 =

−b−√

2a=

1−√

5

2.

Page 64: Resolução de problemas

2.3 Equação do Segundo Grau 53

Exemplo 2.21. Sabendo que x é um número real que satisfaz

x = 1 +1

1 +1

x

,

determine os valores possíveis de x.

Solução: A solução desse problema consiste numa simples manipula-

ção algébrica, que feita com cuidado nos levará a uma equação do

segundo grau. Com efeito,

1 +1

x=x+ 1

x.

Logo,

1 +1

1 +1

x

= 1 +x

1 + x=

1 + 2x

1 + x.

Então devemos ter x =1 + 2x

1 + x, de onde segue-se que

x2 + x = 1 + 2x⇐⇒ x2 − x− 1 = 0.

Resolvendo a equação tem-se

x1 =1 +√

5

2e x2 =

1−√

5

2.

Observação 2.22. O número (1+√5)/2 é chamado de razão áurea.

Este número recebe essa denominação pois, frequentemente, as pro-

porções mais belas e que a natureza nos proporciona estão próximas

da razão áurea. Por exemplo, no arranjo das pétalas de uma rosa, nas

espirais que aparecem no abacaxi, na arquitetura do Parthenon, nos

quadros de da Vinci e nos ancestrais de um zangão podemos encon-

trar a razão áurea.

Page 65: Resolução de problemas

54 2 Equações e Inequações

O problema a seguir está relacionado com a seqüência de Fibonacci

e com a razão áurea. Dizemos que uma seqüência de números ansatisfaz a relação de Fibonacci se, para todo n ≥ 0, temos que

an+2 = an+1 + an. (2.10)

Exemplo 2.23. Encontre todas as sequências an da forma an = xn

para algum x 6= 0 que satisfazem a relação de Fibonacci.

Solução: Sabendo que an satisfaz a relação de Fibonacci e que an é da

forma xn, podemos concluir que para todo n ≥ 0 tem-se

xn+2 − xn+1 − xn = 0.

Colocando xn em evidência na equação acima, temos que

xn(x2 − x− 1) = 0

Logo, temos duas possibilidades: xn = 0 ou x2 − x − 1 = 0. Como

x 6= 0, temos que xn 6= 0 e, portanto, x2 − x − 1 = 0. Observando a

solução do Exemplo 2.20 temos que as únicas sequências são

an =

(1 +√

5

2

)n

ou an =

(1−√

5

2

)n

.

Observação 2.24. Se an e bn satisfazem a relação de Fibonacci (2.10),

então dados números reais α e β, qualquer sequência da forma αan +

βbn satisfaz a relação. Pode-se provar que as sequências dessa forma,

com an = xn1 e bn = xn2 calculados anteriormente, são as únicas

sequências que satisfazem a relação. Veja, por exemplo, [4].

Page 66: Resolução de problemas

2.3 Equação do Segundo Grau 55

2.3.2 Relação entre Coecientes e Raízes

Dada a equação ax2 + bx+ c = 0, com a 6= 0, já calculamos explicita-

mente as suas raízes x1 e x2. Vamos estabelecer agora as relações entre

a, b e c e as raízes x1 e x2. Vamos supor ∆ ≥ 0. Como já sabemos,

temos que

x1 =−b+

√∆

2ae x2 =

−b−√

2a.

Assim, somando x1 com x2 tem-se

x1 + x2 =−b+

√∆

2a+−b−

√∆

2a=−2b

2a= − b

a. (2.11)

Por outro lado, fazendo o produto x1x2 obtemos

x1x2 =

(−b+

√∆

2a

)·(−b−

√∆

2a

)

=

(− b+

√∆)(− b−

√∆)

4a2=b2 −∆

4a2

=4ac

4a2=c

a.

(2.12)

Em particular, quando a = 1, temos o seguinte resultado.

Teorema 2.25. Os números α e β são as raízes da equação

x2 − sx+ p = 0 (2.13)

se, e somente se,

α + β = s e αβ = p. (2.14)

Page 67: Resolução de problemas

56 2 Equações e Inequações

Demonstração. Com efeito, se α e β são as raízes de (2.13) então os

cálculos feitos em (2.11) e (2.12) nos dão (2.14). Reciprocamente, se

vale (2.14) então da igualdade

(x− α)(x− β) = x2 − sx+ p

segue-se que α e β são as raízes de (2.13).

Observação 2.26. Em geral, dada a equação ax2 + bx + c = 0, com

a 6= 0, podemos escrevê-la como a(x2 − sx + p) = 0, com s = −b/a e

p = c/a. Supondo que a equação x2 − sx + p = 0 tem raízes α e β, a

igualdade

ax2 + bx+ c = a(x2 − sx+ p) = a(x− α)(x− β) (2.15)

nos permite concluir que α e β são as raízes da equação de segundo

grau ax2 + bx+ c = 0.

A equação (2.15) mostra que se α é raiz de um polinômio do se-

gundo grau, então a divisão desse polinômio pelo polinômio (x− α) é

uma divisão exata. Voltaremos a tratar desse assunto no Teorema 8.5.

Exemplo 2.27. Paulo cercou uma região retangular de área 28 m2

com 24 metros de corda. Encontre as dimensões dessa região.

Solução: Se chamamos de a e b os lados do retângulo construído por

Paulo, as condições sobre o perímetro e a área desse retângulo nos

levam às seguintes equações:a+ b = 12,

ab = 28.

Como já observamos, a e b são raízes da equação x2 − 12x + 28 = 0.

Calculando o discriminante, obtemos ∆ = 122−4·28 = 32. Utilizando

Page 68: Resolução de problemas

2.3 Equação do Segundo Grau 57

a fórmula, temos que as soluções são

a =12 +

√32

2= 6 + 2

√2

e

b =12−

√32

2= 6− 2

√2.

Exemplo 2.28. Mostre que a equação x2 + bx + 17 = 0 não possui

raiz inteira positiva, se b é um inteiro não negativo.

Solução: Suponhamos que a equação possui alguma raiz inteira n po-

sitiva e seja m a outra raiz (podendo ser m = n). Então, n+m = −b,onde m = −n− b deverá ser necessariamente um número inteiro. Por

outro lado, m e n são números inteiros tais que m · n = 17, o que só

é possível se m = 1 ou n = 1, o que nos daria em qualquer um dos

casos que 1 + b + 17 = 0 (b = −18), sendo isto uma contradição com

o fato de b ser inteiro não negativo.

Exemplo 2.29. Numa reunião havia pelo menos 12 pessoas e todos

os presentes apertaram as mãos entre si. Descubra quantas pessoas

estavam presentes na festa, sabendo que houve menos que 75 apertos

de mão.

Solução: Vamos denotar por a o número de apertos de mão e enumerar

as pessoas com os números do conjunto P = 1, 2, . . . , n. A cada

aperto de mão associaremos um par (i, j), signicando que a pessoa i

apertou a mão da pessoa j. Assim, os apertos de mão envolvendo a

pessoa 1 foram

A1 =

(1, 2), (1, 3), . . . , (1, n).

Page 69: Resolução de problemas

58 2 Equações e Inequações

Do mesmo modo, denimos os apertos de mão envolvendo a pessoa 2

que não envolvem a pessoa 1, como

A2 =

(2, 3), (2, 4), . . . , (2, n).

Note que o aperto (2, 1) é o mesmo que o aperto (1, 2), já que se 1

aperta a mão de 2, então 2 aperta a mão de 1. Analogamente,

Ai =

(i, i+ 1), (i, i+ 2), . . . , (i, n), para 1 ≤ i ≤ n− 1.

Note que Ai ∩ Aj = ∅ para i 6= j. Observe também que todos os

apertos aparecem em um dos conjuntos Ai. Assim, A1 ∪ · · · ∪ An−1

contém todos os apertos de mão. Logo, se |X| denota o número de

elementos do conjunto X e a o número de apertos de mão, temos

|(A1 ∪ A2 ∪ · · · ∪ An−1)| = |A1|+ |A2|+ · · ·+ |An−1|= (n− 1) + (n− 2) + · · ·+ 2 + 1

=(n− 1)n

2= a.

Portanto, n2−n−2a = 0 deve admitir admite uma raiz inteira, maior

ou igual a 12. Deste modo, basta descobrirmos para que valores de

a < 75 a equação acima admite alguma raiz inteira n ≥ 12. Denotemos

as raízes da equação por n1 e n2 e suponhamos que n1 ≥ 12. Das

relações n1n2 = −2a,

n1 + n2 = 1,

concluímos que −n2 = n1 − 1 ≥ 11. Assim, podemos deduzir que

−n2n1 ≥ 11 · 12 = 132, pois −n2 ≥ 11 e n1 ≥ 12.

Observe que o mesmo raciocínio nos leva a concluir que se n1 ≥ 13

então −n2n1 = 2a ≥ 12 · 13 = 156, o que nos daria a ≥ 78, sendo

Page 70: Resolução de problemas

2.3 Equação do Segundo Grau 59

isto impossível pois a < 75. Assim, a raiz positiva para tal equação

não pode ser maior ou igual que 13, restando somente n1 = 12 como

solução. De fato, essa solução é possível, se considerarmos a = 66.

Logo, haviam 12 pessoas na festa.

2.3.3 Equações Biquadradas

A dedução da solução da equação do segundo grau nos permite resolver

equações de grau mais alto, desde que elas se apresentem numa forma

peculiar, que nos permita reduzi-las a uma equação do segundo grau.

Por exemplo,

Exemplo 2.30. Resolva a equação

x4 − 2x2 + 1 = 0. (2.16)

Apesar da equação acima ser de grau quatro, podemos solucioná-la

utilizando o que aprendemos até agora. O truque será denotar por y

o valor x2.

Solução: Denote por y = x2. Neste caso, temos que 0 = y2− 2y+ 1 =

(y − 1)2. Logo, y = 1. Assim, x2 = y = 1, de onde segue-se que x = 1

ou x = −1.

De modo geral consideremos a equação

ax2k + bxk + c = 0, k ∈ N, (2.17)

e façamos a mudança y = xk. Então, a equação se transforma na

seguinte;

ay2 + by + c = 0, (2.18)

Page 71: Resolução de problemas

60 2 Equações e Inequações

a qual já sabemos resolver. Logo, se (2.18) não possui solução então

(2.17) também não terá solução e no caso em que y = α seja uma

raiz de (2.18) então as soluções para (2.17), correspondentes à raiz α,

podem ser encontradas resolvendo a equação simples

xk = α,

a qual tem as seguintes possibilidades:

• uma única solução: x = k√α se k é ímpar;

• nenhuma solução: se α < 0 e k é par;

• duas soluções: x = ± k√α se α > 0 e k é par.

2.3.4 O Método de Vièti

A maneira que François Vièti (1540-1603) descobriu para resolver a

equação do segundo grau baseia-se em relacionar a equação

ax2 + bx+ c = 0 (2.19)

como uma equação do tipo

Ay2 +B = 0, (2.20)

onde A eB são números que dependem de a, b, c, de modo que qualquer

solução da equação (2.20) determinará uma solução da equação (2.19).

Note que a última equação possui soluções

y1 =

√−BA

e y2 = −√−BA, se − B

A≥ 0.

Page 72: Resolução de problemas

2.3 Equação do Segundo Grau 61

Para fazer isso, usamos o seguinte truque: escrevendo x = u+ v como

a soma de duas novas variáveis u e v, a equação (2.19) se escreve como

a(u+ v)2 + b(u+ v) + c = 0,

a qual, desenvolvendo o quadrado, equivale a

au2 + 2auv + av2 + bu+ bv + c = 0.

Agrupando convenientemente, podemos escrever a expressão acima

como uma equação na variável v, isto é,

av2 + (2au+ b)v + au2 + bu+ c = 0.

Assim, podemos obter uma equação do tipo da equação (2.20), esco-

lhendo o valor de u de modo que o termo (2au + b)v se anule. Esco-

lhendo u = −b/2a temos que

av2 + a

(−b2a

)2

− b b2a

+ c = 0⇐⇒ av2 +b2

4a− b2

2a+ c = 0,

o que é equivalente a

av2 +−b2 + 4ac

4a= 0.

Observando que a equação assumiu a forma da equação (2.20), temos

que suas soluções são

v1 =

√b2 − 4ac

4a2e v2 = −

√b2 − 4ac

4a2, se ∆ = b2 − 4ac ≥ 0.

Lembrando que u = −b/2a e que x = u+ v temos que as soluções da

equação (2.19) são

x1 = − b

2a+ v1 e x2 = − b

2a+ v2,

como já obtivemos anteriormente.

Page 73: Resolução de problemas

62 2 Equações e Inequações

2.4 Inequações

Inequações aparecem de maneira natural em várias situações dentro

do contexto matemático, assim como no próprio dia a dia.

Exemplo 2.31. Numa loja de esportes as bolas de tênis Welson en-

traram em promoção, passando a custar cada uma três reais. Pedro

que é um assíduo jogador de tênis quer aproveitar ao máximo a oferta

da loja, mas ele só dispõe de cem reais. Qual é a maior quantidade

possível de bolas que Pedro pode comprar?

Solução. Se denotamos por x o número de bolas que Pedro compra,

então devemos achar o maior valor possível de x tal que

3x ≤ 100. (2.21)

Notemos que o problema se reduz a encontrar o maior múltiplo

positivo de 3 que seja menor ou igual a 100. Observe que 99 = 3 · 33

é o maior múltiplo de 3 menor ou igual a 100, pois 3 · 34 = 102 > 100

e Pedro não teria orçamento para efetuar a compra. Logo, a solução

é x = 33, ou seja, Pedro poderá comprar 33 bolas.

Observemos que no exemplo anterior o que zemos foi achar o

maior valor inteiro de x tal que 3x−100 < 0; porém note que qualquer

número x real menor que 100/3 satisfaz que 3x− 100 < 0. Isto é um

caso particular de resolução de uma inequação, chamada inequação do

primeiro grau.

Page 74: Resolução de problemas

2.5 Inequação do Primeiro Grau 63

2.5 Inequação do Primeiro Grau

Uma inequação do primeiro grau é uma relação de uma das formas

abaixo ax+ b < 0, ax+ b > 0,

ax+ b ≤ 0, ax+ b ≥ 0,(2.22)

onde a, b ∈ R e a 6= 0.

O conjunto solução de uma inequação do primeiro grau é o con-

junto S de números reais que satisfazem a inequação, isto é, o conjunto

de números que quando substituídos na inequação tornam a desigual-

dade verdadeira. Para achar tal conjunto será de vital importância

tomar em conta as seguintes propriedades das desigualdades entre dois

números

• Invariância do sinal por adição de números reais: sejam

a e b números reais tais que a ≤ b, então a + c ≤ b + c para

qualquer número real c. O mesmo vale com as desigualdades do

tipo: <, ≥ ou >.

• Invariância do sinal por multiplicação de números reais

positivos: sejam a e b números reais tais que a ≤ b, então

ac ≤ bc para qualquer número real positivo c. Resultados aná-

logos valem para as desigualdades do tipo: <, ≥ ou >.

• Mudança do sinal por multiplicação de números reais

negativos: sejam a e b números reais tais que a ≤ b, então ac ≥bc para qualquer número real negativo c. Resultados análogos

valem para as desigualdades do tipo: <, ≥ ou >.

Page 75: Resolução de problemas

64 2 Equações e Inequações

Vejamos como solucionar as inequações estritas

ax+ b < 0 e ax+ b > 0.

Para isto, dividimos a análise em dois casos.

• Caso 1: a > 0

Inequação ax + b < 0: neste caso, dividindo por a obtemos

que x + b/a < 0 e somando −b/a, em ambos os membros desta

última inequação, temos que x < −b/a. Portanto,

S = x ∈ R; x < −b/a,

o qual representamos no seguinte desenho:

•−b/a

SInequação ax + b > 0: procedendo do mesmo modo que o

caso anterior, obtemos que o conjunto solução vem dado por

S = x ∈ R; x > −b/a,

representado no desenho abaixo:

•−b/a

S• Caso 2: a < 0

Inequação ax + b < 0: neste caso, quando dividimos por a o

sinal da inequação se inverte, obtendo assim que x + b/a > 0,

logo temos que x > −b/a e, consequentemente,

S = x ∈ R; x > −b/a,

cuja representação na reta é a seguinte:

Page 76: Resolução de problemas

2.5 Inequação do Primeiro Grau 65

•−b/a

SInequação ax + b > 0: similarmente, o conjunto solução vem

dado por

S = x ∈ R; x < −b/a,cuja representação é a seguinte:

•−b/a

SObservação 2.32. Notemos que se queremos resolver as inequações

ax + b ≤ 0 e ax + b ≥ 0, então o conjunto solução S em cada um

dos casos acima continua o mesmo acrescentado apenas do ponto x =

−b/a.

Vejamos agora um exemplo simples.

Exemplo 2.33. Para resolver a inequação 8x−4 ≥ 0, primeiramente

dividimos por 8 a inequação (prevalecendo o sinal da desigualdade)

e imediatamente adicionamos 1/2 em ambos os membros da mesma,

para obter x− 4/8 + 1/2 ≥ 1/2, ou seja,

S = x ∈ R; x ≥ 1/2.

A seguir damos alguns exemplos que podem ser resolvidos usando

inequações lineares.

Exemplo 2.34. Sem fazer os cálculos, diga qual dos números

a = 3456784 · 3456786 + 3456785 e b = 34567852 − 3456788

é maior?

Page 77: Resolução de problemas

66 2 Equações e Inequações

Solução. Se chamamos de x ao número 3456784 então das denições

de a e b temos que a = x · (x + 2) + (x + 1) e b = (x + 1)2 − (x + 4).

Logo, a = x2 + 3x+ 1 e b = x2 + x− 3. Se supomos que a ≤ b, então

x2 + 3x+ 1 ≤ x2 + x− 3,

e somando−x2−x+3 a ambos os membros desta desigualdade obtemos

2x+ 4 ≤ 0.

A solução desta inequação do primeiro grau é o conjunto dos x ∈ Rtais que x ≤ −2, mas isto é falso, desde que x = 3456784. Logo,

nossa suposição inicial de a ser menor ou igual a b é falsa, sendo então

a > b.

O próximo exemplo já foi tratado antes (ver Problema 2.7), porém

apresentamos a seguir uma solução diferente usando inequações do

primeiro grau.

Exemplo 2.35. Um quadrado mágico 3× 3 é um quadrado de lado 3

dividido em 9 quadradinhos de lado 1 de forma tal que os números de 1

até 9 são colocados um a um em cada quadradinho com a propriedade

de que a soma dos elementos de qualquer linha, coluna ou diagonal é

sempre a mesma. Provar que no quadradinho do centro de tal quadrado

mágico deverá aparecer, obrigatoriamente, o número 5.

Solução. Primeiramente observamos que a soma 1 + 2 + 3 + · · ·+ 9 =

45, logo como há três linhas e em cada uma destas guram números

diferentes temos que a soma dos elementos de cada linha é 15. Logo,

a soma dos elementos de cada coluna ou diagonal também é 15.

Chamemos de x o número que aparece no centro do quadrado

mágico, como mostra o desenho a seguir.

Page 78: Resolução de problemas

2.5 Inequação do Primeiro Grau 67

x

Agora fazemos as seguintes observações:

• O número x não pode ser 9, pois nesse caso em alguma linha,

coluna ou diagonal que contém o quadrado central aparecerá

o número 8, que somado com 9 dá 17 > 15 e isto não pode

acontecer.

• O número x não pode ser 1, pois nesse caso formaria uma linha,

coluna ou diagonal com o número 2 e um outro número que

chamamos de y, então 1 + 2 + y = 15 ⇔ y = 12, o qual é

impossível.

Feitas as observações anteriores, temos então que o número x forma

uma linha, coluna ou diagonal com o número 9 e algum outro número

que chamamos de z, logo

z = 15− (x+ 9) ≥ 1⇔ 6− x ≥ 1,

de onde segue que x ≤ 5.

Por outro lado, o número x aparece numa linha, coluna ou diagonal

com o número 1 e algum outro número que chamamos de s, logo

s = 15− (x+ 1) = 14− x ≤ 9, de onde temos que x ≥ 5. Finalmente,

como 5 ≤ x ≤ 5 segue-se que x = 5.

Exemplo 2.36. Num triângulo com lados de comprimento a, b e c

traçamos perpendiculares desde um ponto arbitrário P , sobre o lado

de comprimento c, até cada um dos lados restantes (ver a Figura 2.1).

Se estas perpendiculares medem x e y e a > b, então

Page 79: Resolução de problemas

68 2 Equações e Inequações

(a) Qual a posição onde deve ser colocado P de maneira que ` = x+y

seja mínimo?

(b) Qual a posição onde deve ser colocado P de maneira que ` = x+y

seja máximo?

a

b

c

Px y

B A

C

Figura 2.1: No desenho, os segmentos que partem do ponto P são perpen-

diculares aos lados AC e BC

Solução. Denotemos por S a área do triângulo e notemos que divi-

dindo este em dois triângulos menores: um com base a e altura x e

outro com base b e altura y, temos que

ax

2+by

2= S,

de onde se segue que

ax = 2S − by

x =2S − by

a.

Somando y em ambos os lados da última igualdade, obtemos

x+ y =2S − by

a+ y

=2S − by + ay

a

=2S

a+a− ba

y,

Page 80: Resolução de problemas

2.6 Inequação do Segundo Grau 69

logo

` = α + βy,

onde

α =2S

ae β =

a− ba

.

Agora notemos que 0 ≤ y ≤ hb, onde hb denota a altura relativa ao

lado de comprimento b no triângulo dado. Como β é positivo, por ser

a > b, temos então que 0 ≤ βy ≤ βhb e, portanto, α ≤ α + βy ≤α + βhb, de onde

0 ≤ ` ≤ α + βhb.

Resumindo, o valor mínimo de ` é atingido quando y = 0, portanto

P deve ser colocado no vértice A, e o valor máximo é obtido quando

y = hb, portanto P deve ser colocado no vértice B.

2.6 Inequação do Segundo Grau

Agora passamos a discutir a solução das inequações do segundo grau,

que possuem um maior grau de diculdade quando comparadas com

as inequações do primeiro grau. Será de vital importância o uso das

propriedades da função quadrática ax2 + bx+ c, estudadas no capítulo

anterior.

Uma inequação do segundo grau é uma relação de uma das formas

abaixo ax2 + bx+ c < 0, ax2 + bx+ c > 0,

ax2 + bx+ c ≤ 0, ax2 + bx+ c ≥ 0,(2.23)

onde a, b, c ∈ R e a 6= 0. Por simplicidade, chamaremos o número a de

coeciente líder da função quadrática ax2 + bx+ c.

Page 81: Resolução de problemas

70 2 Equações e Inequações

Por exemplo, para resolver a inequação x2 − 3x+ 2 > 0 fatoramos

o trinômio usando que as raízes da equação x2− 3x+ 2 = 0 são 1 e 2,

isto é,

x2 − 3x+ 2 = (x− 1)(x− 2).

O trinômio toma valores positivos quando o produto (x−1)(x−2) for

positivo, ou seja, quando os fatores (x− 1) e (x− 2) tenham o mesmo

sinal:

• Ambos positivos:

x− 1 > 0⇔ x > 1

e

x− 2 > 0⇔ x > 2,

logo x > 2.

• Ambos negativos:

x− 1 < 0⇔ x < 1

e

x− 2 < 0⇔ x < 2,

logo x < 1.

Portanto, x2 − 3x+ 2 > 0 se, e somente se, x < 1 ou x > 2.

A seguir explicamos como podemos resolver a inequação do se-

gundo grau de forma geral.

Suponhamos primeiramente que queremos resolver a inequação

ax2 + bx+ c > 0. (2.24)

Page 82: Resolução de problemas

2.6 Inequação do Segundo Grau 71

Notemos que valem as seguintes igualdades:

ax2 + bx+ c = a

(x2 +

b

ax+

c

a

)= a

(x2 +

b

ax+

b2

4a2− b2

4a2+c

a

)= a

(x2 +

b

ax+

b2

4a2

)− a

(b2

4a2− c

a

)= a

(x+

b

2a

)2

− ∆

4a,

(2.25)

onde ∆ = b2− 4ac. Considerando esta igualdade, dividimos em vários

casos:

Caso 1: ∆ = b2 − 4ac > 0. Nesta situação procedemos tomando

em conta o sinal de a.

• (a > 0). Usando (2.25) notamos que basta resolver a inequação

a

(x+

b

2a

)2

− ∆

4a> 0.

Como a > 0, multiplicando por 1/a em ambos os membros da

desigualdade anterior o sinal desta não muda, obtendo-se então

(x+

b

2a

)2

− ∆

4a2> 0.

Agora usamos que ∆ > 0 para obtermos que

Page 83: Resolução de problemas

72 2 Equações e Inequações

(x+

b

2a

)2

− ∆

4a2=

(x+

b

2a

)2

−(√

2a

)2

=

(x+

b+√

2a

)(x+

b−√

2a

)

=

(x− −b−

√∆

2a

)(x− −b+

√∆

2a

)= (x− α)(x− β) > 0,

onde α = −b−√

∆2a

e β = −b+√

∆2a

são as raízes de ax2 + bx+ c = 0.

Agora notamos que (x − α)(x − β) > 0 se os fatores (x − α) e

(x − β) são ambos positivos ou ambos negativos. No primeiro

caso (ambos positivos) temos que x > α e x > β, mas como

α < β, então x > β. No segundo caso (ambos negativos), temos

que x < α e x < β, logo x < α, novamente por ser α < β.

Resumindo, a solução da inequação vem dada pelo conjunto

S = x ∈ R; x < α ou x > β,

com a seguinte representação na reta:

• •α β

S S• (a < 0). Esta situação é bem similar à anterior, a única dife-

rença é que ao multiplicar por 1/a o sinal se inverte tendo então

que resolver a inequação(x+

b

2a

)2

− ∆

4a2< 0,

Page 84: Resolução de problemas

2.6 Inequação do Segundo Grau 73

a qual é equivalente a provar (seguindo os mesmos passos do

caso anterior) que

(x− α)(x− β) < 0,

com α = −b−√

∆2a

e β = −b+√

∆2a

raízes de ax2 + bx + c = 0.

Notemos que a desigualdade acima é válida sempre que os sinais

dos fatores (x − α) e (x − β) forem diferentes. Por exemplo,

se x − α > 0 e x − β < 0 temos então que x deve satisfazer a

desigualdade α < x < β, mas isso é impossível considerando que

neste caso α > β, por ser a < 0. No caso restante, se x− α < 0

e x − β > 0 temos então que β < x < α, o que é possível.

Portanto, o conjunto solução, neste caso, é dado por

S = x ∈ R; β < x < α,

cuja representação na reta é:

• •β α

SCaso 2: ∆ = b2 − 4ac = 0. Usando novamente (2.25), devemos

resolver a inequação

a

(x+

b

2a

)2

> 0,

a qual é válida para qualquer x 6= − b2a, se a > 0 e sempre falsa, se

a < 0.

Caso 3: ∆ = b2 − 4ac < 0. Neste caso, quando a é positivo todos

os valores de x reais são solução para (2.24), pois a desigualdade

ax2 + bx+ c = a

(x+

b

2a

)2

− ∆

4a> 0,

Page 85: Resolução de problemas

74 2 Equações e Inequações

é sempre satisfeita, dado que −∆4a> 0. Por outro lado, se a é negativo

não temos nenhuma solução possível para a inequação (2.24) já que

ax2 + bx+ c = a

(x+

b

2a

)2

− ∆

4a

é sempre negativo, dado que −∆4a< 0.

Observação 2.37. Para a desigualdade do tipo

ax2 + bx+ c < 0

são obtidos resultados similares, seguindo o mesmo processo descrito

anteriormente. Além disso, para as inequações

ax2 + bx+ c ≥ 0 e ax2 + bx+ c ≤ 0

os resultados são os mesmos, acrescentados apenas dos pontos α, β

ou −b/2a, dependendo do caso.

Exemplo 2.38. Provar que a soma de um número positivo com seu

inverso é sempre maior ou igual que 2.

Solução. Seja x > 0, então devemos provar que

x+1

x≥ 2.

Partimos da seguinte desigualdade, que sabemos vale para qualquer

x ∈ R:(x− 1)2 ≥ 0

logo

x2 − 2x+ 1 ≥ 0⇐⇒ x2 + 1 ≥ 2x.

Page 86: Resolução de problemas

2.6 Inequação do Segundo Grau 75

Se x é positivo, podemos dividir ambos os membros da última desi-

gualdade sem alterar o sinal da mesma, ou seja,

x+1

x≥ 2,

conforme queríamos provar.

2.6.1 Máximos e Mínimos das Funções Quadráti-

cas

A função quadrática f(x) = ax2 + bx+ c, como já foi observado ante-

riormente, satisfaz a identidade

ax2 + bx+ c = a

(x+

b

2a

)2

− ∆

4a, (2.26)

onde ∆ = b2 − 4ac. O valor mínimo (máximo) da função quadrática

f(x) é o menor (maior) valor possível que pode assumir f(x) quando

fazemos x percorrer o conjunto dos reais.

Da igualdade (2.26) segue-se que, quando a > 0 o valor mínimo

do trinômio é obtido quando x = − b2a

e este vale f(− b2a

) = −∆4a.

Similarmente, quando a < 0 o valor máximo do trinômio é obtido

quando x = − b2a, valendo também f(− b

2a) = −∆

4a

Exemplo 2.39. Sejam a, b reais positivos tais que a+ b = 1. Provar

que ab ≤ 1/4.

Solução. Notemos que ab = a(1 − a) = −a2 + a. Denindo f(a) =

−a2 + a, basta provar que f(a) ≤ 1/4 para qualquer 0 < a < 1.

Completando o quadrado a função f(a), obtemos

f(a) = −(a2 − a) = −(a2 − a+ 1/4− 1/4) = −(a− 1/2)2 + 1/4,

logo este assume seu valor máximo igual a 1/4, quando a = 1/2.

Page 87: Resolução de problemas

76 2 Equações e Inequações

Alguns problemas de máximos ou mínimos não parecem que pos-

sam ser resolvidos achando o máximo ou mínimo de funções quadrá-

ticas. Porém, estes problemas podem ser reformulados de forma tal

que isto seja possível. Vejamos um exemplo.

Exemplo 2.40. Na gura abaixo ABCD é um retângulo inscrito den-

tro do círculo de raio r. Encontre as dimensões que nos dão a maior

área possível do retângulo ABCD.

xyr

A B

CD

Solução. A área do retângulo vem dada pela fórmula

A = 2x · 2y = 4xy.

Usando o teorema de Pitágoras, temos que

y =√r2 − x2, (2.27)

logo, substituindo esta última igualdade na fórmula de área anterior,

obtemos

A = 4x√r2 − x2.

Não é muito difícil nos convencermos de que as dimensões, que nos

dão a maior área possível para o retângulo ABCD, são as mesmas

que nos dão o máximo para o quadrado desta área, ou seja, basta

encontrar as dimensões que maximizam A2. A vantagem que tem esta

Page 88: Resolução de problemas

2.7 Miscelânea 77

reformulação do problema é que A2 tem uma expressão mais simples,

dada por

A2 = 16x2(r2 − x2) = 16r2x2 − 16x4.

Agora fazemos a mudança z = x2, para obter

A2 = −16z2 + 16r2z = −16(z − r2

2

)2

+ 4r4,

de onde segue que o menor valor de A2 é obtido quando z = r2

2e

portanto quando x = r√2. Usando agora a igualdade (2.27) temos que

y =

√r2 − r2

2=

r√2.

Então, o retângulo de maior área possível é o quadrado de lado2r√

2= r√

2.

2.7 Miscelânea

Nesta seção combinamos a teoria desenvolvida nos tópicos anteriores

para resolver outros tipos de equações com um nível de complexidade

maior.

2.7.1 Equações Modulares

Uma equação modular é aquela na qual a variável incógnita aparece

sob o sinal de módulo. Por exemplo, são equações modulares

(a) |2x− 5| = 3;

(b) |2x− 3| = 1− 3x;

Page 89: Resolução de problemas

78 2 Equações e Inequações

(c) |3− x| − |x+ 1| = 4.

Para resolver equações modulares se usam basicamente três méto-

dos:

(1) eliminação do módulo pela denição;

(2) elevação ao quadrado de ambos os membros da equação;

(3) partição em intervalos.

Ilustramos a seguir estes métodos com os exemplos dados em (a),

(b) e (c).

Exemplo 2.41. Resolver a equação |2x− 5| = 3.

Solução: O método (1) pode ser utilizado para resolver esta equação.

Para isto, usamos a denição de módulo:

|a| =

a se a ≥ 0,

−a se a < 0.

de onde segue-se a propriedade: seja b um número não negativo, então

|a| = b ⇐⇒ a = b ou a = −b.

Logo, x é solução da equação se, e somente se, x satisfaz uma das

equações de primeiro grau a seguir:

2x− 5 = 3 ou 2x− 5 = −3.

Da primeira equação obtemos a solução x1 = 4 e da segunda obtemos

a solução x2 = 1.

Page 90: Resolução de problemas

2.7 Miscelânea 79

Exemplo 2.42. Resolver a equação |2x− 3| = 1− 3x

Solução: Resolveremos esta equação pelos métodos (1) e (2).

Método (1): Aplicando a denição de módulo temos que resolver a

equação é equivalente a resolver os sistemas mistos

(a)

2x− 3 ≥ 0,

2x− 3 = 1− 3x,ou (b)

2x− 3 < 0,

−(2x− 3) = 1− 3x.

O sistema (a) não tem solução visto que a solução da equação do

primeiro grau 2x− 3 = 1− 3x⇔ 5x = 4 é x = 4/5 a qual não satisfaz

a desigualdade 2x − 3 ≥ 0. Por outro lado, no sistema (b) a solução

da equação −(2x−3) = 1−3x tem por solução x = −2 a qual satisfaz

a inequação 2x− 3 < 0. Logo, a única solução da equação é x = −2.

Método (2): Observemos que a equação é equivalente ao sistema

misto 1− 3x ≥ 0,

(2x− 3)2 = (1− 3x)2.

Resolvendo agora a equação de segundo grau (2x− 3)2 = (1− 3x)2 a

qual é equivalente a 5x2 + 6x− 8 = 0, temos que as possíveis soluções

são x1 = 4/5 e x2 = −2, mas x1 é descartada pois não satisfaz que

1−3x1 ≥ 0. Assim, a solução do sistema misto e, portanto, da equação

modular é apenas x2 = −2.

Exemplo 2.43. |3− x| − |x+ 1| = 4.

Solução. Neste caso usaremos o método de partição em intervalos que

consiste no seguinte: marcamos na reta real os valores onde |3 − x|e |x + 1| se anulam, neste caso, x1 = 3 e x2 = −1. Com isto a reta

Page 91: Resolução de problemas

80 2 Equações e Inequações

numérica é dividida em 3 intervalos x < −1, −1 ≤ x ≤ 3 e x > 3.

Agora analisamos a equação em cada intervalo:

Intervalo x < −1: Neste caso a equação modular toma a forma

3− x− (−x− 1) = 4⇐⇒ 4 = 4,

Portanto, todo o intervalo x < −1 é solução.

Intervalo −1 ≤ x ≤ 3: Neste caso a equação modular toma a forma

3− x− (x+ 1) = 4⇐⇒ 2− 2x = 4,

de onde segue-se que x = −1. Portanto, neste intervalo a solução é

x = −1.

Intervalo x > 3: Neste caso a equação modular toma a forma

−3 + x− (x+ 1) = 4⇐⇒ −4 = 4,

o que é uma contradição. Portanto, neste intervalo não temos solução.

Em resumo, a solução da equação modular é o intervalo x ≤ −1.

2.7.2 Um Sistema de Equações Não lineares

O seguinte exemplo nos mostra como podemos combinar a técnica de

resolução de sistemas lineares e de equações de segundo grau para

resolver sistemas mais complicados.

Exemplo 2.44. Resolva o sistema de equações:√x2 + 3x− (x2 − 2)3 = 3,√x2 + 3x+ (x2 − 2)3 = 1.

Page 92: Resolução de problemas

2.8 Exercícios 81

Solução. Propomos a seguinte mudança de variáveis:

u =√x2 + 3x e v = (x2 − 2)3.

Assim, o sistema se converte no sistema de equações do primeiro grauu− v = 3,

u+ v = 1,

o qual tem como solução u = 2 e v = −1. Verique! Assim,

√x2 + 3x = 2⇐⇒ x2 + 3x = 4,

sendo x = 1 e x = −4 as soluções desta equação do segundo grau.

Por outro lado

(x2 − 2)3 = −1,

de onde x2−2 = −1, sendo x = 1 e x = −1 as soluções desta equação.

Logo, a solução do sistema é x = 1, que é a única que satisfaz u = 2

e v = −1 simultaneamente.

2.8 Exercícios

1. Observe as multiplicações a seguir:

(a) 12.345.679× 18 = 222.222.222

(b) 12.345.679× 27 = 333.333.333

(c) 12.345.679× 54 = 666.666.666

Para obter 999.999.999 devemos multiplicar 12.345.679 por quan-

to?

Page 93: Resolução de problemas

82 2 Equações e Inequações

2. Outro dia ganhei 250 reais, incluindo o pagamento de horas

extras. O salário (sem horas extras) excede em 200 reais o que

recebi pelas horas extras. Qual é o meu salário sem horas extras?

3. Uma torneira A enche sozinha um tanque em 10 h, uma torneira

B enche o mesmo tanque sozinha em 15 h. Em quantas horas

as duas torneiras juntas encherão o tanque?

4. O dobro de um número, mais a sua terça parte, mais a sua quarta

parte somam 31. Determine o número.

5. Uma certa importância deve ser dividida entre 10 pessoas em

partes iguais. Se a partilha fosse feita somente entre 8 dessas

pessoas, cada uma destas receberia R$5.000,00 a mais. Calcule

a importância.

6. Roberto disse a Valéria: Pense um número, dobre esse número,

some 12 ao resultado, divida o novo resultado por 2. Quanto

deu? Valéria disse 15 ao Roberto, que imediatamente reve-

lou o número original que Valéria havia pensado. Calcule esse

número.

7. Por 2/3 de um lote de peças iguais, um comerciante pagou

R$8.000,00 a mais do que pagaria pelos 2/5 do mesmo lote. Qual

o preço do lote todo?

8. Determine um número real a para que as expressões 3a+68

e 2a+106

sejam iguais.

9. Se você multiplicar um número real x por ele mesmo e do resul-

tado subtrair 14, você vai obter o quíntuplo do número x. Qual

é esse número?

Page 94: Resolução de problemas

2.8 Exercícios 83

10. Eu tenho o dobro da idade que tu tinhas quando eu tinha a

tua idade. Quando tu tiveres a minha idade, a soma das nossas

idades será de 45 anos. Quais são as nossas idades?

11. Um homem gastou tudo o que tinha no bolso em três lojas. Em

cada uma gastou 1 real a mais do que a metade do que tinha ao

entrar. Quanto o homem tinha ao entrar na primeira loja?

12. Com os algarismos x, y e z formam-se os números de dois alga-

rismos xy e yx, cuja soma é o número de três algarismos zxz.

Quanto valem x, y e z?

13. Quantos são os números inteiros de 2 algarismos que são iguais

ao dobro do produto de seus algarismos?

14. Obter dois números consecutivos inteiros cuja soma seja igual a

57.

15. Qual é o número que, adicionado ao triplo do seu quadrado, vale

14?

16. O produto de um número positivo pela sua terça parte é igual a

12. Qual é esse número?

17. Determine dois números consecutivos ímpares cujo produto seja

195.

18. A diferença entre as idades de dois irmãos é 3 anos e o produto

de suas idades é 270. Qual é a idade de cada um?

19. Calcule as dimensões de um retângulo de 16 cm de perímetro e

15 cm2 de área.

Page 95: Resolução de problemas

84 2 Equações e Inequações

20. A diferença de um número e o seu inverso é 83. Qual é esse

número?

21. A soma de dois números é 12 e a soma de seus quadrados é 74.

Determine os dois números.

22. Um pai tinha 30 anos quando seu lho nasceu. Se multiplicarmos

as idades que possuem hoje, obtém-se um produto que é igual

a três vezes o quadrado da idade do lho. Quais são as suas

idades?

23. Os elefantes de um zoológico estão de dieta juntos. Num período

de 10 dias devem comer uma quantidade de cenouras igual ao

quadrado da quantidade que um coelho come em 30 dias. Em

um dia os elefantes e o coelho comem juntos 1.444 kg de cenoura.

Quantos kilos de cenoura os elefantes comem em 1 dia?

24. Sejam α1 e α2 as raízes do polinômio ax2 + bx + c, com a 6= 0.

Calcule as seguintes expressões, em função de a, b e c:

(a)α1 + α2

2;

(b)√α1 +

√α2;

(c) 4√α1 + 4

√α2.

25. O número −3 é a raiz da equação x2 − 7x − 2c = 0. Nessas

condições, determine o valor do coeciente c.

26. Encontre o polinômio p(x) = 2x4 +bx3 +cx2 +dx+e que satisfaz

a equação p(x) = p(1− x).

Page 96: Resolução de problemas

2.8 Exercícios 85

27. (OBM) Dois meninos jogam o seguinte jogo. O primeiro esco-

lhe dois números inteiros diferentes de zero e o segundo monta

uma equação do segundo grau usando como coecientes os dois

números escolhidos pelo primeiro jogador e 1.998, na ordem

que quiser (ou seja, se o primeiro jogador escolhe a e b o se-

gundo jogador pode montar a equação 1.998x2 + ax + b = 0 ou

ax2 +1.998x+b = 0 etc.) O primeiro jogador é considerado ven-

cedor se a equação tiver duas raízes racionais diferentes. Mostre

que o primeiro jogador pode ganhar sempre.

28. (OBM) Mostre que a equação x2 + y2 + z2 = 3xyz tem innitas

soluções onde x, y, z são números inteiros.

29. (Gazeta Matemática, Romênia) Considere a equação

a2x2 − (b2 − 2ac)x+ c2 = 0,

onde a, b e c são números inteiros positivos. Se n ∈ N é tal que

p(n) = 0, mostre que n é um quadrado perfeito.

30. (Gazeta Matemática, Romênia) Sejam a, b ∈ Z. Sabendo que a

equação

(ax− b)2 + (bx− a)2 = x,

tem uma raiz inteira, encontre os valores de suas raízes.

31. (Gazeta Matemática, Romênia) Resolva a equação:

[2x2

x2 + 1

]= x.

Obs.: [x] é o menor inteiro maior ou igual a x.

Page 97: Resolução de problemas

86 2 Equações e Inequações

32. Demonstrar que:

(a) n4 + 4 não é primo se n > 1;

(b) generalize, mostrando que n4 + 4n não é primo, para todo

n > 1.

33. Para fazer 12 bolinhos, preciso exatamente de 100 g de açúcar,

50 g de manteiga, meio litro de leite e 400 g de farinha. Qual a

maior quantidade desses bolinhos que serei capaz de fazer com

500 g de açúcar, 300 g de manteiga, 4 litros de leite e 5 kg de

farinha ?

34. Dadas as frações

966666555557

966666555558e

966666555558

966666555559,

qual é maior?

35. Achar o maior valor inteiro positivo de n tal que

n200 < 5300.

36. Achar o menor valor inteiro positivo de n tal que

10111 · 10

211 · 10

311 · · · 10

n11 > 100000.

37. Nove cópias de certas notas custam menos de R$ 10,00 e dez

cópias das mesmas notas (com o mesmo preço) custam mais de

R$ 11,00. Quanto custa uma cópia das notas?

38. Se enumeram de 1 até n as páginas de um livro. Ao somar estes

números, por engano um deles é somado duas vezes, obtendo-se

o resultado incorreto: 1.986. Qual é o número da página que foi

somado duas vezes?

Page 98: Resolução de problemas

2.8 Exercícios 87

39. Determine os valores de a para os quais a função quadrática

ax2 − ax+ 12 é sempre positiva.

40. Ache os valores de x para os quais cada uma das seguintes ex-

pressões é positiva:

(a)x

x2 + 9(b)

x− 3

x+ 1(c)

x2 − 1

x2 − 3x

41. Resolver a equação:

[x]x+ x = 2x+ 10,

onde [x] denota a parte inteira de x. Por exemplo, [2, 46] = 2 e

[5, 83] = 5. O número x é chamado parte fracionária de x e é

denido por x = x− [x].

42. Mostre que entre os retângulos com um mesmo perímetro, o de

maior área é um quadrado.

43. Entre todos os triângulos isósceles com perímetro p xado, ache

as dimensões dos lados daquele que possui a maior área.

44. (OBM Júnior 1993)

É dada uma equação do segundo grau x2 + ax + b = 0, com

raízes inteiras a1 e b1. Consideramos a equação do segundo grau

x2 + a1x + b1 = 0. Se a equação x2 + a1x + b1 = 0 tem raízes

inteiras a2 e b2, consideramos a equação x2 + a2x+ b2 = 0. Se a

equação x2+a2x+b2 = 0 tem raízes inteiras a3 e b3, consideramos

a equação x2 +a3x+b3 = 0. E assim por diante. Se encontramos

uma equação com ∆ < 0 ou com raízes que não sejam inteiros,

encerramos o processo.

Page 99: Resolução de problemas

88 2 Equações e Inequações

Por exemplo, se começamos com a equação x2 = 0 podemos

continuar o processo indenidamente. Pede-se:

(a) Determine uma outra equação que, como x2 = 0, nos per-

mita continuar o processo indenidamente;

(b) Determine todas as equações do segundo grau completas a

partir das quais possamos continuar o processo indenida-

mente.

Page 100: Resolução de problemas

Referências Bibliográcas

[1] AIGNER, M. e ZIEGLER, G. (2002). As Provas estão

no Livro. Edgard Blücher.

[2] GARCIA, A. e LEQUAIN, I. (2003). Elementos de Ál-

gebra. Projeto Euclides, IMPA.

[3] LIMA, E. L.; CARVALHO, P. C. P.; WAGNER, E. e

MORGADO, A.C. (2004). A Matemática do Ensino Mé-

dio. Volume 1. Sociedade Brasileira de Matemática.

[4] LIMA, E.L.; CARVALHO, P. C. P.; WAGNER, E. e

MORGADO, A.C. (2004). A Matemática do Ensino Mé-

dio. Volume 2. Sociedade Brasileira de Matemática.

[5] LIMA,E.L.; CARVALHO,P. C. P.; WAGNER,E. e

MORGADO,A.C. (2004). A Matemática do Ensino Mé-

dio. Volume 3. Sociedade Brasileira de Matemática.

[6] LIMA, E.L.; CARVALHO, P. C. P.; WAGNER,E. e

MORGADO, A.C. (2001). Temas e Problemas. Socie-

dade Brasileira de Matemática.

[7] LIMA, E.L. (2001). Álgebra Linear. Sociedade Brasileira

de Matemática.

285

Page 101: Resolução de problemas

286 REFERÊNCIAS BIBLIOGRÁFICAS

[8] MORAIS FILHO, D. C. (2007). Um Convite à Matemá-

tica. EDUFCG.

[9] MORGADO, A.; CARVALHO, J.; CARVALHO, P.;

FERNANDEZ, P. (1991). Análise Combinatória e Pro-

babilidade . Sociedade Brasileira de Matemática.

[10] RIBENBOIM, P. (2001). Números Primos: Mistérios e

Recordes. Sociedade Brasileira de Matemática.

[11] SANTOS, J. P. O. (1993) Introdução à Teoria dos Nú-

meros. IMPA.

[12] SANTOS, J. P. O.; MELLO, M. P. e MURARI, I. T.

C. (2006). Introdução à Análise Combinatória. Editora

Unicamp.

[13] SOARES, M. G. (2005). Cálculo em uma Variável Com-

plexa. Sociedade Brasileira de Matemática.

Page 102: Resolução de problemas

Mestrado Profissional

em Matemática em Rede Nacional

Iniciação à Matemática

Autores:

Krerley Oliveira Adán J. Corcho

Unidade II:

Capítulos III e IV

Page 103: Resolução de problemas

3

Divisibilidade

Os números governam o mundo. PlatãoA teoria dos números é o ramo da Matemática que estuda os mis-

térios dos números e teve sua origem na antiga Grécia. Os belíssimos

problemas ligados a esta área constituem, até hoje, uma das princi-

pais fontes inspiradoras dos amantes da Matemática. Além disso, essa

área possui várias aplicações úteis a humanidade, como por exemplo,

o processo de criptograa usado em transações pela Internet.

Alguns problemas em teoria dos números demoram séculos para

serem resolvidos, como por exemplo o último teorema de Fermat, que

arma que não existe nenhum conjunto de inteiros positivos x, y, z e n

com n maior que 2 que satisfaça xn + yn = zn. Esse problema foi ob-

jeto de fervorosas pesquisas durante mais de 300 anos e foi nalmente

demonstrado em 1995 pelo matemático Andrew Wiles.

Ainda hoje persistem muitas questões naturais e simples sem res-

posta. Por exemplo, ninguém sabe mostrar (apesar de todo mundo

89

Page 104: Resolução de problemas

90 3 Divisibilidade

acreditar que é verdade!) que todo natural par é soma de dois pri-

mos. Essa é a famosa conjectura de Goldbach. Essa simplicidade de se

anunciar problemas e a extrema diculdade em resolvê-los faz desta

área um grande atrativo para os matemáticos do mundo todo.

Este capítulo será dedicado ao estudo de algumas propriedades

básicas relativas aos números inteiros.

3.1 Conceitos Fundamentais e Divisão Eu-

clidiana

Denotamos por Z o conjunto dos números inteiros formado pelo con-

junto dos números naturais N = 1, 2, 3, . . . munido do zero e dos

números negativos. Ou seja, Z = . . . ,−3,−2,−1, 0, 1, 2, 3, . . ..Começamos observando que a soma, diferença e produto de núme-

ros inteiros também serão números inteiros. Entretanto, o quociente

de dois inteiros pode ser um inteiro ou não.

Uma das propriedades fundamentais dos números naturais que uti-

lizaremos ao longo do texto é o conhecido princípio da boa ordenação,

que arma o seguinte:

Princípio da Boa Ordenação: todo subconjunto não vazio A ⊆ Npossui um elemento menor que todos os outros elementos deste, ou

seja, existe a ∈ A tal que a ≤ n para todo n ∈ A.Por exemplo, se A é o conjunto dos números pares, o menor ele-

mento de A é o número 2. Por outro lado, observamos que o conjunto

dos números inteiros não goza da boa ordenação.

Apesar do princípio da boa ordenação parecer inocente e natural,

muitos resultados importantes a respeito dos números naturais decor-

Page 105: Resolução de problemas

3.1 Conceitos Fundamentais e Divisão Euclidiana 91

rem do mesmo, como veremos ao longo de todo este capítulo.

Denição 3.1. Sejam a e b inteiros. Dizemos que a divide b se existe

um inteiro q tal que b = aq. Também usaremos as frases a é divisor

de b ou b é múltiplo de a para signicar esta situação.

Usaremos a notação a | b para representar todas as frases equi-

valentes ditas anteriormente. Se a não for divisor de b, então escre-

veremos a - b.

Exemplo 3.2. 7 | 21 pois 21 = 7 · 3. Por outro lado 3 - 8 pois

considerando o conjunto M = 3m, m ∈ N = 3, 6, 9, 12, . . . dos

múltiplos positivos de 3 vemos que 8 não pertence ao mesmo.

A seguinte proposição é um bom exercício para entender os con-

ceitos enunciados acima.

Proposição 3.3. Sejam a, b e c números inteiros. Então,

(a) se a | b e b | c então a | c;

(b) se a | b e a | c então a | (b+ c) e a | (b− c);

(c) se a e b são positivos e a | b então 0 < a ≤ b;

(d) se a | b e b | a então a = b ou a = −b.

Demonstração. Se a | b e b | c então existem inteiros q1 e q2 tais que

b = aq1 (3.1)

e

c = bq2. (3.2)

Page 106: Resolução de problemas

92 3 Divisibilidade

Substituindo (3.1) em (3.2) temos que

c = aq1q2 = aq, onde q = q1q2 ∈ Z, (3.3)

provando isto a armação feita em (a).

Agora provaremos (b). Com efeito, se a | b e a | c valem as

igualdades

b = aq1, q1 ∈ Z (3.4)

e

c = aq2, q2 ∈ Z. (3.5)

Operando com os ambos lados das igualdades (3.4) e (3.5) temos que

b+ c = a(q1 + q2︸ ︷︷ ︸r∈Z

) e b− c = a(q1 − q2︸ ︷︷ ︸s∈Z

),

obtendo assim o resultado desejado.

Continuamos agora com a prova de (c). De fato, se a | b, sendoambos positivos, então b = aq com

q ≥ 1. (3.6)

Logo, multiplicando por a ambos lados de (3.6) temos (como a é posi-

tivo) que

b = aq ≥ a > 0,

como esperávamos.

Finalmente, provaremos (d). Com este propósito observamos que

se a | b e b | a então |a| divide |b| e |b| divide |a|. Portanto, pelo item

(c) temos que |a| ≤ |b| e |b| ≤ |a|, ou seja, |a| ≤ |b| ≤ |a|. Logo,

|a| = |b| e consequentemente a = b ou a = −b.

Page 107: Resolução de problemas

3.1 Conceitos Fundamentais e Divisão Euclidiana 93

Exemplo 3.4. Prove que o número N = 545362−7 não é divisível por

5.

Solução. Vamos mostrar isso utilizando o método do absurdo. Se

este número fosse divisível por 5, então 545362 − 7 = 5q. Logo, 7 =

545362 − 5q, ou seja, 7 seria divisível por 5, o que é um absurdo.

O próximo passo de nossa discussão é ver o que acontece quando

um número não é divisível por outro. Por exemplo, analisemos se 31 é

divisível por 7 e para isto listaremos a diferença entre 31 e os múltiplos

positivos de 7, isto é:

r1 = 31− 7 · 1 = 24,

r2 = 31− 7 · 2 = 17,

r3 = 31− 7 · 3 = 10,

r4 = 31− 7 · 4 = 3,

r5 = 31− 7 · 5 = −4,r6 = 31− 7 · 6 = −11,...

Claramente 31 não é divisível por 7, pois caso contrário teríamos

que alguma das diferenças acima seria igual a zero, o que é impossível

pois as diferenças rq = 31 − 7q com 1 ≤ q ≤ 4 são todas positivas

e com q ≥ 5 são todas negativas. Entretanto, notamos que entre as

diferenças positivas a única que é menor que 7 corresponde ao caso

q = 4. O resultado seguinte nos diz o que acontece no caso geral da

divisão de um inteiro b por um inteiro positivo a.

Page 108: Resolução de problemas

94 3 Divisibilidade

Teorema 3.5 (Divisão Euclidiana). Dados dois inteiros a e b, sendo

a positivo, existem únicos inteiros q e r tais que

b = aq + r, 0 ≤ r < a.

Se a - b, então r satisfaz a desigualdade estrita 0 < r < a.

Demonstração. Por simplicidade, suporemos que b é positivo. Se b <

a, basta tomar q = 0 e r = b. Se b = a, então tomamos q = 1 e r = 0.

Assim, assumiremos também que b > a > 0. Consideremos o conjunto

R = b− aq ∈ Z; b− aq ≥ 0 ⊆ N ∪ 0 (3.7)

Notemos que o conjunto R é não vazio, pois b − a ∈ R, já que

b − a > 0. Deste modo, pelo princípio da boa ordenação temos que

R admite um menor elemento, que denotaremos por r. Claramente

r = b − aq ≥ 0, para algum q ≥ 0. Além disso, r < a pois caso

contrário

r = b− aq ≥ a⇒ b− a(q + 1) ≥ 0. (3.8)

Por outro lado,

a > 0⇒ b− a(q + 1) < b− aq. (3.9)

Das desigualdades (3.8) e (3.9) segue que

0 ≤ b− a(q + 1) < b− aq,

contradizendo o fato de que r = b−aq é o menor elemento não negativo

de R.

Agora provaremos que de fato r e q, escolhidos desta forma, são

únicos. Com efeito, suponhamos que existem outros inteiros r1 e q1

tais que

b = aq1 + r1, 0 ≤ r1 < a.

Page 109: Resolução de problemas

3.1 Conceitos Fundamentais e Divisão Euclidiana 95

Então resulta que aq + r = aq1 + r1. Logo,

(r − r1) = (q1 − q)a; (3.10)

sendo assim, r−r1 é múltiplo de a. Mas, em virtude de−a < r−r1 < a,

o único valor que r − r1 pode tomar, sendo este múltiplo de a, é

r − r1 = 0. Portanto, r = r1, de onde se deduz diretamente de (3.10)

que q = q1.

Os números q e r no enunciado do teorema acima são chamados,

respectivamente, de quociente e resto da divisão de b por a.

Um resultado imediato da divisão euclidiana é o seguinte.

Corolário 3.6. Dados dois números naturais a e b com 1 < a ≤ b,

existe um número natural n tal que

na ≤ b < (n+ 1)a.

Demonstração. Pela divisão euclidiana, existem únicos q, r ∈ N com

0 ≤ r < a tais que b = aq + r. Assim

aq ≤ b = aq + r < aq + a = a(q + 1).

Basta agora tomar q = n para obtermos o resultado.

Os exemplos a seguir apresentam a utilidade do Teorema 3.5.

Exemplo 3.7. Se a é um natural com a ≥ 3, então a2 deixa resto 1

na divisão por a− 1. Consequentemente, a− 1 divide a2 − 1.

Solução. Usando a identidade a2 − 1 = (a − 1)(a + 1) temos que

a2 = (a− 1)(a+ 1) + 1 com 1 < a− 1, de onde segue o resultado.

Page 110: Resolução de problemas

96 3 Divisibilidade

O próximo exemplo, como veremos, motiva a procura de cami-

nhos ecientes para encontrar o resto que deixa um número quando é

dividido por outro.

Exemplo 3.8. Um turista brasileiro chega a Cuba e troca parte de

seu dinheiro na casa de câmbio, recebendo 175 notas de 50 pesos e

213 notas de 20 pesos. Ele decide trocar este dinheiro pela maior

quantidade possível das famosas moedas de 3 pesos cubanos, porque

elas têm gravada a imagem do guerrilheiro Che Guevara. Quanto

sobrou do dinheiro depois de fazer a troca pelas moedas?

Solução. Para resolver este problema basta achar o resto que deixa o

número n = 175 · 50 + 213 · 20 quando é dividido por 3. Entretanto,

queremos destacar que não é preciso fazer os produtos e a soma envol-

vidos no número n. Em lugar de fazer isto substituímos cada número

que aparece em n pelo resto que este deixa na divisão por 3, formando

assim um novo número n1, ou seja,

n1 = 1 · 2 + 0 · 2 = 2.

Agora procuramos o resto que n1 deixa na divisão por 3, que obvi-

amente é 2. A surpresa é que este resto é o mesmo que deixa n na

divisão por 3. Logo, sobraram 2 pesos depois de fazer a troca.

A solução do exemplo anterior é uma aplicação particular do se-

guinte lema que é de muita utilidade na resolução de problemas.

Lema 3.9 (Lema dos Restos). A soma e o produto de quaisquer dois

números naturais deixa o mesmo resto que a soma e o produto dos

seus restos, na divisão por um inteiro positivo a.

Page 111: Resolução de problemas

3.1 Conceitos Fundamentais e Divisão Euclidiana 97

Demonstração. Sejam n1, n2 ∈ Z. Fazendo a divisão com resto de

ambos os números por a temos que

n1 = aq1 + r1 e n2 = aq2 + r2,

com 0 ≤ r1, r2 < a. Então,

n1n2 = (aq1 + r1)(aq2 + r2)

= a2q1q2 + aq1r2 + aq2r1 + r1r2

= a(aq1q2 + q1r2 + q2r1) + r1r2

= aq + r1r2,

(3.11)

onde q = aq1q2+q1r2+q2r1. Agora dividimos r1r2 por a para obtermos

r1r2 = ap+ r, p ∈ Z, 0 ≤ r < a. (3.12)

Das igualdades (3.11) e (3.12) segue que

n1n2 = aq + ap+ r = a(p+ q) + r, 0 ≤ r < a. (3.13)

Portanto, de (3.12) e (3.13) concluímos que os restos que deixam n1n2

e r1r2 na divisão por a são iguais, cando provado o resultado para o

produto. A prova para a soma é análoga.

Observação 3.10. A vantagem do lema é que em certos problemas

que envolvem números muito grandes podemos substituir estes por nú-

meros muito menores e mais confortáveis para trabalhar.

Vejamos como aplicar o lema dos restos nos seguintes exemplos a

seguir.

Exemplo 3.11. Prove que o produto de dois números naturais con-

secutivos é sempre divisível por 2.

Page 112: Resolução de problemas

98 3 Divisibilidade

Solução. Se n ∈ N temos que provar que an = n(n+1) é divisível por

2. Quando fazemos a divisão de n por 2 temos duas possibilidades

para o resto: r = 0 ou r = 1. Analisemos os dois casos por separado.

• [r = 0] Neste caso o resto que deixa an na divisão por 2 é o

mesmo que o resto que deixa 0(0+1)=0, logo an é divisível por

2.

• [r = 1] Neste caso podemos substituir an por 1(1+1)=2 e o

resto que este último deixa quando é dividido por 2 é 0, logo antambém é divisível por 2.

Mostraremos agora como utilizar o exemplo anterior pra resolver

um dos problemas da 1a Olimpíada Brasileira de Matemática.

Exemplo 3.12. Prove que se n é ímpar, então n2 − 1 é múltiplo de

8.

Solução. Como n é ímpar, podemos escrever n = 2m+ 1, para algum

k ∈ Z. Logo

n2 − 1 = (2m+ 1)2 − 1 = 4m2 + 4m+ 1− 1 = 4m2 + 4m.

Assim,

n2 − 1 = 4m(m+ 1).

Observe que de acordo com o exemplo 3.11, m(m + 1) é múltiplo de

2. Portanto, m(m+ 1) = 2q para algum q ∈ Z, de aonde

n2 − 1 = 4m(m+ 1) = 4 · 2q = 8q,

como queríamos demonstrar.

Page 113: Resolução de problemas

3.2 Bases Numéricas 99

Exemplo 3.13. Prove que em qualquer triângulo retângulo com lados

inteiros, pelo menos um deles é múltiplo de 3.

Solução. Comecemos analisando quais são os restos possíveis para a

divisão por 3 de um número que é quadrado. De acordo com o lema

dos restos temos a seguinte tabela para os restos de n e n2, na divisão

por 3:

n n2

0 0

1 1

2 1

Resumindo, se um número não é múltiplo de 3 então o resto da divisão

de seu quadrado por 3 deve ser igual a 1.

Agora denotemos por a e b os catetos e por c a hipotenusa. Supo-

nhamos que nenhum deles é divisível por 3. Então a2 e b2 deixam resto

1 na divisão por 3. Logo, a2 + b2 deixa resto 12 + 12 = 2 na divisão

por 3; mas isto é uma contradição pois, pelo Teorema de Pitágoras,

a2 + b2 = c2 e c2 deixa resto 1 quando é dividido por 3.

3.2 Bases Numéricas

Começamos esta seção com uma brincadeira interessante.

João, ao sair da aula de matemática do professor Peitágoras, en-

controu Pedro e lhe propôs a seguinte brincadeira:

Pense numa peça de dominó, Pedro. Vou adivinhar que peça é

essa usando uma fórmula mágica.

Ok, João. Pode começar, já pensei.

Page 114: Resolução de problemas

100 3 Divisibilidade

x y

Figura 3.1: Peça de Dominó

- Escolha um dos números na peça e multiplique por 5. Depois

disso some três a esse resultado. Multiplique agora o número que você

obteve por dois. Some isto com o outro número da peça. Qual foi o

resultado?

Foi 40.

Então a peça que você escolheu foi a 3 com 4!

Como você acertou? Me ensina!

Claro que de mágico João não tinha nada e decidiu contar seu

segredo a Pedro.

O jogo funciona assim: cada parte da peça de dominó pode ser

considerada como um dos dígitos de um número de 2 algarismos, o qual

denotamos por n = xy = 10x+ y (veja a Figura 3.1). Acompanhando

os passos de João, temos que:

(5x+ 3)2 + y = 40⇔ 10x+ y = 34, (3.14)

que claramente, tem por soluções: x = 3 e y = 4, usando a represen-

tação de 34 na base decimal.

No sistema de numeração decimal, também conhecido como sis-

tema numérico na base 10, todo número pode ser representado como

uma sequência de 10 símbolos, constituídos pelo 0 (zero) e os alga-

rismos 1, 2, 3, . . . , 9. Por exemplo, 345 escreve-se na base decimal da

Page 115: Resolução de problemas

3.2 Bases Numéricas 101

seguinte forma

345 = 300 + 40 + 5 = 3 · 102 + 4 · 10 + 5,

assim como 2768 se escreve da forma

2768 = 2000 + 700 + 60 + 8 = 2 · 103 + 7 · 102 + 6 · 10 + 8.

De modo geral, se denotamos por a = anan−1 . . . a1a0 o número inteiro

positivo formado pelos algarismos an, an−1, . . . , a1 e a0, nessa ordem,

então a se escreve na base decimal da forma

a = an10n + an−110

n−1 + . . .+ a110 + a0 (3.15)

Antes de provar alguns dos critérios de divisibilidade mais po-

pulares do sistema de numeração decimal, provamos uma identidade

muito útil.

Lema 3.14. Sejam a, b, n ∈ N. Temos que

an − bn = (a− b)(an−1 + an−2b+ · · ·+ abn−2 + bn−1).

Consequentemente, se 0 < b < a, então a− b divide an − bn.

Demonstração. Primeiro provaremos que a propriedade vale para b =

1. Com efeito, considerando a soma geométrica

s = 1 + a+ a2 + · · ·+ an−1

e multiplicando s por a temos que

as = (a+ a2 + · · ·+ an−1) + an = s− 1 + an.

Page 116: Resolução de problemas

102 3 Divisibilidade

Assim, (a− 1)s = as− s = an − 1, de onde se segue que

an − 1 = (a− 1)(an−1 + an−2 + · · ·+ a+ 1). (3.16)

Daí temos a validade para b = 1.

Para b ∈ N qualquer, observe que an− bn = bn[(ab)n − 1

]. Usando

esta expressão e (3.16) tem-se

an − bn = bn(ab− 1)

[(ab)n−1 + (a

b)n−2 + · · ·+ (a

b) + 1

]= (a− b)bn−1

[(ab)n−1 + (a

b)n−2 + · · ·+ (a

b) + 1

]= (a− b)(an−1 + an−2b+ · · ·+ abn−2 + bn−1),

(3.17)

obtendo-se a igualdade clamada.

Proposição 3.15 (Critérios de Divisibilidade). Seja a = an . . . a1a0

um inteiro positivo, então

(a) a é divisível por 10 se, e somente se, a0 for igual a 0;

(b) a é divisível por 3 ou por 9 se, e somente se, a soma dos seus

dígitos é divisível por 3 ou por 9, respectivamente;

(c) a é divisível por 5 se, e somente se, a0 for igual a 0 ou 5.

Demonstração. Utilizando a representação decimal (3.15) temos que

a = 10(an10n−1 + an−110

n−2 + · · ·+ a1) + a0.

Então, pela Proposição 3.3-(b) tem-se que 10 | a se, e somente se,

10 | a0, prondose-se assim o critério (a).

Para provar (b) observemos que

a = an10n + an−110

n−1 + · · ·+ 10a1 + a0

= an(10n − 1) + an−1(10

n−1 − 1) + · · ·+ (10− 1)a1

+ an + an−1 + · · ·+ a1 + a0.

(3.18)

Page 117: Resolução de problemas

3.2 Bases Numéricas 103

Pelo Lema 3.14 temos que 10j − 1 = 9qj para todo 1 ≤ j ≤ n, daí

segue-se

a = 9(anqn + an−1qn−1 + · · ·+ a1) + an + an−1 + · · ·+ a1 + a0.

Então, aplicando novamente o item (b) da Proposição 3.3 temos que

9 | a se, e somente se, 9 | (an + an−1 + · · ·+ a1 + a0).

A prova para o caso da divisibilidade por 3 segue de maneira idêntica,

logo ca provado o item (b).

A prova do item (c) segue de maneira muito semelhante e deixamos

a mesma a cargo do leitor.

Exemplo 3.16. Prove sem fazer muitas contas que o número

N = 13424136 + 1234567890

é divisível por 3.

Solução. Note que não precisamos fazer a soma dos números ante-

riores. Para mostrar isso, basta aplicar o item (b) da Proposição 3.3 e

o item (b) da Proposição 3.15, observando que cada um dos números

acima é divisível por 3, pois a soma de seus dígitos é um múltiplo de

3.

Finalizamos esta seção com uma aplicação da divisão euclidiana

que nos mostra que, analogamente à representação decimal, qualquer

número admite uma representação única em qualquer outra base nu-

mérica.

Page 118: Resolução de problemas

104 3 Divisibilidade

Teorema 3.17 (Bases Numéricas). Dados a, b ∈ N, com b > 1, exis-

tem únicos números naturais r0, r1, . . . , rn tais que 0 ≤ ri ≤ b− 1,

0 ≤ i ≤ n, e satisfazendo

a = rnbn + rn−1b

n−1 + · · ·+ r1b+ r0.

A representação acima é dita representação de a na base b e usaremos

a notação

a = (rncn−1 . . . r1r0)b,

para fazer referência a esta.

Demonstração. Apliquemos sucessivamente a divisão euclidiana como

segue:

a = bq0 + r0, r0 < b,

q0 = bq1 + r1, r1 < b,

q1 = bq2 + r2, r2 < b,

......

......

qj−1 = bqj + rj, rj < b,

e assim por diante. Como a > q0 > q1 > q2 > · · · > qj−1, para algum

j = n deveremos ter que qn−1 < b. Logo, qj = 0 para todo j ≥ n,

assim como rj = 0 para todo j ≥ n + 1. Das igualdades acima, para

Page 119: Resolução de problemas

3.2 Bases Numéricas 105

1 ≤ j ≤ n, tem-se

a = bq0 + r0,

bq0 = b2q1 + br1,

b2q1 = b3q2 + b2r2,

......

...

bn−1qn−2 = bnqn + bn−1rn−1

bnqn−1 = bn+10 + bnrn.

(3.19)

Efetuando a soma de todas as igualdades em (3.19) obtemos

a = rnbn + rn−1b

n−1 + · · ·+ r1b+ r0.

A unicidade dos números ri vem da unicidade dos restos na divisão

euclidiana.

Observação 3.18. O sistema de numeração na base 2 é também co-

nhecido como sistema binário e é o sistema habitualmente utilizado no

funcionamento dos computadores.

Exemplo 3.19. Se deseja pesar qualquer número inteiro de gramas de

ouro, entre 1g e 100g, numa balança de dois pratos, onde os pesos só

podem ser usados no prato esquerdo da balança. Mostre que a escolha

adequada de 7 pesos diferentes é suciente para realizar esta tarefa.

Demonstração. Usando o sistema de numeração em base 2 temos que

qualquer número a tal que 1 ≤ a ≤ 100 pode ser expressado de forma

única como

a = r626 + r52

5 + r424 + r32

3 + r222 + r12 + r01,

Page 120: Resolução de problemas

106 3 Divisibilidade

com ri ∈ 0, 1, 0 ≤ i ≤ 1. Observe que 2n ≥ 128, com n ≥ 7, logo

estas potências não são consideradas. notemos também que o fato de

cada ri ser 0 ou 1 nos diz que não precisamos repetir nenhum dos

pesos na realização de qualquer pesada. Logo, os pesos

1, 22, 23, 24, 25, 26

são sucientes para realizar as pesadas de gramas de ouro entre 1g e

100g.

3.3 Máximo Divisor Comum eMínimoMúl-

tiplo Comum

Nesta seção estudaremos dois conceitos fundamentais, que aparecem

naturalmente em vários problemas de divisibilidade, assim como a

relação existente entre eles.

3.3.1 Máximo Divisor Comum

O primeiro destes conceitos está relacionado com os inteiros positivos

que dividem simultaneamente a dois inteiros prexados e é denomi-

nado máximo divisor comum.

Daqui por diante só consideraremos os divisores positivos dos nú-

meros.

Denição 3.20 (Máximo Divisor Comum). Sejam a e b inteiros dife-

rentes de zero. O máximo divisor comum, resumidamente mdc, entre

a e b é o número d que satisfaz as seguintes condições:

(a) d é um divisor comum de a e b, isto é, d | a e d | b;

Page 121: Resolução de problemas

3.3 Máximo Divisor Comum e Mínimo Múltiplo Comum 107

(b) d é o maior inteiro positivo com a propriedade (a).

Neste caso, denotamos o mdc entre a e b por d = mdc(a, b) ou por

d = (a, b). Se (a, b) = 1, então dizemos que a e b são primos entre si.

Exemplo 3.21. Observando que 12 = 6 · 2, 18 = 6 · 3 temos que

mdc.(12, 18) = 6. Por outro lado, mdc.(4, 15) = 1, logo os números 4

e 15 são primos entre si.

Vejamos agora algumas das propriedades mais importantes dos

divisores comuns de dois inteiros.

Proposição 3.22. Sejam a e b dois inteiros. Então valem as seguintes

armações.

(a) Se a é múltiplo de b, então (a, b) = b.

(b) Se a = bq+ c, c 6= 0, então o conjunto dos divisores comuns dos

números a e b coincide com o conjunto dos divisores comuns dos

números b e c. Particularmente, (a, b) = (b, c).

Demonstração. Começamos com a prova de (a). Com efeito, todo

divisor comum dos números a e b é um divisor de b. Reciprocamente,

usando que a é múltiplo de b, todo divisor de b é também um divisor

de a, ou seja, um divisor comum dos números a e b. Portanto, o

conjunto dos divisores comuns dos números a e b é igual ao conjunto

dos divisores de b. Como o maior divisor de b é ele mesmo, resulta que

(a, b) = b.

Vejamos (b). Usando o item (b) da Proposição 3.3 temos que

todo divisor comum de a e b também divide c e, consequentemente, é

um divisor de b e c. Pela mesma razão todo divisor comum de b e c

também divide a e, consequentemente, é um divisor de a e b. Portanto

Page 122: Resolução de problemas

108 3 Divisibilidade

os divisores comuns de a e b são os mesmos que os divisores comuns

de b e c. Particularmente, também coincidem os maiores divisores

comuns, ou seja, (a, b) = (b, c).

O teorema a seguir é uma das ferramentas básicas na resolução

de problemas que envolvem o mdc entre dois números. O resultado

foi provado pela primeira vez por Claude-Gaspard Bachet de Méziriac

(1581-1638) e mais tarde generalizado para polinômios por Étienne

Bézout (1730-1783). Frequentemente, na literatura se enuncia este

resultado como teorema (ou identidade) de Bézout, esquecendo-se o

nome de Bachet.

Teorema 3.23 (Teorema de Bachet-Bézout). Se d é o mdc de a e b,

então existem números inteiros x0 e y0 tais que d = (a, b) = ax0+ by0.

Demonstração. Considere a combinação linear ax + by, onde x e y

percorrem todos os inteiros. Este conjunto de inteiros, denotado por

Ca,b = ax+ by; x, y ∈ Z,

inclui valores positivos e negativos. Além disso, escolhendo x = y = 0,

vemos que Ca,b também contém o zero.

Pelo princípio da boa ordenação, podemos escolher x0 e y0 tais que

λ = ax0+by0 seja o menor número inteiro positivo contido no conjunto

Ca,b.

Agora mostraremos que λ | a e λ | b. Provaremos que λ | a e o

outro segue analogamente. Usaremos para este propósito o método de

redução ao absurdo, ou seja, vamos supor que λ - a e obteremos uma

contradição.

Page 123: Resolução de problemas

3.3 Máximo Divisor Comum e Mínimo Múltiplo Comum 109

Usando a divisão euclidiana, de λ - a segue que existem inteiros q

e r tais que a = λq + r com 0 < r < λ. Portanto,

r = a− λq = a− q(ax0 + by0) = a(1− qx0) + b(−qy0)

e assim r está no conjunto Ca,b, o que contradiz a hipótese de λ ser o

menor elemento positivo contido em Ca,b.

Uma vez que λ divide a e b só resta provar que λ = d. Com efeito,

desde que d = (a, b), podemos escrever a = da1, b = db1 e

λ = ax0 + by0 = d(a1x0 + b1y0).

Assim d | λ. Logo pela parte (c) da Proposição 3.3, concluímos que

d ≤ λ. Agora d < λ é impossível pois d = mdc(a, b), e portanto

d = λ = ax0 + by0.

A seguinte proposição resume algumas consequências importantes

da demonstração dada ao teorema de Bézout.

Proposição 3.24. Sejam d, λ ∈ N e a, b, c ∈ Z. Então valem as

seguintes armações:

(a) Se d | a e d | b, então d | (a, b).

(b) O mdc.(a, b) é o menor valor positivo de ax + by, onde x e y

percorrem todos os números inteiros.

(c) (λa, λb) = λ(a, b).

(d) Se d | a e d | b, então (ad, bd) = 1

d(a, b). Consequentemente,(

a

(a, b),

b

(a, b)

)= 1.

Page 124: Resolução de problemas

110 3 Divisibilidade

(e) Se (a, c) = (b, c) = 1, então (ab, c) = 1.

(f) Se c | ab e (b, c) = 1, então c | a.

Demonstração. A prova de (a) é consequência imediata da igualdade

(a, b) = ax0 + by0 anunciada no teorema de Bézout; assim como (b)

segue diretamente da demonstração dada a este teorema.

Para provar (c), primeiro observamos que

(λa)x+ (λb)y = λ(ax+ by) onde x, y ∈ Z.

Usando o item (a) e o fato de λ ser positivo, da igualdade acima segue

que

(λa, λb) = min(λa)x+ (λb)y > 0; x, y ∈ Z

= λmin

ax+ by ; x, y ∈ Z

= λ(a, b).

A armação feita em (d) segue diretamente de (c), observando que

(a, b) =

(da

d, db

d

)= d

(a

d,b

d

).

Continuamos com a prova de (e). De (a, c) = (b, c) = 1, temos que

existem inteiros xj e yj, j = 1, 2, tais que

ax1 + cy1 = 1,

bx2 + cy2 = 1.

Multiplicando lado a lado as igualdades obtemos

(x1x2︸︷︷︸x

)ab+ (ax1y2 + y1bx2 + cy1y2︸ ︷︷ ︸y

)c = 1.

Page 125: Resolução de problemas

3.3 Máximo Divisor Comum e Mínimo Múltiplo Comum 111

Então, usando o item (b) e a igualdade acima resulta que (ab, c) = 1.

Finalmente, provaremos (f). Das hipóteses temos que existem in-

teiros x0 e y0 tais que

bx0 + cy0 = 1.

Multiplicamos a igualdade acima por a em ambos lados para obtermos

abx0 + acy0 = a.

Por outro lado, ab = cq para algum inteiro q. Usando esta condição

na última igualdade temos que

cqx0 + acy0 = c(qx0 + ay0) = a,

logo c | a.

3.3.2 Algoritmo de Euclides

Apesar de conhecermos propriedades teóricas do mdc entre dois intei-

ros, encontrá-lo de fato pode ser uma tarefa complicada, sem auxílio

das ferramentas corretas. Lembrando o seu signicado, o leitor talvez

pudesse pensar que devemos calcular todos os divisores de a, todos

os divisores de b e descobrir qual é o maior elemento comum aos dois

conjuntos.

Para achar o mdc se faz uso de um importante método denominado

algoritmo de Euclides .

Teorema 3.25 (Algoritmo de Euclides). Dados dois inteiros positivos,

a e b, aplicamos sucessivamente a divisão euclidiana para obter a se-

Page 126: Resolução de problemas

112 3 Divisibilidade

guinte sequência de igualdades

b = aq1 + r1, 0 ≤ r1 < a,

a = r1q2 + r2, 0 ≤ r2 < r1,

r1 = r2q3 + r3, 0 ≤ r3 < r2,

· · · · · · · · · · · · · · ·rn−2 = rn−1qn + rn, 0 ≤ rn < rn−1,

rn−1 = rnqn+1,

(3.20)

até algum rn dividir rn−1. Assim, o mdc.(a, b) = rn, ou seja, é o

último resto não-nulo no processo de divisão anterior.

Observação 3.26. Quando lidamos com números pequenos achar o

mdc é uma tarefa fácil pois podemos calcular o mdc valendo-nos das

fatorações dos números envolvidos. No entanto, quando estamos tra-

balhando com números grandes o algoritmo de Euclides, em geral, é

mais fácil que a fatoração, podendo ser esta última bem difícil.

Demonstração do algoritmo de Euclides. Começamos observando que

o processo de divisão (3.20) é nito. Com efeito, a sequência de núme-

ros inteiros rk é estritamente decrescente e está contida no conjunto

r ∈ Z, 0 ≤ r < a, portanto não pode conter mais do que a intei-

ros positivos. Examinando as igualdades (3.20) de cima para baixo e

usando a Proposição 3.22 temos que

(a, b) = (a, r1) = (r1, r2) = · · · = (rn−1, rn) = rn.

Page 127: Resolução de problemas

3.3 Máximo Divisor Comum e Mínimo Múltiplo Comum 113

Observação 3.27. Notemos que o teorema de Bézout também pode

ser obtido como consequência do processo de divisão (3.20). Com

efeito, podemos escrever

rn = rn−2 − rn−1qnrn−1 = rn−3 − rn−2qn−1

⇒ rn = rn−2 − (rn−3 − rn−2qn−1)qn.

Logo, conseguimos escrever rn em termos de rn−2 e rn−3. Utilizando a

expressão rn−2 = rn−4− rn−3qn−2 podemos escrever rn como combina-

ção de rn−3 e rn−4. Repetindo este processo várias vezes, concluímos

que existem x, y ∈ Z tais que

d = rn = xr1 + yr2.

Ora, como r1 = b − aq1 e r2 = a − r1q2 = a(1 + q1q2) − bq2, então,substituindo estes valores na última igualdade obtemos o Teorema de

Bézout.

Exemplo 3.28. Achar o máximo divisor comum dos números 471 e

1.176.

Solução. Aplicando o algoritmo de Euclides obtemos a seguinte sequên-

cia de divisões com resto:

1176 = 471 · 2 + 234,

471 = 234 · 2 + 3,

234 = 78 · 3,

então o mdc(471, 1176) = 3.

Exemplo 3.29. Provar que a fração2n+ 8

4n+ 15é irredutível para todo

número natural n.

Page 128: Resolução de problemas

114 3 Divisibilidade

Solução. Usando o algoritmo de Euclides temos que

4n+ 15 = (2n+ 8) · 1 + 2n+ 7,

2n+ 8 = (2n+ 7) · 1 + 1,

2n+ 7 = (2n+ 7) · 1.

Então o mdc.(4n + 15, 2n + 8) = 1 e portanto 4n + 15 e 2n + 8 são

primos entre si para qualquer valor de n.

Exemplo 3.30. Achar o mdc.(111 . . . 111︸ ︷︷ ︸100 vezes

, 11 . . . 11︸ ︷︷ ︸60 vezes

)

Solução. Primeiro escrevemos os números na base decimal, isto é,

111 . . . 111︸ ︷︷ ︸100 vezes

= 1099 + 1098 + · · ·+ 1,

11 . . . 11︸ ︷︷ ︸60 vezes

= 1059 + 1058 + · · ·+ 1.

Aplicamos agora o algoritmo de Euclides para obter as seguintes igual-

dades

111 . . . 111︸ ︷︷ ︸100 vezes

= (1059 + 1058 + · · ·+ 1)1040 + 1039 + 1038 + · · ·+ 1,

1059 + 1058 + · · ·+ 1 = (1039 + 1038 + · · ·+ 1)1020+

+ 1019 + 1018 + · · ·+ 1,

1039 + 1038 + · · ·+ 1 = (1019 + 1018 + · · ·+ 1)1020+

+ 1019 + 1018 + · · ·+ 1.

Disso resulta que

mdc.(111 . . . 111︸ ︷︷ ︸100 vezes

, 11 . . . 11︸ ︷︷ ︸60 vezes

) = 1019 + 1018 + · · ·+ 1 = 11 . . . 11︸ ︷︷ ︸20 vezes

.

Page 129: Resolução de problemas

3.3 Máximo Divisor Comum e Mínimo Múltiplo Comum 115

3.3.3 Mínimo Múltiplo Comum

Agora passamos ao segundo conceito importante desta seção. O mesmo

está relacionado com os inteiros positivos que são simultaneamente

múltiplos de dois inteiros prexados e é denominado mínimo múltiplo

comum.

Denição 3.31 (Mínimo Múltiplo Comum). Sejam a e b inteiros

diferentes de zero. O mínimo múltiplo comum, resumidamente mmc,

entre a e b é o inteiro positivo m que satisfaz as seguintes condições:

(a) m é um múltiplo comum de a e b, isto é, a | m e b | m;

(b) m é o menor inteiro positivo com a propriedade (a).

Neste caso, denotamos o mmc entre a e b por m = mmc(a, b) ou por

m = [a, b].

Resumimos a seguir algumas das propriedades fundamentais do

mmc de dois inteiros.

Proposição 3.32. Sejam a, b, c ∈ Z e λ ∈ Z. Então valem as se-

guintes armações:

(a) se c é múltiplo comum de a e b, então [a, b] | c;

(b) [λa, λb] = λ[a, b];

(c) |ab| = [a, b] · (a, b).

Demonstração. Começamos com a prova de (a). A divisão com resto

de c por [a, b] nos dá

c = [a, b]q + r, 0 ≤ r < [a, b]. (3.21)

Page 130: Resolução de problemas

116 3 Divisibilidade

Da igualdade anterior, basta provar que r = 0 para obter o resultado

desejado. Suponhamos, pelo contrário, que 0 < r < [a, b]. Notemos

que tanto a como b dividem c e [a, b]. Logo, pelo item (b) da Pro-

posição 3.3 e a igualdade (3.21), temos que a e b também dividem r,

ou seja, r é múltiplo comum de a e b e não pode ser menor que [a, b],

contradizendo nossa suposição.

Prosseguimos com a prova de (b). Observemos que λ[a, b] é múlti-

plo comum de λa e λb, logo pelo item (i) vale que

[λa, λb] ≤ λ[a, b]. (3.22)

Por outro lado, [λa, λb] = q1λa = q2λb, para alguns inteiros q1 e q2;

logo, [λa,λb]λ

é um múltiplo comum de a e b. Portanto,

[a, b] ≤ [λa, λb]

λ⇔ λ[a, b] ≤ [λa, λb]. (3.23)

Das igualdades (3.22) e (3.23) segue que

λ[a, b] ≤ [λa, λb] ≤ λ[a, b],

de onde vem diretamente o resultado.

Para provar (c) podemos supor sem perda de generalidade que a e

b são positivos devido às igualdades

[a, b] = [a,−b] = [−a, b] = [−a,−b].

Dividiremos a prova em dois casos:

Caso 1: (a, b) = 1.

Sabemos que b | [a, b] e [a, b] = qa, para algum q ∈ N. Então b | qae além disso (a, b) = 1. Logo, pelo item (v) da Proposição 3.24 temos

que b | q. Portanto, b ≤ q e consequentemente

ab ≤ aq = [a, b]. (3.24)

Page 131: Resolução de problemas

3.3 Máximo Divisor Comum e Mínimo Múltiplo Comum 117

Entretanto, da denição de [a, b] vale que

[a, b] ≤ ab. (3.25)

Das desigualdades (3.24) e (3.25) segue que ab ≤ [a, b] ≤ ab. Assim,

ab = [a, b] = [a, b] · 1 = [a, b] · (a, b).

Caso 2: (a, b) > 1.

Da parte (c) da Proposição 3.24 sabemos que(

a(a,b)

, b(a,b)

)= 1.

Aplicando o caso anterior vale que

a

(a, b)· b

(a, b)=

[a

(a, b),

b

(a, b)

]·(

a

(a, b),

b

(a, b)

).

Multiplicamos esta última igualdade por (a, b)2 e usamos o item (b)

provado anteriormente, assim como a parte (d) da Proposição 3.24

para obter

ab = (a, b)

[a

(a, b),

b

(a, b)

](a, b)

(a

(a, b),

b

(a, b)

)= [a, b] · (a, b).

Exemplo 3.33. Dois amigos passeiam de bicicleta, na mesma dire-

ção, em torno a uma pista circular. Para dar uma volta completa um

deles demora 15 minutos e o outro demora 18 minutos. Eles partem

juntos e combinam interromper o passeio quando os dois se encontra-

rem pela primeira vez no ponto de partida. Quantas voltas deu cada

um?

Solução. Denotemos por n1 e n2, respectivamente, o número de voltas

que dá cada um dos amigos. Notemos que o tempo total da corrida é

o menor valor positivo de T que satisfaz as igualdades

T = 15n1 = 18n2,

Page 132: Resolução de problemas

118 3 Divisibilidade

ou seja

T = [15, 18] =15 · 18

3= 90.

Portanto, n1 = 6 e n2 = 5.

Finalizamos esta seção com um exemplo que nos fornece uma bela

interpretação geométrica do mínimo múltiplo comum. O mesmo foi

proposto na Olimpíada Brasileira de Matemática.

Exemplo 3.34. Um retângulo de lados inteiros AB = m e CD = n,

é dividido em quadrados de lado 1. Em cada um dos vértices ele possui

um pequeno orifício. Um raio de luz entra no retângulo por um dos

vértices, na direção da bissetriz do ângulo reto, e é reetido sucessi-

vamente nos lados do retângulo. Quantos quadrados são atravessados

pelo raio de luz?

A B

CD

Figura 3.2: Interpretação geométrica do mmc

Solução. Se zermos alguns testes preliminares dando valores a m e

n, veremos que em cada caso a resposta coincidirá com o mmc(m,n).

Provemos que isto de fato vale para m e n quaisquer. Para realizar a

prova nos auxiliaremos da Figura 3.2.

Page 133: Resolução de problemas

3.3 Máximo Divisor Comum e Mínimo Múltiplo Comum 119

Primeiramente, notemos que cada vez que o raio de luz atravessa

um quadrado ele avança uma unidade tanto na direção horizontal como

na direção vertical. Usando este fato fazemos as observações a seguir.

• Se o raio entra pelo vértice A, terá que atravessar m quadrados

até chegar ao lado BC, imediatamente mais m para chegar ao

lado AD, depois mais m para chegar novamente ao lado BC, e

assim sucessivamente. Além disso, depois do raio percorrer pm

quadrados, com p ∈ N, estará batendo no lado BC ou no lado

AD.

• Analogamente o raio baterá no lado AB ou no lado DC se, e

somente se, atravessar qn quadrados, com q ∈ N.

• Somente nos vértices B, C e D do retângulo pode acontecer que

o raio incidente saia do retângulo, terminando assim o processo

de reexão.

Usando as observações acima é fácil ver que o raio chegará a um

vértice quando chegar simultaneamente a dois lados perpendiculares

do retângulo. Portanto, deve ter atravessado um número x de quadra-

dos tal que x = pm = qn, ou seja, x deverá ser um múltiplo comum

de m e n. É claro que a primeira vez que o raio chega a um vértice o

número x é o menor múltiplo comum de m e n, isto é, x = [m,n].

Finalmente, observamos que nenhum dos quadrados é atravessado

duas vezes no percurso do raio de A até bater no primeiro vértice, pois

como vemos na gura numa das direções os quadrados atravessados

serão todos cinzas e na outra direção, serão todos brancos.

Page 134: Resolução de problemas

120 3 Divisibilidade

3.3.4 Equações Diofantinas Lineares

Consideremos a equação

ax+ by = c, (3.26)

onde a, b, c ∈ Z, com a 6= 0 e b 6= 0.

A equação (3.26) é chamada de equação diofantina linear e uma

solução desta é qualquer par de inteiros (x, y) que satisfaçam (3.26).

É conhecido que todos os pontos do plano, com coordenadas (x, y),

que satisfazem a igualdade (3.26) representam, geometricamente, uma

reta. Logo, as soluções de uma equação diofantina linear são os pontos

de coordenadas inteiras do plano cartesiano, que estão dispostos sobre

a reta que esta representa. Por exemplo, os pontos (−1,−2) e (1, 1)

são soluções da equação diofantina 3x− 2y = 1, veja a Figura 3.3.

-3 -2 -1 0 1 2-3

-2

-1

0

1

2

3

x

y

Figura 3.3: A equação da reta ` é 3x− 2y = 1.

Naturalmente nos perguntamos: É sempre possível achar soluções

para uma equação diofantina linear? A resposta é não; o próximo

resultado nos diz quando isto é possível. Além disso, se uma equação

diofantina linear tem uma solução na verdade ela tem uma innidade

de soluções.

Page 135: Resolução de problemas

3.3 Máximo Divisor Comum e Mínimo Múltiplo Comum 121

Proposição 3.35. A equação diofantina linear

ax+ by = c, a, b, c ∈ Z, com a 6= 0 e b 6= 0, (3.27)

tem solução se, e somente se, d | c, onde d = (a, b). Além disso,

se (x0, y0) é uma solução, então o conjunto de soluções de (3.27) é

constituído por todos os pares de inteiros (x, y) da forma:

x = x0 + t bd

e y = y0 − tad , t ∈ Z. (3.28)

Demonstração. Primeiramente suponhamos que (x0, y0) é uma solução

de (3.27), logo ax0 + by0 = c. Usando que d = (a, b) sabemos que

existem inteiros q1 e q2, tais que dq1 = a e dq2 = b. Portanto, se

verica a igualdade

dq1x0 + dq2y0 = d(q1x0 + q2y0) = c,

de onde segue obviamente que d | c.Reciprocamente, suponhamos que d | c e portanto c = qd com q

inteiro. O teorema de Bézout nos garante a existência de dois inteiros,

x0 e y0, tais que ax0 + by0 = d. Multiplicando ambos os lados desta

última igualdade por q temos que

ax0q + by0q = c,

logo o par (x1, y1), com x1 = x0q e y1 = y0q, é solução da equação

diofantina.

Resta provar agora que temos innitas soluções da forma (3.28).

Com efeito, sendo (x, y) uma outra solução qualquer além de (x0, y0),

vale que ax0 + by0 = c = ax+ by, de onde ax0 + by0 = ax+ by. Desta

igualdade obtemos a(x− x0) = b(y0 − y) e dividimos esta última por

d para obtermosa

d(x− x0) =

b

d(y0 − y).

Page 136: Resolução de problemas

122 3 Divisibilidade

Como (ad, bd) = 1, então temos que a

d| (y0 − y) e b

d| (x − x0). Logo,

existe inteiro t tal que

x = x0 + t bd

e y = y0 − tad .

Por outro lado, é fácil vericar que para qualquer inteiro t as expressões

achadas acima para x e y resolvem a equação diofantina.

A seguir damos um exemplo de como proceder para resolver equa-

ções diofantinas.

Exemplo 3.36. Achar todas as soluções inteiras da equação

12x+ 33y = 27.

Solução. Observemos que (12, 33) = 3 e que 3 | 27, logo a equa-

ção tem innitas soluções. Como sabemos, basta achar uma delas

e teremos as restantes. Para achar esta solução particular podemos

trabalhar de duas maneiras, que descrevemos a seguir:

Alternativa 1: reduzimos a equação à forma equivalente

4x+ 11y = 9,

e por tentativa e erro vemos que x0 = 5 e y0 = −1 solucionam a

mesma. Então pela Proposição 3.35 temos que

x = 5 + 11t e y = 4t− 1, t ∈ Z,

esgotam todas as soluções que procuramos.

Alternativa 2: aplicamos o algoritmo de Euclides para achar o

mdc (12, 33), obtendo os seguintes resultados:

33 = 12 · 2 + 9,

12 = 9 · 1 + 3,

9 = 3 · 3 + 0.

Page 137: Resolução de problemas

3.4 Números Primos e Compostos 123

Da segunda e primeira igualdades temos, respectivamente, que

3 = 12− 9 · 1 e 9 = 33− 12 · 2.

Usando estas duas obtemos

3 = 12− (33− 12 · 2) · 1= 12− 33 + 12 · 2= 3 · 12− 1 · 33,

ou seja, achamos x0 = 3 e y0 = −1, garantidos pelo teorema de

Bézout, que validam 3 = 12x0+33y0. Multiplicamos por 9 esta última

igualdade para obter

27 = 12(9x0) + 33(9y0).

Portanto, x0 = 9x0 = 27 e y0 = 9y0 = −9 resolvem, particularmente,

a equação diofantina. Analogamente, como na alternativa anterior,

podemos escrever a solução geral da forma:

x = 27 + 11s e y = 4s− 9, s ∈ Z.

3.4 Números Primos e Compostos

Ao longo da história da Matemática, os números primos foram pro-

tagonistas de célebres problemas que motivaram o desenvolvimento

de teorias e técnicas pelas mentes mais férteis, como Fermat, Euler e

Gauss. Até hoje muitos desses problemas, simples de enunciar, que

envolvem números primos são desaos intelectuais para toda a huma-

nidade.

Page 138: Resolução de problemas

124 3 Divisibilidade

Esta seção será dedicada ao estudo de propriedades básicas dos

números primos. Todo número natural n maior do que 1 tem pelo

menos 2 divisores, claramente 1 e n. Isto motiva a seguinte denição.

Denição 3.37 (Números Primos e Compostos). Um inteiro positivo

n ≥ 2 é dito primo se os únicos divisores que ele tem são 1 e ele

próprio; caso contrário, é dito composto.

Observação 3.38. De modo geral o número 1 não é considerado nem

primo nem composto.

Exemplo 3.39. Os números 2, 3, 5, 7, e 11 são primos e os números

10, 15, 35 e 348 são compostos.

Exemplo 3.40. O número n = 220 − 254 é composto.

Solução. Escrevemos n de outra forma, com o objetivo de facilitar

nosso trabalho. Com efeito, observemos que

n = (210)2 − (252)2 = 10242 − 6252,

logo é composto por ser diferença de quadrados. Além disso,

n = 10242 − 6252,

= (1024− 625)(1024 + 625),

= 399 · 1649,= 3 · 133 · 1649.

(3.29)

Portanto, podemos concluir que 3 | n.

Proposição 3.41. Seja n > 1 um número inteiro. Então

(a) o menor divisor de n diferente de 1 é um número primo;

Page 139: Resolução de problemas

3.4 Números Primos e Compostos 125

(b) se n é composto, o seu menor divisor diferente de 1 não é maior

que√n. Em outras palavras, se n não possui divisores diferentes

de 1, menores ou igual que√n, então n é primo.

Demonstração. Começamos provando (a). Seja p o menor divisor de

n, diferente de 1. Se p fosse composto teria algum divisor q tal que

1 < q < p; mas

q | p e p | n,

o que nos diz que q | n, e isto contradiz a hipótese levantada sobre p.

Para provar (b) denotamos por p o menor divisor de n, diferente

de 1. Portanto, n = pq com q ≥ p. Multiplicando ambos lados da

desigualdade por p obtemos

n = pq ≥ p2,

e consequentemente vale√n ≥ p.

Agora vamos enunciar um dos resultados mais clássicos da Mate-

mática, que garante a existência de innitos números primos. Até

onde se conhece, a demonstração a seguir foi a primeira demonstração

escrita utilizando o método de redução ao absurdo e é devida a Eu-

clides cerca de 300 a.C. Para outras seis provas, incluindo a moderna

prova de Fustenberg, recomendamos os livros [1] e [10].

Teorema 3.42 (Teorema de Euclides). A quantidade de números pri-

mos é innita.

Demonstração. Faremos a prova por redução ao absurdo. Suponha

que existe uma quantidade nita de números primos e denotemos estes

por

p1, p2, p3, . . . , pk.

Page 140: Resolução de problemas

126 3 Divisibilidade

Consideremos o número

n = p1p2p3 · · · pk + 1

e chamemos de q o seu menor divisor primo. Obviamente q não coin-

cide com nenhum dos números pi, 1 ≤ i ≤ k, pois caso contrário,

como ele divide n, teria que dividir 1, o que é impossível. Logo, te-

mos uma contradição à hipótese de termos uma quantidade nita de

primos.

Os números primos também podem ser caracterizados da seguinte

maneira:

Proposição 3.43. Um inteiro positivo p é primo se, e somente se,

satisfaz a seguinte propriedade:

p | ab =⇒ p | a ou p | b (3.30)

onde a, b ∈ Z.

Demonstração. Primeiramente, suponhamos que p é primo e que p - b,logo (p, b) = 1. Então, pelo item (f) da Proposição 3.24 temos que

p | a.Reciprocamente, suponhamos que, a propriedade 3.30 é válida e

além disso vamos supor, pelo absurdo, que p não é primo. Então,

p = d1d2, com 1 < d1 < p, 1 < d2 < p. (3.31)

De (3.30) segue que p | d1 ou p | d2; consequentemente

p ≤ d1, ou p ≤ d2, (3.32)

contradizendo isto o armado em (3.31).

Page 141: Resolução de problemas

3.5 Procurando Primos 127

3.5 Procurando Primos

Os números primos além de belos e desaadores do ponto de vista

matemático, são extremamente importantes para as atividades usuais

de nosso dia a dia. Por exemplo, nenhuma transação bancária ou pela

internet estaria segura sem o uso de números primos muito grandes.

Assim, surge naturalmente a pergunta de como podemos produzi-los

em grandes quantidades. Essa pergunta sempre intrigou os matemá-

ticos e continua sem solução até os dias atuais. Apesar deles serem

abundantes, em quantidade innita de acordo com o Teorema 3.42,

não existe nenhum método razoável de produção de números primos,

mesmo tendo em mãos a alta tecnologia de hoje em dia. Porém, ao

longo do tempo algumas fórmulas e algoritmos se mostraram úteis

para a descoberta de números primos.

3.5.1 O Crivo de Eratóstenes

O crivo de Eratóstenes é um algoritmo que nos permite achar todos

os números primos que são menores ou iguais que um natural N dado.

Segundo a tradição, este método foi criado pelo matemático grego

Eratóstenes (285-194 a.C.).

O método consiste nos seguintes passos: escrevemos os números de

forma ordenada a partir de 2, isto é,

2, 3, 4, 5, 6, 7, 8, 9, 10, 11, 12, 13, 14, 15, 16, 17, . . . , n (3.33)

• Observamos que o primeiro primo que aparece em (3.33) é 2 e

imediatamente apagamos da lista (3.33) todos os múltiplos de

2 maiores que ele, por serem compostos; resta assim a seguinte

Page 142: Resolução de problemas

128 3 Divisibilidade

lista

2, 3, 5, 7, 9, 11, 13, 15, 17 . . .

• O primeiro número não apagado que aparece na lista restante é

3, que também é primo. Imediatamente apagamos da lista todos

os múltiplos de 3 maiores que ele, por serem compostos; resta

agora a lista

2, 3, 5, 7, 11, 13, 17, . . .

• O primeiro número não apagado que aparece na lista que restou

do passo anterior é 5, que também é primo. Imediatamente

apagamos da lista todos os múltiplos de 5 maiores que ele, por

serem compostos.

• Repetimos este processo até que o primeiro número não apagado

da lista em questão seja maior que√n, pois graças à Proposição

3.41-(b) a partir desse momento todos os números restantes são

os primos menores ou iguais que n..

Por exemplo, se n = 40, temos que√40 = 6, 324555. Então,

aplicando o método:

2 3 4 5 6 7 8 9 10

11 12 13 14 15 16 17 18 19 20

21 22 23 24 25 26 27 28 29 30

31 32 33 34 35 36 37 38 39 40

Passo 1: ordenamos os números

2 3 5 7 9

11 13 15 17 19

21 23 25 27 29

31 33 35 37 39

Page 143: Resolução de problemas

3.5 Procurando Primos 129

Passo 2: tiramos os múltiplos de 2

2 3 5 7

11 13 17 19

23 25 29

31 35 37

Passo 3: tiramos os múltiplos de 3

2 3 5 7

11 13 17 19

23 29

31 37

Passo 4: tiramos os múltiplos de 5

Como 72 = 49 > 40, paramos agora.

Observação 3.44. Note que ao começar a apagar os múltiplos de um

número primo p podemos começar a apagar a partir de p2, pois se

supomos que existe um número composto m não apagado menor que

p2, temos que m = p1q1, sendo p1 seu menor divisor primo. Então,

pelo item (b) da Proposição 3.41, p1 <√m < p, logo m deveria ter

sido apagado pois é múltiplo de um primo menor que p.

3.5.2 Primos de Mersenne

Marin Mersenne (1588-1648) foi um monge francês que nasceu na ci-

dade de Maine e foi um dos grandes inuenciadores da Matemática

Page 144: Resolução de problemas

130 3 Divisibilidade

2 3 5 7 11 13 17 19 23 29

31 37 41 43 47 53 59 61 67 71

73 79 83 89 97 101 103 107 109 113

127 131 137 139 149 151 157 163 167 173

179 181 191 193 197 199 211 223 227 229

233 239 241 251 257 263 269 271 277 281

283 293 307 311 313 317 331 337 347 349

353 359 367 373 379 383 389 397 401 409

419 421 431 433 439 443 449 457 461 463

467 479 487 491 499 503 509 521 523 541

Tabela 3.1: Os primeiros 100 números primos

francesa nos séculos XVI e XVII. Apaixonado pelos números, teve en-

tre seus correspondentes Descartes, Fermat, Pascal e Galileu. Entre

suas várias descobertas, ele estudou os números da forma:

Mn = 2n − 1.

Observe que vale o seguinte fato a respeito desses números:

Proposição 3.45. Se Mn é primo, então n é primo.

Demonstração. Provar essa proposição equivale a mostrar que a sua

forma contrarrecíproca vale. Ou seja, que se n é composto, digamos

n = a.b, com a ≥ b > 1, então Mn também é composto. De fato,

usando o Lema 3.14, podemos decompô-lo do seguinte modo:

Ma.b = 2ab − 1 =(2a(b−1) − 2a(b−2) + · · ·+ 2a + 1

)(2b − 1

).

Page 145: Resolução de problemas

3.5 Procurando Primos 131

Porém, não é verdade a recíproca da armação acima. Por exem-

plo, Hudalricus Regius mostrou em 1536 que M11 = 211 − 1 = 2.047

não é primo, já que 2.047 = 23 · 89.Em 1643, Mersenne armou que para

n = 2, 3, 5, 7, 13, 17, 19, 31, 67, 127 e 257,

os valores de Mn são todos primos e para todos os outros valores de n

menores que 257, Mn é composto.

Hoje sabemos que Mersenne errou na sua armação, esquecendo

três valores de n ondeMn é primo: 61, 89 e 107 e incluindoM67 eM257

como números primos. Para mais informações, sugerimos a página web

http://primes.utm.edu/mersenne/index.html.

Finalizamos esta seção, com um critério interessante, devido à ma-

temática francesa Sophie Germain (1776-1831), que nos permite saber

quando um número não é primo.

Proposição 3.46 (Identidade de Sophie Germain). Dados a, b ∈ R,vale a igualdade

a4 + 4b4 = (a2 + 2b2 + 2ab)(a2 + 2b2 − 2ab).

Demonstração. A prova segue das seguintes igualdades:

a4 + 4b4 = a4 + 4a2b2 + 4b4 − 4a2b2

= (a2 + 2b2)2 − 4a2b2

= (a2 + 2b2 + 2ab)(a2 + 2b2 − 2ab).

Como aplicação desta identidade vejamos os seguintes exemplos.

Page 146: Resolução de problemas

132 3 Divisibilidade

Exemplo 3.47. qn = n4 + 4n é composto, para todo n ∈ N.

Solução. O conjunto dos números naturais é particionado em duas

classes disjuntas:o conjunto dos números pares e o conjunto dos nú-

meros ímpares. Estudaremos cada classe por separado. Assim,

• se n é um número par, então n = 2m para algum inteiro positivo

m ≥ 1. Deste modo,

n4 + 4n = (2m)4 + 42m = 16m4 + 24m,

= 2(8m4 + 24m−1

).

Portanto, neste caso, n4 + 4n ≥ 2. Logo, se n > 1 é qualquer

número inteiro positivo par temos que n4+4n não é um número

primo;

• se n é um número ímpar, então n = 2m + 1 para algum inteiro

positivo m ≥ 1. Assim,

n4 + 4n = (2m+ 1)4 + 42m+1 = (2m+ 1)4 + 4 · 42m

= (2m+ 1)4 + 4 · 24m = (2m+ 1)4 + 4 · (2m)4.

Logo, tomando a = 2m + 1 e b = 2m, o resultado é uma con-

sequência direta da identidade de Sophie Germain.

Exemplo 3.48. 520 + 230 é um número composto.

Solução. Escrevemos

520 + 230 = 55·4 + 22 · 228 =(55)4

+ 4 ·(27)4,

de onde podemos usar a Identidade de Sophie Germain com a = 55 e

b = 27 para comprovar que o número 520 + 230 é composto.

Page 147: Resolução de problemas

3.5 Procurando Primos 133

3.5.3 O Teorema Fundamental da Aritmética

Os números primos são as células dos números naturais, no sentido

de que qualquer número natural é produto de números primos. Por

exemplo,

560 = 56 · 10 = 7 · 8 · 5 · 2 = 7 · 2 · 2 · 2 · 5 · 2,

onde cada um dos fatores que aparecem no produto são números pri-

mos. Perguntamo-nos, o que acontece se começamos com uma outra

fatoração inicial de 560, por exemplo, 560 = 28 · 20. Vejamos:

560 = 28 · 20 = 14 · 2 · 10 · 2 = 7 · 2 · 2 · 5 · 2 · 2.

Surpreendentemente chegamos à mesma representação anterior, salvo

a ordem dos fatores.

2222 75

Figura 3.4: O número 560 é composto de 4 células do tipo 2, uma célula

do tipo 7 e uma célula do tipo 5.

O fato observado acima vale para qualquer número natural maior

que 1. Especicamente, temos o seguinte resultado conhecido como

teorema fundamental da aritmética.

Page 148: Resolução de problemas

134 3 Divisibilidade

Teorema 3.49 (Teorema Fundamental da Aritmética). Todo número

natural n maior que 1 pode ser escrito como um produto

n = pα11 p

α22 p

α33 · · · pαm

m , (3.34)

onde m ≥ 1 é um número natural, αi ∈ N e pi é primo para todo

1 ≤ i ≤ m . Além disso, a fatoração em (3.34) é única se exigirmos

que p1 < p2 < · · · < pm.

Demonstração. Seja n um inteiro maior que 1. Denotando por p1 seu

menor divisor primo tem-se que

n = p1β1, 1 ≤ β1 < n.

Se β1 = 1, então N1 = p1 e a fatoração desejada é obtida. Caso

contrário, denotando por p2 o menor divisor primo de β1 tem-se que

n = p1p2β2, 1 ≤ β2 < β1.

Se β2 = 1, então n = p1p2 e novamente chegamos à fatoração desejada.

Caso contrário, denotando por p3 o menor divisor primo de β2 tem-se

que

n = p1p2p3β3, 1 ≤ β3 < β2.

Continuando este processo sucessivamente obtemos então uma sequên-

cia estritamente decrescente de números naturais αn, ou seja,

n > β1 > β2 > β2 > · · · > βn > βn+1 > · · · ≥ 1,

Então, pelo princípio da boa ordem, só pode existir uma quantidade

nita de índices n tais que βn > 1 e consequentemente βn+1 = 1, de

onde segue que

n = p1p2 · · · pn.

Page 149: Resolução de problemas

3.5 Procurando Primos 135

Notemos que na representação acima os pi podem-se repetir, resul-

tando nalmente a representação desejada em (3.34).

Provaremos agora a unicidade de tal fatoração. Com efeito, supo-

nha que existem duas fatorações:

pα11 p

α22 p

α33 · · · pαm

m = n = qβ11 qβ22 q

β33 · · · qβss

Pela Proposição 3.43 temos que cada pi divide algum qj, logo pi =

qj, por serem primos. Portanto, cada pi aparece no lado direito da

igualdade acima, e, um argumento análogo nos dá que cada qj também

aparece no lado esquerdo da igualdade. Então, como os pis e os qjssão diferentes dois a dois e organizados crescentemente, temos m = s

e a igualdade se reduz a

pα11 p

α22 p

α33 · · · pαm

m = pβ11 pβ22 p

β33 · · · pβmm .

Suponhamos agora que α1 seja diferente de β1; sem perda de ge-

neralidade vamos supor que α1 < β1. Portanto,

pα22 · pα3

3 · · · pαmm = pβ1−α1

1 pβ22 pβ33 · · · pβmm ,

e como β1 − α1 > 0 então, pela Proposição 3.43 temos que p1 di-

vide algum pj, com j > 1, o que é impossível. Portanto, α1 = β1.

Similarmente provamos que αi = βi, com i = 1, . . . , n.

Observação 3.50. O teorema fundamental da aritmética foi enun-

ciado precisamente por Gauss (1777-1855). Seus antecessores, Fer-

mat, Euler, Lagrange e Legendre, utilizavam este teorema sem a preo-

cupação de tê-lo enunciado ou demonstrado com precisão. Uma prova

alternativa deste teorema será apresentada no Capítulo 6, usando o

método de indução.

Page 150: Resolução de problemas

136 3 Divisibilidade

Exemplo 3.51. Prove que um número n é par se, e somente se, o

número 2 aparece na fatoração de n em fatores primos.

Solução. Obviamente, se 2 aparece na fatoração em primos de N ,

então N é par. Ora, se n é par temos que n = 2q. Por outro lado q e

n se fatoram, respectivamente, como

q = qα11 qα2

2 · · · qαmm e n = pβ11 p

β22 · · · pβss .

Logo,

2 · qα11 qα2

2 · · · qαmm = pβ11 p

β22 · · · pβss .

Pela unicidade da fatoração, para algum i, com 1 ≤ i ≤ s, o cor-

respondente pi deve ser igual a 2. Portanto, 2 aparece na fatoração de

n.

Exemplo 3.52. Seja A = 1, 2, 3, 4, 5, 6, 7. É possível decompor

o conjunto A em dois subconjuntos disjuntos tais que o produto dos

elementos de um seja igual ao produto dos elementos do outro?

Solução. Mostraremos que é impossível fazer esta decomposição. Com

efeito, suponha que existem tais conjuntos, A1 = p1, p2, . . . , pr e

A2 = q1, q2, . . . , qs. Então

p1p2 · · · pr︸ ︷︷ ︸α

= q1q2 · · · qs︸ ︷︷ ︸β

e além disso, como os conjuntos A1 e A2 são disjuntos, temos que o

número 5 aparece no produto α ou no produto β, mas não em ambos

simultaneamente. Por outro lado, o Teorema 3.49 nos diz que a fatora-

ção em primos de α é igual à fatoração em primos de β, logo o número

5 deveria aparecer tanto no produto α como no produto β, contra-

dizendo isto o fato anterior. Portanto não existe uma decomposição

com as condições exigidas.

Page 151: Resolução de problemas

3.5 Procurando Primos 137

Exemplo 3.53. Encontre todos os números inteiros e positivos n com

a propriedade de que o conjunto

A = n, n+ 1, n+ 2, n+ 3, n+ 4, n+ 5

pode ser particionado em dois subconjuntos tais que o produto dos

elementos de um dos subconjuntos seja igual ao produto dos elementos

do outro.

Demonstração. Digamos que seja possível essa decomposição para al-

gum n e vamos denotar os conjuntos que obtemos com a decomposição

por A1 e A2. Observando a decomposição dos elementos dos subcon-

juntos em fatores primos, temos que todo fator primo de A1 também

deverá pertencer a A2. No conjunto dos seis números só podemos ter

um múltiplo de 7, por isso não podemos tomar n como múltiplo deste

primo. Analogamente para primos maiores que 7. Analisando o primo

5, concluímos que n e n+ 5 são múltiplos de 5, pois se não, cairíamos

na análise anterior. Assim, os números n+ 1, n+ 2, n+ 3 e n+ 4 são

da forma 2α3β. Como entre eles existem dois ímpares, logo teremos

duas potências de 3 cuja diferença é 2, um absurdo. Assim, não existe

n que satisfaz as condições do enunciado.

Finalizamos esta seção com um exemplo que mostra como podemos

combinar os fatos estudados para resolver problemas mais difíceis

Exemplo 3.54. Encontre todos os números que são formados por 4

algarismos da forma aabb e que sejam quadrados perfeitos.

Page 152: Resolução de problemas

138 3 Divisibilidade

Solução. Como o número aabb é um quadrado perfeito, signica que:

n2 =aabb

n2 =103a+ 102a+ 10b+ b =(103 + 102

)· a+ (10 + 1) · b

n2 =1100 · a+ 11 · bn2 =11

(100a+ b

)= 11

(99a+ a+ b

).

Como 11 é primo é fácil ver, usando a Proposição 3.43, que 112 | N2.

Segue-se então que 11 | (99a+a+b). Portanto, 11 | (a+b). Como aabb

tem 4 algarismos, segue-se que a 6= 0; portanto a ∈ 1, 2, 3, . . . , 9 eb ∈ 0, 1, 2, . . . , 9. De onde a + b ≤ 18. Logo, necessariamente

devemos ter a + b = 11. Podemos observar que a 6= 1, pois se a = 1

então b = 10. Analogamente, b 6= 0, 1. Portanto,

a ∈ 2, 3, 4, . . . , 9 e b ∈ 2, 3, 4, . . . , 9.

Como em todo número quadrado perfeito o algarismo das unidades

somente pode acabar em 0, 1, 4, 5, 6 e 9. Segue-se que

b ∈ 4, 5, 6, 9.

Certamente b 6= 5, pois todo número que acaba em 5 quando é elevado

ao quadrado sempre acaba em 25. Assim,

b ∈ 4, 6, 9.

• Se b = 4, então a = 7. Neste caso o número seria 7.744 que é

um quadrado perfeito;

• Se b = 6, então a = 5. Neste caso o número seria 5.566 que não

é um quadrado perfeito;

Page 153: Resolução de problemas

3.6 Exercícios 139

• Se b = 9, então a = 2. Neste caso o número seria 2.299 que não

é um quadrado perfeito.

Finalmente, a única solução possível é aabb = 7.744 = 882.

3.6 Exercícios

1. Encontre o resto que deixa

(a) 2001 · 2002 · 2003 · 2004 + 20052 quando é dividido por 7;

(b) 2100 quando é dividido por 3;

(c) (1237156 + 34)28 quando é dividido por 111.

2. Provar que o número n5 + 4n é divisível por 5 para qualquer

número natural n.

3. Prove que se n é ímpar

(a) n3 − n é divisível por 24;

(b) n2 − 1 é divisível por 8;

(c) n2 + (n+ 2)2 + (n+ 4)2 + 1 é divisível por 12.

4. O número 21093 − 2 é divisível por 10932 ?

5. Prove que (999994)1234567890 − 1 é divisível por 333331.

6. O número N = 42005 + 20054 é primo?

7. Demonstre que o número 1 000 . . . 00︸ ︷︷ ︸2006 zeros

1 é composto.

Page 154: Resolução de problemas

140 3 Divisibilidade

8. Utilizando o fato de que o resto de um quadrado quando dividido

por 4 só pode ser 0 ou 1, dê uma outra solução para o problema

do Exemplo 3.54.

9. Dados três inteiros, x, y, z, tais que x2 + y2 = z2, mostre que x

e y não são ambos ímpares e que xy é múltiplo de 6.

10. Demonstre que o quadrado de um inteiro é da forma 8n ou 8n+1

ou 8n+ 4.

11. Três números primos p, q e r, maiores que 3, formam uma pro-

gressão aritmética, ou seja, q = p+ d e r = p+ 2d. Prove que d

é divisível por 6.

12. Demonstrar que existem innitos números primos da forma 4m+

3 e da forma 6m+ 5, onde m ∈ Z.

13. Encontrar o último dígito dos números

(a) 19892005;

(b) 777777 + 250;

(c) 1 + 22 + 32 + · · ·+ 20052.

14. Prove que a soma dos quadrados de cinco números consecutivos

não é um quadrado perfeito.

15. Prove que 1 00 · · · 00︸ ︷︷ ︸100−zeros

5 00 · · · 00︸ ︷︷ ︸100−zeros

1 não é um cubo perfeito.

16. Seja b um inteiro positivo. Enuncie e prove o critério de divisi-

bilidade por b no sistema de numeração de base b.

17. Prove que os números

Page 155: Resolução de problemas

3.6 Exercícios 141

(a) αn = 1 +1

2+

1

3+ · · ·+ 1

n, com n > 1,

(b) βn =1

3+

1

5+ · · ·+ 1

2n+ 1, com n > 0,

não são inteiros.

18. Considere o polinômio p(n) = amnm + am−1n

m−1 + · · · + a0 de

grau m ≥ 1 com coecientes inteiros e n ∈ N. Prove que p(n) éum número composto para innitos valores de n.

Sugestão: Use o fato de que existe a ∈ N tal que α = |p(a)| > 1

e mostre que α divide a p(αk + a), para todo k ∈ Z.

19. Dizemos que um conjunto An formado por n inteiros positivos

escritos no sistema binário (base 2) é regular se, para qualquer

s inteiro não negativo a quantidade de números de An que con-

templam 2s na representação binária é par. Dizemos que An é

irregular se, pelo menos para algum s, este número é ímpar. De-

monstre que um sistema irregular pode se converter em regular

excluindo-se apenas um único elemento do mesmo, e, um sistema

regular pode se converter em irregular excluindo-se qualquer um

dos seus elementos.

20. Seja n um inteiro positivo. Demonstrar que todos os coecientes

do desenvolvimento do binômio de Newton (a+ b)n são ímpares

se, e somente se, n é da forma 2s − 1.

21. Prove que se (x0, y0) é uma solução da equação diofantina linear

ax − by = 1, então a área do triângulo cujos vértices são (0, 0),

(b, a) e (x0, y0) é 1/2.

Page 156: Resolução de problemas

142 3 Divisibilidade

22. Qual é a menor distância possível entre dois pontos (x1, y1) e

(x2, y2), com coordenadas inteiras, situados sobre a reta denida

pela equação diofantina ax+ by = c?

Page 157: Resolução de problemas

4

O Princípio da Casa dosPombos

Uma vez um matemáti o me falou que o verdadeiro prazer não estáem a har a verdade, mas em pro urar por ela. Leo TolstoyUm interessante instrumento elementar para tratar problemas mate-

máticos relacionados à existência de elementos de conjuntos validando

certas exigências é o chamado princípio de Dirichlet , também conhe-

cido como princípio da casa dos pombos (PCP). Este princípio foi

usado por Dirichlet (1805-1859) para resolver problemas na Teoria

dos Números, entretanto ele possui um grande número de aplicações

em diversos ramos da Matemática como Combinatória e Geometria.

A seguir enunciamos a versão mais simples do PCP.

Proposição 4.1 (PCP Versão Simples). Se distribuímos N + 1

pombos em N casas, então alguma das casas contém dois ou mais

pombos.

143

Page 158: Resolução de problemas

144 4 O Princípio da Casa dos Pombos

· · · · · · · · ·P1 P2 PN

C1 C2 CN

PN+1

Figura 4.1: Em cada casa Cj , 1 ≤ j ≤ N , coloca-se um único pombo,

denotado por Pj . O pombo restante, denotado por PN+1, deve ir para

alguma das casas, juntando-se ao que já se encontrava contido nela

Demonstração. A prova deste princípio é muito fácil e decorre de fa-

zer uma simples contagem dos pombos contidos em todas as casas de-

pois de distribuídos. Com efeito, suponhamos pelo contrário que em

cada casa não existe mais do que um pombo, então contando todos

os pombos contidos nas N casas não teremos mais do que N pombos,

contradizendo isto a hipóteses de termos N + 1 pombos distribuídos

nas N casas (ver Figura 4.1).

Não é difícil detectar quando o princípio pode ser usado, mas a

principal diculdade para aplicá-lo reside em identicar, em cada pro-

blema, quem faz papel de pombos e quem faz papel de casas.

Nas seguintes seções discutiremos vários exemplos de diferentes

naturezas onde o princípio da casa dos pombos é aplicado com sucesso.

Page 159: Resolução de problemas

4.1 Primeiros Exemplos 145

4.1 Primeiros Exemplos

Exemplo 4.2. Numa oresta crescem 1.000 jaqueiras. É conhecido

que uma jaqueira não contém mais do que 600 frutos. Prove que

existem 2 jaqueiras na oresta que têm a mesma quantidade de frutos.

Solução. Temos 1.000 jaqueiras, representando os pombos, e 601 casas

identicadas pelos números 0, 1, 2, 3, . . . , 600. O número k associado

a cada casa signica que nela serão colocadas jaqueiras que têm exa-

tamente k frutos. Como 1000 > 602 = 601 + 1, o PCP nos garante

que existem duas jaqueiras com a mesma quantidade de frutos.

Exemplo 4.3. Em uma reunião há n pessoas. Mostre que existem

duas pessoas que conhecem exatamente o mesmo número de pessoas.

Solução. Os pombos neste caso são as n pessoas. As casas são enume-

radas com os números 0, 1, 2, . . . , n−1, indicando estes que na mesma

serão colocadas pessoas que têm essa quantidade de conhecidos. No-

temos que uma das casas enumeradas com 0 ou n − 1 permanece

desocupada, pois a possibilidade de conhecer 0 e n − 1 pessoas não

acontece simultaneamente. Logo, nas n − 1 casas restantes haverá

uma ocupada por dois ou mais pombos, depois de serem distribuídos.

Portanto, existem no mínimo duas pessoas com o mesmo número de

conhecidos.

Exemplo 4.4. Dados 8 números inteiros mostre que existem dois

deles cuja diferença é divisível por 7.

Solução. Consideramos os 8 números como sendo os pombos e as casas

como sendo os 7 possíveis restos na divisão por 7. Como temos 8 =

7 + 1 números o PCP nos diz que existem dois números dentro dos

Page 160: Resolução de problemas

146 4 O Princípio da Casa dos Pombos

8 dados que têm o mesmo resto quando divididos por 7. Finalmente,

observamos que se dois números deixam o mesmo resto na divisão por

7 então a diferença entre eles é divisível por 7.

Uma forma alternativa e muito útil na qual pode-se apresentar o

princípio da casa dos pombos é a seguinte:

Proposição 4.5 (PCP Versão Alternativa). Se a soma de n nú-

meros naturais é igual S, então existe pelo menos um deles que não

é maior que S/n, assim como existe pelo menos um deles que não é

menor que S/n.

Exemplo 4.6. Numa família formada por 5 pessoas a soma das idades

é de 245 anos. Prove que podem ser selecionados 3 membros da família

cuja soma das idades não é menor que 147.

Solução. Temos um total de(53

)= 5!

3!2!= 10 trios diferentes formados

por membros da família. Além disso, cada pessoa aparece exatamente

em(42

)= 4!

2!2!= 6 trios. Então, denotando por Ej a soma das idades

dos membros de cada trio Tj, j = 1, 2 . . . 10, temos que

E1 + E2 + · · ·+ E10 = 6 · 245 = 1470;

consequentemente existe algum trio Tj∗ tal que Ej∗ ≥ 147010

= 147.

4.2 Uma Versão mais Geral

A seguinte versão mais geral do PCP é bastante útil na resolução de

alguns problemas.

Proposição 4.7 (PCP Versão Geral). Se distribuímos Nk+1 pom-

bos em N casas, então alguma das casas contém pelo menos k + 1

pombos.

Page 161: Resolução de problemas

4.2 Uma Versão mais Geral 147

A prova deste enunciado mais geral é similar à anterior. Com efeito,

suponhamos pelo contrário que em cada casa não existe mais do que

k pombos, então contando todos os pombos contidos nas N casas não

teremos mais do que Nk pombos, contradizendo isto a hipóteses de

termos Nk + 1 pombos distribuídos nas N casas.

Notemos que se k = 1, esta versão mais geral coincide com a versão

mais simples.

Exemplo 4.8. Num colégio com 16 salas são distribuídas canetas nas

cores preta, azul e vermelha para realizar uma prova de concurso. Se

cada sala recebe canetas da mesma cor então prove que existem pelo

menos 6 salas que receberam canetas da mesma cor.

Solução. Fazendo a divisão com resto de 16 por 3 temos que 16 =

3 · 5 + 1. Consideramos as 16 salas como sendo os pombos e as três

cores, preto, azul e vermelho como sendo as casas. Logo, podemos

colocar cada sala em uma das três cores. Assim, o PCP com N = 3

e k = 5 nos dá que existe uma casa com pelo menos 6 pombos, ou seja,

existem no mínimo 6 salas que receberam canetas da mesma cor.

Exemplo 4.9. Uma equipe formada por seis alunos de Matemática é

selecionada para representar o Brasil numa olimpíada internacional.

Mostre que necessariamente existem três deles que se conhecem mu-

tuamente, ou três deles que não se conhecem mutuamente.

Solução. Resolveremos o problema com o auxílio da Figura 4.2. Cada

aluno Aj, com j = 1, 2, . . . , 6, é representado por um dos vértices de

um hexágono regular. Quando dois alunos se conhecem traçamos o

segmento de reta que liga os vértices correspondentes com uma linha

contínua; caso contrário traçamos este segmento com uma linha pon-

tilhada. Logo, usando este esquema, o problema equivale a provar

Page 162: Resolução de problemas

148 4 O Princípio da Casa dos Pombos

que sempre existe um triângulo de lados contínuos ou um triângulo de

lados pontilhados com vértices no conjunto A = A1, A2, . . . , A6.Temos 5 segmentos (pombos) incidindo no vértice A1, cada um

deles contínuo ou pontilhado (estes dois tipos de linhas são conside-

radas como as casas). Como 5 = 2 · 2 + 1, pelo PCP temos que 3

dos 5 segmentos são contínuos ou pontilhados. Suponhamos que 3 são

contínuos (caso contrário o argumento é similar) e denotemos estes

por A1A3, A1A4 e A1A6 (ver Figura 4.2). Se algum dos segmentos

A3A4, A3A6 ou A4A6 for contínuo então este segmento junto aos que

se ligam com A1 formam um triângulo de lados contínuos. Por outro

lado, se nenhum deles for contínuo, então eles formam um triângulo

de lados pontilhados, completando isto a demonstração.

A1

A2A3

A4

A5 A6

Figura 4.2: O triângulo A1A2A5 indica que os alunos A1, A2 e A5 não se

conhecem mutuamente e o triângulo A1A4A6 indica que os alunos A1, A4

e A6 se conhecem mutuamente

Page 163: Resolução de problemas

4.3 Aplicações na Teoria dos Números 149

4.3 Aplicações na Teoria dos Números

Nesta seção apresentamos alguns exemplos de aplicações do PCP na

Teoria dos Números. A primeira delas é:

Exemplo 4.10. Se n e m são números naturais, então o conjunto

A = m+ 1,m+ 2, . . . ,m+ n possui algum divisor de n.

Solução. Temos n números diferentes no conjunto acima. Vamos utili-

zar o método de redução ao absurdo. Se não existisse nenhum múltiplo

de n, quando dividíssemos os números do conjunto A por n, os res-

tos pertenceriam ao conjunto B = 1, 2, . . . , n− 1, que possui n− 1

elementos. Logo, devem existir dois números m + i e m + j, com

1 ≤ i < j ≤ n tais que o resto da divisão de m + i por n é o mesmo

que o resto da divisão de m + j por n. Logo, m + j − (m + i) é um

múltiplo de n, o que implica que n > j − i ≥ 1 é múltiplo de n menor

que n (absurdo!). Logo, deve existir algum múltiplo de n no conjunto

A.

Como consequência desse exemplo, podemos resolver o próximo

problema.

Exemplo 4.11. Demonstrar que todo inteiro tem um múltiplo cuja

representação decimal começa com o bloco de dígitos 1234567890.

Solução. Se m e n são inteiros positivos, pelo exemplo anterior um

dos número m + 1,m + 2, . . . ,m + n é múltiplo de n. Assim, dado n

um inteiro qualquer, escolhe-se m = 1234567890×10n+1. Deste modo,

todos os inteiros m+1,m+2, . . . ,m+ n começam com 1234567890 e

algum deles é múltiplo de n.

Page 164: Resolução de problemas

150 4 O Princípio da Casa dos Pombos

Exemplo 4.12. Dado um número inteiro positivo n, mostre que existe

um múltiplo de n que se escreve com os algarismos 0 e 1 apenas. (Por

exemplo, se n = 3, temos 111 ou 1.101 etc.)

Solução. Consideramos os n+ 1 números

1, 11, 111, 1111, . . . , 111 · · · 1︸ ︷︷ ︸n+1−vezes

(4.1)

como sendo os pombos e n casas enumeradas com os números

0, 1, 2, 3, . . . , n− 1,

ou seja, com os possíveis restos na divisão por n. Similarmente ao

exemplo anterior existem dois números na lista (4.1) que deixam o

mesmo resto na divisão por n e, portanto, a diferença entre o maior e

o menor é múltiplo de n. Obviamente a diferença entre dois números

quaisquer da lista (4.1) resulta em um número formado apenas pelos

algarismos 0 e 1.

Exemplo 4.13. Prove que entre n + 1 elementos escolhidos no con-

junto 1,2,3, . . . , 2n existem dois que são primos relativos.

Solução. A escolha das casas e dos pombos neste exemplo não é tão ób-

via. Os pombos representam os n + 1 números escolhidos do conjunto

1, 2, . . . , 2n e as casas são escolhidas como sendo os n conjuntos:

Cj = 2j − 1, 2j, 1 ≤ j ≤ n.

Logo, pelo PCP, quando distribuímos os n + 1 números nos n conjun-

tos Cj, 1 ≤ j ≤ n, dois deles carão juntos em algum conjunto Cj, ou

seja, estes números serão consecutivos e portanto primos entre si.

Page 165: Resolução de problemas

4.4 Aplicações Geométricas 151

Finalizaremos esta seção com uma outra prova do teorema de

Bachet-Bézout, (veja o Teorema 3.23).

Exemplo 4.14. Seja d = (a, b) o mdc entre os números naturais a e

b. Então, existem x e y números inteiros tais que

ax+ by = d.

Solução. Denotando por m = a/d e n = b/d, podemos supor que a e

b são primos entre si. Realmente, se podemos escrever

mx+ ny = 1

então, substituindo os valores de m e n na equação acima, temos que

ax+ by = d.

Se (a, b) = 1, considere a sequência A = a, 2a, . . . , ba. Armamos

que existe algum número no conjunto A que deixa resto 1 quando

dividido por b. De fato, se isso não ocorresse, teríamos b números em

A deixando no máximo b − 1 restos diferentes quando divididos por

b. Logo, pelo PCP, dois deles, digamos ia e ja com b > j > i ≥ 1,

devem deixar o mesmo resto quando divididos por b. assim, (j − i)aé divisível por b. Como estamos supondo que (a, b) = 1, temos que b

deve dividir j − i > 0. Como b > j − i, temos um absurdo.

Assim, algum dos números em a deixa resto 1 quando divididos

por b. Digamos que esse número seja ax. Logo, ax − 1 é múltiplo de

b, onde ax− 1 = by, o que encerra nossa prova.

4.4 Aplicações Geométricas

Na geometria também encontramos belas aplicações do PCP. Vejamos

os problemas a seguir para constatar isto.

Page 166: Resolução de problemas

152 4 O Princípio da Casa dos Pombos

Exemplo 4.15. Mostre que se tomamos cinco pontos quaisquer sobre

um quadrado de lado 1, então pelo menos dois deles não distam mais

que√2/2.

Solução. Vamos dividir o quadrado em quatro quadradinhos de lado

1/2, como mostra a gura. Logo, pelo PCP pelo menos dois deles de-

1

••

••

vem estar no mesmo quadradinho, uma vez que temos 4 quadradinhos

e 5 pontos. Logo, como a maior distância num quadrado é a diagonal,

o Teorema de Pitágoras nos garante que a distância desses dois pontos

é no máximo√2/2, como queríamos mostrar.

Exemplo 4.16. Na região delimitada por um triângulo equilátero de

lado 4 são marcados 10 pontos no interior deste. Prove que existe ao

menos um par destes pontos cuja distância entre eles não é maior que√3.

Solução. Dividimos o triângulo equilátero de lado 4 em 16 triângulos

equiláteros menores de lado 1, conforme a Figura 4.3.

Agora pintamos os triângulos nas cores branco e cinza de maneira

que dois triângulos vizinhos, isto é, com um lado comum, são pintados

de cores diferentes. Se tivéssemos dois pontos no mesmo triângulo a

distância máxima possível entre eles seria 1 e o problema estaria resol-

vido. Se tivéssemos pontos em triângulos vizinhos, a maior distância

possível entre eles seria√3 e também isto resolveria o problema. Se

não tivéssemos nenhum dos casos anteriores, não seria difícil ver que

Page 167: Resolução de problemas

4.5 Miscelânea 153

A B

C

D

E

• • ••

•••

Figura 4.3: O triângulo DBE é equilátero de lado 3

os 10 pontos deveriam estar situados sobre os 10 triângulos brancos,

contendo cada triângulo exatamente um ponto. Dividindo o triângulo

DBE em 4 triângulos congruentes de lado 3/2 pelo PCP temos que

pelo menos dois dos 6 pontos contidos em DBE estão num destes 4

triângulos, logo a distância entre eles não é maior que 3/2 <√3. Com

isto terminamos nossa prova.

4.5 Miscelânea

Os problemas que apresentamos a seguir usam o PCP combinado com

outras idéias que são muito empregadas nas suas soluções.

Exemplo 4.17. Em cada quadradinho de um tabuleiro 3×3 é colocadoum dos números: -1, 0 ou 1. Prove que entre todas as somas das

linhas, colunas e diagonais do tabuleiro há duas que são iguais. Por

exemplo, no tabuleiro abaixo a soma da segunda linha é 2, que coincide

com a soma da terceira coluna.

Page 168: Resolução de problemas

154 4 O Princípio da Casa dos Pombos

-1 -1 1

1 0 1

0 -1 0

Solução. Seja S = a1 + a2 + a3, onde cada a1, a2 e a3 podem tomar

valores: −1, 0 e 1. Então, temos 7 valores possíveis para S (casas),

que são: −3,−2,−1, 0, 1, 2, 3.O tabuleiro 3×3 tem 3 linhas, 3 colunas e 2 diagonais, portanto, ao

somarmos os elementos de cada uma das linhas, colunas e diagonais,

obteremos 8 números (pombos). Como existem somente 7 valores

possíveis para estes números, pelo PCP pelo menos dois deles devem

ser iguais.

Exemplo 4.18. Dado qualquer conjunto A formado por 10 números

naturais escolhidos entre 1 e 99, inclusos, demonstre que existem dois

subconjuntos disjuntos e não vazios de A tal que a soma dos seus res-

pectivos elementos é igual.

Solução: É conhecido que A tem 210 − 1 = 1.023 subconjuntos não-

vazios diferentes. A soma dos elementos de cada um deles dá uma

quantidade menor do que 1.000, pois o subconjunto com no máximo

10 elementos de maior soma possível é o formado por 90, 91, . . . , 99,

e nesse caso 90+ 91+ · · ·+99 = 945. Agora consideramos os pombos

como sendo os 1.023 subconjuntos distintos de A e as casas como

sendo as somas possíveis dos elementos de cada um dos conjuntos.

Logo, como o número de conjuntos é maior que o número de somas

possíveis, devem existir dois conjuntos B e C de A, de tal modo que

a soma dos elementos de B é igual à soma dos elementos de C. Se B

Page 169: Resolução de problemas

4.5 Miscelânea 155

e C são disjuntos, acabou a prova. Se não, considere D = B −B ∩ Ce E = C − B ∩ C. Logo, os conjuntos D e E são disjuntos e a soma

dos seus elementos é a mesma, pois retiramos de ambos a mesma

quantidade.

Exemplo 4.19. Qual é o maior número de quadradinhos de um ta-

buleiro de 8 × 8 que podem ser pintados de preto, de forma tal que

qualquer arranjo de três quadradinhos, como mostra a Figura 4.4, te-

nha pelo menos um dos quadradinhos não pintado de preto?

Figura 4.4: Tridominós

Solução. Primeiramente, pintamos o tabuleiro de 8×8 como um tabu-

leiro de jogar xadrez, ou seja, 32 quadradinhos pintados de branco e

32 quadradinhos pintados de preto (ver Figura 4.5).

Figura 4.5: Tabuleiro de xadrez

Page 170: Resolução de problemas

156 4 O Princípio da Casa dos Pombos

Notemos que uma vez pintado o tabuleiro desta forma é satisfeita

a exigência do problema, pois nunca temos 2 quadradinhos vizinhos

(quadradinhos com um lado comum) pintados de preto.

Mostraremos agora que se pintamos 33 quadradinhos de preto en-

tão a condição exigida no problema falha. De fato, se dividimos o

tabuleiro em 16 quadrados de 2 × 2 (casas) e pintamos 33 quadra-

dinhos de preto (pombos); então, como 33 = 16 · 2 + 1, pela versão

geral do PCP um dos 16 quadrados de 2× 2 contém 3 quadradinhos

pintados de preto. Portanto, este último contém um arranjo como na

Figura 4.4 completamente pintado de preto.

Resumindo, o número máximo de quadradinhos que podemos pin-

tar de preto é 32.

Exemplo 4.20. Dados sete números reais arbitrários, demonstre que

existem dois deles, digamos x e y, tais que

0 ≤ x− y1+ xy

≤ 1√3

Solução. Primeiramente observamos que a expressão x−y1+xy

nos faz pen-

sar na fórmula

tan(α− β) = tanα− tan β

1 + tanα tan β. (4.2)

Sejam x1, x2, · · · , x7 os sete números selecionados arbitrariamente.

Lembramos que a função tangente é uma bijeção entre o intervalo

(−π2, π2) e os números reais R, logo para cada xi, 1 ≤ i ≤ 7, existe um

αi ∈ (−π2, π2) tal que tan(αi) = xi. Dividimos o intervalo (−π

2, π2) em

seis subintervalos de comprimento π6, como mostra o desenho a seguir.

Pelo PCP dois dos números αi pertencem ao mesmo subintervalo.

Denotemos os mesmos por αi1 e αi2 e suponhamos, sem perda de

Page 171: Resolução de problemas

4.6 Exercícios 157

αi1 αi2

−π2

π2π

6

generalidade, que αi1 ≤ αi2 . Então vale

0 ≤ αi2 − αi1 ≤π

6.

Usando o fato de que a tangente é uma função crescente e a fórmula

(4.2) temos que

tan(0) ≤ tan(αi2 − αi1) ≤ tan(π

6).

Equivalentemente,

0 ≤ xi2 − xi11 + xi2xi1

≤ 1√3.

4.6 Exercícios

1. Seja C um conjunto formado por cinco pontos de coordenadas

inteiras no plano. Prove que o ponto médio de algum dos seg-

mentos com extremos em C tem também coordenadas inteiras.

2. O conjunto dos dígitos 1, 2, ..., 9 é dividido em três grupos.

Prove que o produto dos números de algum dos grupos deve ser

maior que 71.

3. Prove que se N é ímpar então para qualquer bijeção

p : IN → IN

Page 172: Resolução de problemas

158 4 O Princípio da Casa dos Pombos

do conjunto IN = 1, 2, . . . , N o produto P (p) = (1−p(1))(2−p(2)) · · · (N − p(N)) é necessariamente par.

(Dica: O produto de vários fatores é par se, e somente se, um dos

fatores é par.)

4. Dado um conjunto de 25 pontos no plano tais que entre quaisquer

3 deles existe um par com distância menor que 1. Prove que

existe um círculo de raio 1 que contém pelo menos 13 dos 25

pontos dados.

5. Prove que entre quaisquer 5 pontos escolhidos dentro de um

triângulo equilátero de lado 1 sempre existe um par deles cuja

distância não é maior que 0,5.

6. Marquemos todos os centros dos 64 quadradinhos de um ta-

buleiro de xadrez de 8× 8. É possível cortar o tabuleiro com 13

linhas retas que não passem pelos pontos marcados e de forma

tal que cada pedaço de recorte do tabuleiro tenha no máximo

um ponto marcado?

7. Prove que existem duas potências de 3 cuja diferença é divisível

por 1.997.

8. São escolhidos 6 números quaisquer pertencentes ao conjunto

A = 1, 2, 3, . . . , 10.

Prove que existem dois desses seis números cuja soma é ímpar.

9. Seja x um número real arbitrário. Prove que entre os números

x, 2x, 3x, . . . , 101x

Page 173: Resolução de problemas

4.6 Exercícios 159

existe um tal que sua diferença com certo número inteiro é menor

0,011.

10. Mostre que entre nove números que não possuem divisores pri-

mos maiores que cinco, existem dois cujo produto é um qua-

drado.

11. Um disco fechado de raio um contém sete pontos, cujas distân-

cias entre quaisquer dois deles é maior ou igual a um. Prove que

o centro do disco é um destes pontos.

12. Na região delimitada por um retângulo de largura quatro e altura

três são marcados seis pontos. Prove que existe ao menos um

par destes pontos cuja distância entre eles não é maior que√5.

13. Seja a um número irracional. Prove que existem innitos núme-ros racionais r = p/q tais que |a− r| < 1/q2.

14. Suponha que cada ponto do reticulado plano é pintado de vermelho

ou azul. Mostre que existe algum retângulo com vértices no reticulado

e todos da mesma cor.

15. Um certo livreiro vende pelo menos um livro por dia. Sabendo que o

livreiro vendeu 463 livros durante 305 dias consecutivos, mostre que

em algum período de dias consecutivos o livreiro vendeu exatamente

144 livros.

Page 174: Resolução de problemas

Referências Bibliográcas

[1] AIGNER, M. e ZIEGLER, G. (2002). As Provas estão

no Livro. Edgard Blücher.

[2] GARCIA, A. e LEQUAIN, I. (2003). Elementos de Ál-

gebra. Projeto Euclides, IMPA.

[3] LIMA, E. L.; CARVALHO, P. C. P.; WAGNER, E. e

MORGADO, A.C. (2004). A Matemática do Ensino Mé-

dio. Volume 1. Sociedade Brasileira de Matemática.

[4] LIMA, E.L.; CARVALHO, P. C. P.; WAGNER, E. e

MORGADO, A.C. (2004). A Matemática do Ensino Mé-

dio. Volume 2. Sociedade Brasileira de Matemática.

[5] LIMA,E.L.; CARVALHO,P. C. P.; WAGNER,E. e

MORGADO,A.C. (2004). A Matemática do Ensino Mé-

dio. Volume 3. Sociedade Brasileira de Matemática.

[6] LIMA, E.L.; CARVALHO, P. C. P.; WAGNER,E. e

MORGADO, A.C. (2001). Temas e Problemas. Socie-

dade Brasileira de Matemática.

[7] LIMA, E.L. (2001). Álgebra Linear. Sociedade Brasileira

de Matemática.

285

Page 175: Resolução de problemas

286 REFERÊNCIAS BIBLIOGRÁFICAS

[8] MORAIS FILHO, D. C. (2007). Um Convite à Matemá-

tica. EDUFCG.

[9] MORGADO, A.; CARVALHO, J.; CARVALHO, P.;

FERNANDEZ, P. (1991). Análise Combinatória e Pro-

babilidade . Sociedade Brasileira de Matemática.

[10] RIBENBOIM, P. (2001). Números Primos: Mistérios e

Recordes. Sociedade Brasileira de Matemática.

[11] SANTOS, J. P. O. (1993) Introdução à Teoria dos Nú-

meros. IMPA.

[12] SANTOS, J. P. O.; MELLO, M. P. e MURARI, I. T.

C. (2006). Introdução à Análise Combinatória. Editora

Unicamp.

[13] SOARES, M. G. (2005). Cálculo em uma Variável Com-

plexa. Sociedade Brasileira de Matemática.

Page 176: Resolução de problemas

Mestrado Profissional

em Matemática em Rede Nacional

Iniciação à Matemática

Autores:

Krerley Oliveira Adán J. Corcho

Unidade III:

Capítulos V e VI

Page 177: Resolução de problemas

160

Page 178: Resolução de problemas

5

Contagem

Toda vez que puder, onte. Fran is GaltonNeste capítulo discutiremos problemas envolvendo a contagem de

elementos de um conjunto nito dado. Por exemplo, responderemos

perguntas do tipo: de quantos modos podemos distribuir 32 seleções

nacionais de futebol em seis grupos de quatro times cada?

Para solucionar questões como esta, utilizaremos como ferramentas

básicas os princípios aditivo e multiplicativo da contagem. Veremos

também que o uso simultâneo destes princípios será muito útil para

resolver problemas com certos níveis de complexidade. Além disso,

serão abordados os conceitos de permutações, arranjos e combinações,

sendo estes de muita importância por serem os alicerces de um ramo

da matemática denominado combinatória.

Antes de prosseguirmos daremos algumas denições e notações que

serão úteis ao longo de todo o capítulo. Dado um conjunto A deno-

tamos por |A| a quantidade de elementos que este possui. O produto

cartesiano de n conjuntos A1, A2, . . . , An−1 e An é o conjunto denido

161

Page 179: Resolução de problemas

162 5 Contagem

por

A1 × A2 × · · · × An :=

(a1, a2, . . . , an); ai ∈ Ai, i = 1, 2, . . . , n,

onde cada elemento (a1, a2, . . . , an) é chamado de n-upla ordenada.

Denotaremos o conjunto vazio com o símbolo ∅. O leitor que deseja

rever os conceitos básicos da teoria de conjuntos, pode achá-los muito

bem expostos em [3].

5.1 Princípio Aditivo da Contagem

O princípio aditivo da contagem garante que dados dois conjuntos

nitos que não têm elemento em comum, o número de elementos da

união é exatamente a soma do número de elementos de cada um, ou

seja, se A1 e A2 são disjuntos (isto é, A1 ∩ A2 = ∅), então

|A1 ∪ A2| = |A1|+ |A2|.

Apesar de sua simplicidade, muitos problemas podem ser resolvi-

dos utilizando esse simples princípio. A seguir enunciamos uma ex-

tensão deste princípio para um número nito qualquer de conjuntos.

Princípio Aditivo da contagem: Dados os conjuntos nitos A1,

A2, . . . , An dois a dois disjuntos (isto é, Ai ∩ Aj = ∅ ,∀ i 6= j),

temos que

|A1 ∪ A2 ∪ · · · ∪ An| = |A1|+ |A2|+ · · ·+ |An|.

Exemplo 5.1. Em Maceió entraram em cartaz 4 lmes distintos e

2 peças de teatro. Se Pedro Vítor só tem dinheiro para assistir a um

lme ou a uma peça de teatro, diga quantos são os possíveis programas

de Pedro Vítor.

Page 180: Resolução de problemas

5.1 Princípio Aditivo da Contagem 163

Solução. Denotemos por f1, f2, f3 e f4 os quatro lmes que estão em

cartaz e por t1 e t2 as duas peças de teatro. Agora, representemos pelo

par (i, j), com 0 ≤ i ≤ 4 e 0 ≤ j ≤ 2, o programa que consiste em as-

sistir ao lme fi e à peça tj (caso i = 0 ou j = 0 isso signica que não

será assistido a nenhum lme ou a nenhuma peça, respectivamente).

Pelas limitações econômicas do Pedro Vítor temos que ele só pode

escolher um programa dentro dos seguintes conjuntos disjuntos:

A1 =

(1, 0), (2, 0), (3, 0), (4, 0)

e A2 =

(0, 1), (0, 2).

Logo, no total são |A1 ∪ A2| = |A1| + |A2| = 6 programas distintos,

entre os quais Pedro Vítor terá que escolher um.

Exemplo 5.2. Numa reunião havia um certo número de pessoas e

todos os presentes apertaram as mãos entre si. Sabendo-se que ao todo

foram feitos 66 cumprimentos, calcule o número de pessoas presentes

à reunião.

Solução. Vamos enumerar as pessoas com os números do conjunto

P = 1, 2, . . . , n. A cada aperto de mão associaremos um par (i, j),

signicando que a pessoa i apertou a mão da pessoa j. Assim, os

apertos de mão envolvendo a pessoa 1 foram:

A1 = (1, 2), (1, 3), . . . , (1, n).Do mesmo modo, denimos os apertos de mão envolvendo a pessoa 2

que não envolvem a pessoa 1, como:

A2 = (2, 3), (2, 4), . . . , (2, n).

Note que o aperto (2, 1) é o mesmo que o aperto (1, 2), já que se 1

aperta a mão de 2, então 2 aperta a mão de 1. Analogamente,

Ai = (i, i+ 1), (i, i+ 2), . . . , (i, n), para 1 ≤ i ≤ n.

Page 181: Resolução de problemas

164 5 Contagem

Note que Ai ∩ Aj = ∅ para i 6= j. Observe também que todos os

apertos aparecem em um dos conjuntos Ai. Assim, A1 ∪ · · · ∪ Ancontém todos os apertos de mão. Logo, pelo princípio aditivo:

|A1 ∪ A2 ∪ · · · ∪ An| = |A1|+ |A2|+ . . . |An|= (n− 1) + (n− 2) + · · ·+ 2 + 1

=(n− 1)n

2= 66.

Resolvendo em n, temos que n = 12.

Vimos que o princípio aditivo nos fornece o número de elementos

de qualquer união de conjuntos dois a dois disjuntos. Discutiremos

agora uma extensão do princípio para qualquer união de conjuntos,

não necessariamente dois a dois disjuntos.

Proposição 5.3. Sejam A1 e A2 dois conjuntos nitos quaisquer.

Então,

|A1 ∪ A2| = |A1|+ |A2| − |A1 ∩ A2|.

Demonstração. Observe que

A1 ∪ A2 = (A1 − A2) ∪ A2

onde a união é dois a dois disjunta. Pelo princípio aditivo, temos que

|A1 ∪ A2| = |A1 − A2|+ |A2|. (5.1)

Analogamente, aplicando novamente este princípio, temos que

|A1| = |A1 − A2|+ |A1 ∩ A2|; (5.2)

A proposição segue imediatamente combinando as igualdades (5.1) e

(5.2).

Page 182: Resolução de problemas

5.1 Princípio Aditivo da Contagem 165

Para chegar a uma expressão análoga à do princípio aditivo, vamos

fazer mais um caso, considerando agora três conjuntos.

Corolário 5.4. Sejam A1, A2 e A3 três conjuntos nitos quaisquer.

Então,

|A1 ∪ A2 ∪ A3| =|A1|+ |A2|+ |A3|−(|A1 ∩ A2|+ |A1 ∩ A3|+ |A2 ∩ A3|

)

+ |A1 ∩ A2 ∩ A3|.

Demonstração. Pela Proposição 5.3 temos que,

|A1 ∪ (A2 ∪ A3)| = |A1|+ |A2 ∪ A3| − |A1 ∩ (A2 ∪ A3)|,

de onde,

|A1 ∪ A2 ∪ A3| = |A1|+ |A2 ∪ A3| − |(A1 ∩ A2) ∪ (A1 ∩ A3)|.

Novamente, pela Proposição 5.3 temos que,

|A1∪A2∪A3| = |A1|+ |A2|+ |A3|− |A2∩A3|− |(A1∩A2)∪ (A1∩A3)|.

Aplicando mais uma vez a Proposição 5.3 temos que,

|(A1∩A2)∪ (A1∩A3)| = |A1∩A2|+ |A1∩A3|− |(A1∩A2)∩ (A1∩A3).

Combinando as duas últimas igualdades obtemos

|A1 ∪ A2 ∪ A3| =|A1|+ |A2|+ |A3|−(|A1 ∩ A2|+ |A1 ∩ A3|+ |A2 ∩ A3|

)

+ |A1 ∩ A2 ∩ A3| ,

como desejávamos.

Page 183: Resolução de problemas

166 5 Contagem

Para facilitar nossa escrita, vamos denotar por A1A2 . . . Ak o con-

junto A1 ∩A2 ∩ · · · ∩Ak. Assim, outra forma de enunciar o Corolário

5.4 é a seguinte:∣∣∣∣∣

3⋃

i=1

Ai

∣∣∣∣∣ =3∑

i=1

|Ai| −∑

1≤i1<i2≤3|Ai1Ai2 |+

1≤i1<i2<i3≤3|Ai1Ai2Ai3|.

De forma geral, dados os conjuntos nitos A1, A2, . . . , An, as ex-

pressões anteriores nos levam a denir os números:

S1 =n∑

i=1

|Ai|

S2 =∑

1≤i1<i2≤n|Ai1Ai2|,

...

Sk =∑

1≤i1<i2<···<ik≤n|Ai1Ai2 . . . Aik |,

...

Sn = |A1A2 . . . An|.

Assim, a versão mais geral do princípio aditivo, também conhecida

como princípio de inclusão e exclusão, é:

Princípio Aditivo - Versão Geral: Sejam A1, A2 . . . , An

conjuntos nitos quaisquer. Então,∣∣∣∣∣n⋃

i=1

Ai

∣∣∣∣∣ = S1 − S2 + S3 − S4 + · · ·+ (−1)n−1Sn.

Não iremos provar essa versão, mas o leitor pode (e deve!) mostrá-la

como exercício, repetindo os argumentos anteriores.

Page 184: Resolução de problemas

5.1 Princípio Aditivo da Contagem 167

Exemplo 5.5. No Colégio Fantástico foram entrevistados 78 estudan-

tes. Destes, 32 estavam fazendo um curso de francês; 40 um curso de

física; 30 um curso de matemática; 23 um curso de história; 19 francês

e física; 13 francês e matemática; 15 física e matemática; 2 francês e

história; 15 física e história; 14 matemática e história; 8 francês, fí-

sica e matemática; 8 francês, física e história; 2 francês, matemática

e história; 6 física, matemática e história e 2 estavam fazendo todos

os quatro cursos. Quantos estudantes estavam fazendo pelo menos 1

curso nas 4 áreas mencionadas?

Solução. Denotemos por A1, A2, A3, e A4 os conjuntos dos estudan-

tes que fazem francês, física, matemática e história, respectivamente.

Observemos que as igualdades

|A1| = 32,

|A2| = 40,

|A3| = 30,

|A4| = 23,

nos dão que S1 =4∑

i=1

|Ai| = 125; as igualdades

|A1A2| = 19,

|A1A3| = 13,

|A1A4| = 2,

|A2A3| = 15,

|A2A4| = 15,

|A3A4| = 14,

Page 185: Resolução de problemas

168 5 Contagem

nos dão que S2 =∑

1≤i1<i2≤4|Ai1Ai2 | = 78; as igualdades

|A1A2A3| = 8,

|A1A2A4| = 8,

|A1A3A4| = 2,

|A2A3A4| = 6,

nos dão que S3 =∑

1≤i1<i2<i3≤4|Ai1Ai2Ai3| = 24; assim como que S4 =

|A1A2A3A4| = 2.

Segue-se então, do princípio aditivo, que

∣∣∣∣∣4⋃

i=1

Ai

∣∣∣∣∣ = 125−78 + 24−

2 = 69.

Denição 5.6. Denimos o complementar do conjunto A em relação

ao conjunto U como sendo um subconjunto de U dado por

Ac =x ∈ U ; x /∈ A

.

U

A

Figura 5.1: A área branca corresponde a Ac e o conjunto U é representado

por todo o retângulo

Page 186: Resolução de problemas

5.1 Princípio Aditivo da Contagem 169

Neste caso é fácil vericar que os conjuntos A e Ac são disjuntos e

que U = A ∪ Ac. Segue-se do princípio aditivo que |U| = |A| + |Ac|;portanto,

|Ac| = |U| − |A|.Analogamente, dados dois conjuntos A1 ⊂ U e A2 ⊂ U , temos que

A1∪A2 e (A1∪A2)c são disjuntos e, aliás, U = (A1∪A2)∪(A1∪A2)

c.

Novamente, pelo princípio aditivo, vale que

|U| = |A1 ∪ A2|+ |(A1 ∪ A2)c|;

e consequentemente temos que

|(A1 ∪ A2)c| = |U| − (|A1|+ |A2|) + |A1A2|.

Similarmente, dados três conjuntos A1 ⊂ U , A2 ⊂ U e A3 ⊂ Upodemos demonstrar que

|(A1 ∪ A2 ∪ A3)c| = |U| − (|A1|+ |A2|+ |A3|)

+ (|A1A2|+ |A1A3|+ |A2A3|)− |A1A2A3|.

Então, usando a notação S0 = |U|, temos a seguinte proposição:

Proposição 5.7. Para toda família de subconjuntos Ai ⊂ U , i =

1, 2, . . . , n, vale a relação:∣∣∣∣∣

(n⋃

i=1

Ai

)c∣∣∣∣∣ = S0 −(S1 − S2 + S3 − S4 − · · ·+ (−1)n−1Sn

)

= S0 − S1 + S2 − S3 + S4 − · · ·+ (−1)nSn,

ou resumidamente,∣∣∣∣∣

(n⋃

i=1

Ai

)c∣∣∣∣∣ = |Ac1Ac2 · · ·Acn| =n∑

j=0

(−1)jSj.

Page 187: Resolução de problemas

170 5 Contagem

Observação 5.8. Observemos que na última relação da proposição

usamos a conhecida Lei de DeMorgan: o complementar da união de

uma família nita de conjuntos, em relação a um conjunto U , é a

intersecção dos complementares de cada um deles.

5.2 Princípio Multiplicativo de Contagem

Começamos esta seção discutindo um problema relacionado com o

apaixonante jogo de xadrez. Ele consiste no seguinte: queremos saber

de quantas maneiras diferentes podemos colocar duas torres num tabu-

leiro de xadrez de forma tal que nenhuma ataque a outra. Uma situa-

ção como a que procuramos é mostrada na Figura 5.2, pois lembramos

que torres só se movimentam na direção horizontal ou na direção verti-

cal do tabuleiro. Antes de prosseguir deixamos claro o seguinte: se na

Figura 5.2 trocamos a posição da torre a com a torre b consideraremos

isto como uma situação diferente.

a

b

Figura 5.2: Torres que não se atacam

Notemos o seguinte: uma vez que coloquemos uma das torres numa

Page 188: Resolução de problemas

5.2 Princípio Multiplicativo de Contagem 171

casa do tabuleiro não podemos colocar a segunda torre na mesma

linha ou coluna em que esta se encontra, pois ela seria ameaçada.

Como cada linha e cada coluna contém 8 casas do tabuleiro, sendo

uma delas comum a ambas, então temos 15 posições proibidas para

colocar a segunda torre, ou seja, ela só pode ser colocada em 64−15 =

49 posições diferentes. Resumindo, por cada uma das 64 possíveis

posições para a torre a temos 49 possibilidades diferentes para colocar

a torre b, totalizando 64·49 = 3.136 formas diferentes de colocar ambas

as torres no tabuleiro sem que elas se ataquem.

O exemplo acima traz a essência do que é chamado princípio mul-

tiplicativo da contagem: se um evento A1 pode ocorrer de m maneiras

distintas e, se para cada uma dessas m maneiras possíveis de A1 ocor-

rer, um outro evento A2 pode ocorrer de n maneiras distintas, então o

número de maneiras de ocorrerem sucessivamente os eventos A1 e A2

é m · n.Na linguagem matemática: relembramos que dados dois conjuntos

A1 e A2, podemos construir um par ordenado (a1, a2) tomando um

elemento a1 ∈ A1, denominado o primeiro elemento do par, e um

elemento a2 ∈ A2, denominado o segundo elemento do par. O conjunto

A1×A2 é constituido por todos os pares ordenados construídos dessa

forma. Assim sendo, a versão mais simples do princípio multiplicativo

nos garante que

|A1 × A2| = |A1| |A2|.

Uma extensão deste princípio para um número nito qualquer de

conjuntos é a seguinte:

princípio multiplicativo da contagem: Dados os conjuntos

Page 189: Resolução de problemas

172 5 Contagem

nitos A1, A2, . . . , An temos que

|A1 × A2 × · · · × An| = |A1| · |A2| · · · |An|.

Note que neste princípio, não é necessária nenhuma hipótese adi-

cional sobre os conjuntos Ai. Vamos agora dar alguns exemplos de

como aplicar esse princípio.

Exemplo 5.9. Em Maceió entraram em cartaz 4 lmes distintos e

2 peças de teatro. Se agora o Pedro Vítor tem dinheiro para assistir

exatamente a um lme e a uma peça de teatro, diga quantos são os

possíveis programas que Pedro Vítor pode fazer.

Solução. Denotemos por f1, f2, f3 e f4 os quatro lmes que estão em

cartaz e por t1 e t2 as duas peças de teatro. Denamos os conjuntos

A1 = f1, f2, f3, f4 e A2 = t1, t2.

Neste caso, as condições econômicas do Pedro Vítor permitem que

ele escolha um elemento do conjunto A1 e outro elemento do conjunto

A2. Este tipo de escolha representa-se pelo conjunto

A1 × A2 =

(fi, tj); 1 ≤ i ≤ 4 e 1 ≤ j ≤ 2,

onde cada par (fi, tj) representa o programa que consiste em assistir

ao lme fi e à peça tj. Logo, no total são |A1 × A2| = |A1| · |A2| = 8

programas distintos.

Exemplo 5.10. Se numa loja de doces existem 9 tipos distintos de

balas e 5 tipos distintos de chiclete, diga quantas escolhas podemos

fazer para comprar somente uma bala e um chiclete.

Page 190: Resolução de problemas

5.2 Princípio Multiplicativo de Contagem 173

Solução. Denotemos por b1, b2, b3, b4, b5, b6, b7, b8 e b9 os nove tipos

distintos de balas e por c1, c2, c3, c4 e c5 os cinco tipos distintos de

chicletes. Denamos os conjuntos

B = b1, b2, b3, b4, b5, b6, b7, b8, b9 e C = c1, c2, c3, c4, c5.

Como precisamos comprar simultaneamente um elemento do conjunto

B e um elemento do conjunto C, então o conjunto B × C me dá o

conjunto de todas as escolhas possíveis. Logo, o número de escolhas

possíveis para comprar simultaneamente um tipo de bala e um tipo

de chiclete é |B × C| = 9 · 5 = 45.

Exemplo 5.11. De quantas maneiras 2 pessoas podem estacionar seus

carros numa garagem com 10 vagas?

Solução. Observando que a primeira pessoa pode estacionar seu carro

de 10 formas distintas e que a segunda pessoa pode estacionar seu

carro de 9 formas distintas, temos pelo princípio multiplicativo que

existem 9 · 10 = 90 formas possíveis nas quais duas pessoas podem

estacionar seus carros numa garagem com 10 vagas.

Exemplo 5.12. Dado o número 720, diga

(a) quantos divisores inteiros e positivos ele possui;

(b) entre seus divisores inteiros e positivos, quantos são pares;

(c) entre seus divisores inteiros e positivos, quantos são ímpares;

(d) dos divisores acima, quantos são quadrados perfeitos.

Page 191: Resolução de problemas

174 5 Contagem

Solução. Pelo teorema fundamental da aritmética, todo número in-

teiro positivo é primo ou produto de primos. Observe que a decom-

posição de 720 em fatores primos vem dada por:

720 = 24 · 32 · 51. (5.3)

Agora denamos os seguintes conjuntos:

A =todos os divisores de 720 que são da forma 2k, onde k ∈ Z+,B =todos os divisores de 720 que são da forma 3m, onde m ∈ Z+,C =todos os divisores de 720 que são da forma 5n, onde n ∈ Z+.

Observemos que 0 ≤ k ≤ 4, pois se k > 4 então pelo menos a potência

25 deveria estar presente em (5.3); como isto não acontece segue-se

que 0 ≤ k ≤ 4, de modo que

A =

20, 21, 22, 23, 24,

seguindo o mesmo raciocínio, podemos demonstrar que 0 ≤ m ≤ 2 e

que 0 ≤ n ≤ 1. Assim,

B =

30, 31, 32

e C =

50, 51.

(a) O conjunto de todos os possíveis divisores de 720 pode ser identi-

cado com o conjunto A×B×C. De onde o número de divisores

inteiros e positivos de 720 é |A×B×C|. Porém, o princípio mul-

tiplicativo nos garante que |A×B×C| = |A| · |B| · |C|. Portanto,o número de divisores inteiros e positivos de 720 é 5×3×2 = 30,

pois |A| = 5, |B| = 3 e |C| = 2.

(b) Para obter o conjunto de todos os divisores pares de 720 deve-

mos remover o elemento 20 do conjunto A. Assim, o conjunto de

Page 192: Resolução de problemas

5.2 Princípio Multiplicativo de Contagem 175

todos os divisores pares e positivos de 720 vem dado pelo con-

junto A− 20 ×B ×C. O princípio multiplicativo nos garante

que∣∣(A − 20

)× B × C

∣∣ =∣∣A − 20

∣∣ · |B| · |C|. Portanto, onúmero de divisores pares e positivos de 720 é 4 × 3 × 2 = 24,

pois∣∣A− 20

∣∣ = 4, |B| = 3 e |C| = 2.

(c) Para obter o conjunto de todos os divisores ímpares de 720 deve-

mos remover os elementos 21, 22, 23 e 24 do conjunto A. Assim,

o conjunto de todos os divisores ímpares e positivos de 720 vem

dado pelo conjunto(A− 21, 22, 23, 24

)×B × C.

O princípio multiplicativo nos garante que∣∣(A−21, 22, 23, 24

)×B ×C

∣∣ =∣∣A−21, 22, 23, 24

∣∣ · |B| · |C|.

Portanto, o número de divisores ímpares e positivos de 720 é

1× 3× 2 = 6; pois∣∣A− 21, 22, 23, 24

∣∣ = 1, |B| = 3 e |C| = 2.

(d) Para obter o conjunto de todos os divisores de 720 que são qua-

drados perfeitos devemos car com as potências pares nos con-

juntos A, B e C, respectivamente. Portanto, devemos remover

os elementos 21, 23 do conjunto A. Também devemos remover o

elemento 31 do conjunto B. Finalmente do conjunto C devemos

remover o elemento 51. Logo, o conjunto de todos os divisores

quadrados perfeitos e positivos de 720 vem dado pelo conjunto

D :=(A− 21, 23

)×(B − 31

)×(C − 51

).

O princípio multiplicativo nos garante que∣∣D∣∣ =

∣∣A− 21, 23∣∣ ·∣∣B − 31

∣∣ ·∣∣C − 51

∣∣.

Page 193: Resolução de problemas

176 5 Contagem

Portanto, o número de divisores quadrados perfeitos e positivos

de 720 é 3 · 2 · 1 = 6; pois∣∣A − 21, 23

∣∣ = 3,∣∣B − 31

∣∣ = 2

e∣∣C − 31

∣∣ = 1. Observe que 1, 4, 9, 16, 36, 144 é o conjunto

dos divisores de 720 que são quadrados perfeitos.

Exemplo 5.13. Se um número natural n se fatora como

n = pk11 · pk22 · · · pkrr , (5.4)

onde os pi são números primos distintos e cada ki ∈ Z+, então o

número de divisores positivos de n, denotado por d(n) é

d(n) = (k1 + 1)(k2 + 1) . . . (kr + 1).

Solução. Dena o conjunto

A1 =todos os divisores de n que são da forma pm11 , onde m ∈ Z+,

e em geral, dena

Ai = todos os divisores de n que são da forma pmii , onde t ∈ Z+.

Observemos que mi ≤ pi, pois se mi > pi, então pelo menos a potência

pki+1i deveria estar presente em (5.4);como isto não acontece segue-se

que mi ≤ pi, de modo que

Ai =p0i , p

1i , p

2i , . . . , p

kii

, para i = 1, 2, 3, . . . , ki.

É imediato ver que∣∣Ai∣∣ = ki + 1.

Page 194: Resolução de problemas

5.2 Princípio Multiplicativo de Contagem 177

O conjunto de todos os possíveis divisores de n vem dado pelo

conjunto A1 × A2 × · · · × Ar, de onde se conclui que o número de

divisores inteiros e positivos de n é

d(n) = |A1 × A2 × · · · × Ar| = |A1| · |A2| · · · |Ar|,onde na última igualdade usamos o princípio multiplicativo. Portanto,

o número de divisores inteiros e positivos de n é

d(n) = (k1 + 1)(k2 + 1) · · · (kr + 1).

Exemplo 5.14. De quantas maneiras podemos escolher dois inteiros

de 1 a 20 de forma que a soma seja ímpar?

Solução. Observemos que

• a soma de dois números inteiros pares é um número par. Com

efeito, para quaisquer a, b ∈ Z temos que 2a+ 2b = 2(a+ b);

• a soma de dois números inteiros ímpares é um número par. Com

efeito, para quaisquer a, b ∈ Z temos que (2a + 1) + (2b + 1) =

2(a+ b+ 1);

• a soma de um número inteiro par com qualquer outro inteiro

ímpar sempre é um inteiro ímpar. Com efeito, para quaisquer

a, b ∈ Z temos que 2a+ (2b+ 1) = 2(a+ b) + 1.

Isto nos sugere denir os conjuntos

P = 2, 4, 6, 8, 10, 12, 14, 16, 18, 20,I = 1, 3, 5, 7, 9, 11, 13, 15, 17, 19,

onde P × I são todas as formas possíveis de somar um número inteiro

par com outro ímpar. O princípio multiplicativo nos garante que nossa

resposta é |P × I| = |P | · |I| = 100, pois |P | = |I| = 10.

Page 195: Resolução de problemas

178 5 Contagem

5.3 Uso Simultâneo dos Princípios Aditivo

e Multiplicativo

Aproveitamos esta seção para apresentar problemas um pouco mais

difíceis que os tratados nas seções anteriores. Nestes problemas, pre-

cisaremos empregar simultaneamente o Princípio Aditivo e o princípio

multiplicativo. Vamos ao primeiro deles:

Exemplo 5.15. Sabemos que no início da premiação da 1a fase da

Olimpíada Alagoana de Matemática existem 10 livros diferentes de

Álgebra, 7 livros diferentes de combinatória e 5 livros diferentes de

geometria para homenagear os vencedores. Danielle é a primeira a

pegar o prêmio que consiste em 2 livros, com a condição de que estes

não podem ser da mesma matéria. Diga quantas escolhas Danielle

pode fazer para pegar seu prêmio.

Solução. Denotemos por

A = a1, . . . , a10, C = c1, . . . , c7 e G = g1, . . . , g5,

os conjuntos de livros de álgebra, combinatória e geometria, respecti-

vamente. Observemos que |A| = 10, |C| = 7 e |G| = 5 e Danielle tem

as seguintes possibilidades de escolha:

• escolher um livro de A e um livro de C. Neste caso, Danielle tem

|A × C| = |A| · |C| = 70 escolhas possíveis (devido ao princípio

multiplicativo).

• escolher um livro de A e um livro de G. Neste caso, Danielle tem|A × G| = |A| · |G| = 50 escolhas possíveis (devido ao princípio

multiplicativo) ou

Page 196: Resolução de problemas

5.3 Uso Simultâneo dos Princípios Aditivo e Multiplicativo 179

• escolher um livro de C e um livro de G. Neste caso, Danielle tem|C × G| = |C| · |G| = 35 escolhas possíveis (devido ao princípio

multiplicativo).

Agora o Princípio Aditivo nos garante que o número total de escolhas

que Danielle pode fazer é 70 + 50 + 35 = 155.

Exemplo 5.16. Há 18 moças e 12 rapazes, onde 5 deles são irmãos

(3 moças e 2 rapazes) e os restantes não possuem parentesco. Diga

quantos casamentos são possíveis naquela turma (sabendo que irmãos

não se casam).

Solução. Observemos que 15, entre as 18 moças, não têm parentesco

nenhum com os 12 rapazes, logo, pelo princípio multiplicativo temos

que é possível efetuar 15 · 12 = 180 casamentos diferentes entre eles.

Por outro lado, as 3 moças restantes podem efetuar casamento com 10

dos 12 rapazes, pois 2 deles são seus irmãos. Novamente, pelo princípio

multiplicativo é possível realizar 3·10 = 30 casamentos diferentes neste

caso. Finalmente, o Princípio Aditivo nos dá que podem ser realizados

um total de 180 + 30 = 210 casamentos.

Exemplo 5.17. Quantas palavras de 5 caracteres podem ser formadas

com as letras α, β e γ de modo que em cada palavra não falte nenhuma

dessas letras?

Solução. Denamos os seguintes conjuntos,

U =palavras de 5 caracteres só com as letras α, β e γ;Aα =palavras que estão em U e onde não aparece a letra α;Aβ =palavras que estão em U e onde não aparece a letra β;Aγ =palavras que estão em U e onde não aparece a letra γ.

Page 197: Resolução de problemas

180 5 Contagem

Por exemplo,

• a palavra γγγγγ ∈ Aα ∩ Aβ;

• a palavra γααγα ∈ Aβ;

• a palavra βαβββ ∈ Aγ.

Primeiramente, notemos que cada caracter de U pode ser escolhido

de 3 formas distintas. Segue-se então do Princípio Multiplicativo que

existem 35 formas de escrever uma palavra de 5 caracteres usando um

alfabeto de 3 letras, isto é,

S0 = |U| = 35 = 243.

Calculemos agora |Aα|, isto é, o número de palavras onde não apa-

rece a letra α. Para isto, observemos que cada caractere em Aα pode

ser escolhido de 2 formas. Logo, o princípio multiplicativo nos garante

que existem 25 palavras em Aα, ou seja, |Aα| = 25. Analogamente,

podemos mostrar que |Aβ| = |Aγ| = 25. Portanto,

S1 = |Aα|+ |Aβ|+ |Aγ| = 25 + 25 + 25 = 96.

Prosseguimos com o cálculo de |AαAβ|, isto é, do número de pala-

vras onde não aparecem as letras α e β; portanto, cada caractere em

AαAβ pode ser escolhido de 1 forma. Logo, o princípio multiplicativo

nos garante que existe 15 = 1 palavra em AαAβ, ou seja, |AαAβ| = 1.

Similarmente, podemos mostrar que |AαAγ| = |AβAγ| = 1. Portanto,

S2 = |Aα|+ |Aβ|+ |Aγ| = 3.

Por m, achamos |AαAβAγ|, que nos dá o número de palavras onde

não aparecem as letras α, β e γ; mas cada palavra em AαAβAγ tem

Page 198: Resolução de problemas

5.4 Permutações Simples 181

que usar pelo menos um dos caracteres proibidos. Logo,

S3 = |AαAβAγ| = 0.

Finalmente, observamos que o conjunto das palavras de 5 caracte-

res que podem ser formadas com as letras α, β e γ de modo que em

cada palavra não falte nenhuma dessas letras é exatamente o conjunto

AcαAcβA

cγ. Usando a Proposição 5.7, temos:

|AcαAcβAcγ| =S0 − S1 + S2 − S3

=243− 96 + 3− 0

=150.

5.4 Permutações Simples

Denimos o fatorial n! de um inteiro positivo n

n! = n · (n− 1) · (n− 2) · · · 2 · 1

se n > 0 e 0! = 1, por convenção. Observe que o fatorial cresce muito

rapidamente quando n cresce. Por exemplo, para os 10 primeiros

valores de n

1!=1 2!=2 3!=6 4!=24 5!=120

6!=720 7!=5.040 8!=40.320 9!=362.880 10!=3.628.800

Denição 5.18. Uma permutação simples de n objetos distintos é

qualquer agrupamento ordenado desses n objetos. Denotaremos por

Pn o número de todas as permutações simples de n objetos dados.

Page 199: Resolução de problemas

182 5 Contagem

Por exemplo, todas as permutações dos 3 elementos do conjunto

A = a1, a2, a3 são:

σ1 = (a1, a2, a3),

σ2 = (a1, a3, a2),

σ3 = (a2, a1, a3),

σ4 = (a2, a3, a1),

σ5 = (a3, a1, a2),

σ6 = (a3, a2, a1).

Proposição 5.19. Seja n ≥ 1. O número total de permutações sim-

ples de n objetos O = o1, o2, . . . , on é dado por Pn = n!

Demonstração. É claro que a fórmula vale para n = 1. Vejamos agora

que existe a seguinte relação entre Pn e Pn−1 para n ≥ 2:

Pn = nPn−1. (5.5)

Para comprovar isto, para cada i denamos Ai como sendo as permu-

tações dos n − 1 objetos o1, . . . , oi−1, oi+1, . . . , on. Note que |Ai| =

Pn−1, para cada i = 1, 2, . . . , n. Assim, para obtermos uma permu-

tação dos n objetos, basta que xemos o objeto inicial oi e tomemos

um elemento do conjunto Ai, que é uma permutação dos n−1 objetos

restantes. Pelo princípio aditivo, temos que:

Pn = |A1|+ |A2|+ · · ·+ |An| = nPn−1.

Como a equação (5.5) é válida para todo n ≥ 2, podemos aplicá-la

para n− 1, obtendo:

Pn−1 = (n− 1)Pn−2,

Page 200: Resolução de problemas

5.4 Permutações Simples 183

de onde vem que

Pn = n(n− 1)Pn−2.

Repetindo este argumento, obtemos que

Pn = n(n− 1)(n− 2) · · · 3 · 2 · 1 = n!,

como queríamos demonstrar.

Exemplo 5.20. De quantas maneiras podemos formar uma la com

4 pessoas?

Demonstração. Observe que se enumeramos os lugares da la e enu-

meramos as pessoas, pa, pb, pc, pd, cada distribuição vai corresponder a

uma permutação do conjunto 1, 2, 3, 4. Por exemplo, a distribuição

(pc, pa, pb, pd) corresponde à permutação (3, 1, 2, 4). Assim, o número

de distribuições na la é 4! = 24.

Exemplo 5.21. De quantas maneiras k moças e k rapazes podem

formar pares para uma dança?

Solução. Estando as moças em uma la e os rapazes em outra, pode-

mos enumerá-los com números de 1, 2, . . . , k. A uma permutação des-

ses números, digamos (a1, a2, . . . , ak) com ai ∈ 1, 2, . . . , k faremos

uma associação da mulher i com o rapaz ai. Por exemplo, a permu-

tação (2, 1, 3, . . . , k) signica que a moça 1 dançará com o rapaz 2, a

moça 2 com o rapaz 1, e a moça i com o rapaz i, para i ≥ 3.

Observe que toda associação de k moças e k rapazes produz uma

permutação, de modo que o número de associações possíveis das mo-

ças com os rapazes é igual ao número de permutações dos elementos

do conjunto 1, 2, 3, . . . , k. Pela Proposição 5.19 existem k! modos

diferentes de combinar as moças com os rapazes.

Page 201: Resolução de problemas

184 5 Contagem

5.5 Arranjos Simples

Denição 5.22. Consideremos n objetos e p um inteiro positivo tal

que 0 < p ≤ n. Um arranjo simples de classe p dos n objetos dados

é uma seleção de p objetos distintos dentre estes que diferem entre si

pela ordem de colocação ou pela natureza de cada um, isto é, o que

importa é quem participa ou o lugar que ocupa. Denotaremos por Apno número de arranjos simples de classe p de n objetos.

Por exemplo, dados os objetos o1, o2 e o3 todos os arranjos possíveis

de classe 2 são: A1 = (o1, o2), A2 = (o2, o1), A3 = (o1, o3), A4 =

(o3, o1), A5 = (o2, o3) e A6 = (o3, o2).

Observação 5.23. Notemos que um arranjo simples de classe n de n

objetos dados não é mais que uma permutação desses n objetos. Logo,

Pn = Ann = n!.

Proposição 5.24. Seja n ≥ 1. O número total de arranjos simples

de classe p de n objetos O = o1, o2, . . . , on é dado por Apn = n!(n−p)! .

Demonstração. Para n = 1 a fórmula é obviamente válida. Similar-

mente ao caso das permutações, primeiramente provaremos que para

n ≥ 2 vale a seguinte igualdade:

Apn = nAp−1n−1. (5.6)

Agora denimos os conjuntos Ai como sendo os arranjos simples de

classe p − 1 dos n − 1 objetos o1, . . . , oi−1, oi+1, . . . , on. Note que

|Ai| = Ap−1n−1, para cada i = 1, 2, . . . , n. Assim, para obtermos um

arranjo simples de classe p dos n objetos, basta que xemos o objeto

inicial oi e tomemos um elemento do conjunto Ai, que é uma arranjo

Page 202: Resolução de problemas

5.5 Arranjos Simples 185

de classe p − 1 dos n − 1 objetos restantes. Pelo princípio aditivo,

temos que:

Apn = |A1|+ |A2|+ · · ·+ |An| = nAp−1n−1.

Como nossa equação (5.6) é válida para todo n ≥ 2, podemos aplicá-la

para n− 1, obtendo:

Ap−1n−1 = (n− 1)Ap−2n−2,

de onde vem que

Apn = n(n− 1)Ap−2n−2.

Repetindo este argumento sucessivamente, obtemos que

Apn = n(n− 1)(n− 2) · · · (n− (p− 2))Ap−(p−1)n−(p−1)

= n(n− 1)(n− 2) · · · (n− p+ 2)A1n−p+1.

Notemos agora que A1n−p+1 = n − p + 1; logo, da igualdade anterior

segue-se que

Apn = n(n− 1)(n− 2) · · · (n− p+ 2)(n− p+ 1)

=n(n− 1)(n− 2) · · · (n− p+ 2)(n− p+ 1)× (n− p) · · · 1

(n− p) · · · 1

=n!

(n− p)! ,

como desejávamos.

Agora vamos dar alguns exemplos de como aparecem problemas

práticos que requerem fazer este tipo de cálculo. O primeiro dele tem

Page 203: Resolução de problemas

186 5 Contagem

a ver com a formação de palavras diferentes com um conjunto dado

de letras.

Um anagrama de uma palavra é uma permutação de letras dessa

palavra para formar outra, a qual pode carecer de signicado. Por

exemplo:

• um anagrama de amor é roma;

• um anagrama de celia é alice;

• um anagrama de caterina é natercia;

• um anagrama de elvis é lives.

Exemplo 5.25. Quantos anagramas de p letras distintas podemos

formar com um alfabeto de 23 letras, sendo p < 23?

Solução. Como as letras são diferentes, nosso problema consiste em

achar todos os arranjos de classe p de 23 objetos dados, que neste caso

são as 23 letras do alfabeto. Logo, este número é

Ak23 =23!

(23− k)!.

Exemplo 5.26. De quantos modos 2 pessoas podem se sentar em 5

cadeiras que estão em la?

Solução. Este problema é equivalente a achar o número total de ar-

ranjos de classe 2 de 5 objetos, correspondendo as 5 cadeiras aos 5

objetos e as duas pessoas indicando a ordem do arranjo. Logo, este

número é dado por

A25 =

5!

3!= 20.

Page 204: Resolução de problemas

5.5 Arranjos Simples 187

Exemplo 5.27. Considere os dígitos 2, 3, 4, 5, 7 e 9. Supondo que a

repetição de dígitos não seja permitida, responda às seguintes pergun-

tas:

(a) Quantos números de três dígitos podem ser formados?

(b) Entre os achados em (a) quantos são pares?

(c) Entre os achados em (a) quantos são ímpares?

(d) Entre os achados em (a) quantos são múltiplos de 5?

(e) Entre os achados em (a) quantos são menores do que 400?

Solução. Seja O = 2, 3, 4, 5, 7, 9 nosso conjunto de objetos.

(a) A quantidade de números de três dígitos que podemos formar

sem repetição de algum deles é claramente o número de arranjos

de classe 3 dos 6 dígitos de O, isto é,

A36 =

6!

3!= 120.

(b) Sabemos que em todo número par o último dígito é um múltiplo

de 2, isto é, ele acaba em 0, 2, 4, 6 ou 8. Então, em nosso caso

as únicas possibilidades são que o número termine em 2 ou 4.

Supondo que o último dígito seja 2, temos que preencher as duas

casas restantes com os dígitos pertencentes ao conjunto O−2.Assim, existem A2

|O−2| = A25 = 5!

3!= 20 números dos achados

em (a) que nalizam em 2. De forma análoga, existem A2|O−4| =

A25 = 5!

3!= 20 números dos achados em (a) que nalizam em 4.

Logo, entre os números achados em (a) existem 20 + 20 = 40

números pares.

Page 205: Resolução de problemas

188 5 Contagem

(c) Todo conjunto de números pode ser dividido em duas classes

disjuntas: a classe dos números pares e a classe dos números ím-

pares que pertencem ao mesmo. Segue-se que dentre os números

achados em (a) existem 120− 40 = 80 números ímpares.

(d) Todo número múltiplo de 5 acaba em 0 ou 5; no nosso caso te-

mos que a única possibilidade para o último dígito é 5. Assim

o problema consiste em preencher as duas casas restantes com

dígitos do conjunto O − 5. De onde se segue que a quanti-

dade de números múltiplos de 5 existentes em (a) vem dada por

A2|O−5| = A2

5 = 5!3!

= 20.

(e) Para obter os números menores do que 400 a casa das centenas

só poderá ser ocupada pelos dígitos 1, 2 ou 3. Como 1 /∈ O,temos que as únicas possibilidades em nosso caso são 2 ou 3.

Então, supondo que o primeiro dígito do número seja 2, devemos

preencher duas casas restantes com os dígitos pertencentes a

O − 2. De forma análoga, existem A2|O−3| = A2

5 = 5!3!

= 20

números dos achados em (a) e que começam com 3. Logo, dentre

os números achados em (a) existem 20+20 = 40 menores do que

400.

5.6 Combinações Simples

O conceito de combinação simples surge naturalmente quando tenta-

mos responder à seguinte pergunta:de quantas formas diferentes pode-

mos selecionar p objetos dentro de n objetos dados?

Page 206: Resolução de problemas

5.6 Combinações Simples 189

Por exemplo, suponha que queremos enfeitar uma festa de aniver-

sário com bolas de dois tipos de cores e na loja onde as compraremos

existem bolas nas cores azul, verde e vermelha. De quantas formas

distintas podemos enfeitar nossa festa? É claro que podemos enfeitar

a festa de 3 formas diferentes: com bolas em azul e verde; com bolas

em azul e vermelho ou com bolas em verde e vermelho.

Notemos que, ao contrário do caso em que trabalhamos com arran-

jos, quando fazemos uma seleção de duas cores não estamos inte-

ressados na ordem em que elas foram escolhidas.

Denição 5.28. Consideremos n objetos e p um inteiro positivo tal

que 0 < p ≤ n. Uma combinação simples de classe p dos n objetos

dados é uma seleção de p objetos distintos entre estes que diferem

entre si apenas pela natureza de cada um, isto é, o que importa é

simplesmente quem participa no grupo selecionado. Denotaremos por(np

)o número de combinações simples de classe p de n objetos.

Proposição 5.29. Seja n ≥ 1. O número total de combinações

simples de classe p de n objetos O = o1, o2, . . . , on é dado por(np

)= n!

p!(n−p)! .

Demonstração. Veremos a seguir que arranjos simples e combinações

simples de classe p estão estreitamente relacionados. Com efeito, para

cada combinação simples formada por p objetos distintos de O pode-

mos gerar todos os arranjos simples de classe p formados por estes p

objetos. Basta para isto fazer todas as suas permutações possíveis.

Obtém-se assim p ! arranjos simples diferentes com esses p objetos.

Resumindo, para cada combinação simples de classe p formada com

p objetos diferentes de O podemos fazer p ! arranjos simples diferen-

tes de classe p com estes mesmos objetos; logo, no total, teremos a

Page 207: Resolução de problemas

190 5 Contagem

seguinte relação:

p !

(n

p

)= Apn =

n!

(n− p)! ,

de onde segue-se que(n

p

)=

n!

p!(n− p)! .

Exemplo 5.30. De quantas formas diferentes podemos construir uma

palavra de tamanho n com i letras a e n− i letras b?

Solução. A solução do problema equivale em escolher a posição das

i letras a em questão, uma vez que a posição das (n − i) letras b

restantes estará determinada. Se enumeramos as posições das letras

de 1 a n, uma palavra será formada ao xarmos a posição das i letras

a. Isso é exatamente(ni

), já que corresponde ao número de grupos

com i elementos (posições com letra a) tomados em um conjunto de n

elementos (todas as posições), que diferem somente por sua natureza.

Exemplo 5.31. De quantas formas podemos dividir um grupo 5 pes-

soas em um grupo de duas e outro de três?

Solução. Temos(52

)= 5 !

2 !3 != 10 formas diferentes de escolher duas

pessoas do grupo. Por cada uma dessas escolhas o outro grupo de três

pessoas é automaticamente determinado; logo, temos 10 possibilidades

diferentes de fazer a divisão.

Exemplo 5.32. De quantos modos podemos dividir 6 pessoas em:

(a) Dois grupos de 3 pessoas cada?

Page 208: Resolução de problemas

5.6 Combinações Simples 191

(b) Três grupos de 2 pessoas cada?

Solução. Começamos por (a). À primeira vista, parece que a resposta

deve ser(n3

)= 6!

3! 3!= 20, similarmente ao exemplo anterior. Porém,

aqui há um problema devido ao fato de estarmos dividindo em grupos

que têm a mesma quantidade de pessoas e, portanto, as permutações

de cada dois grupos formados são consideradas divisões iguais; logo,

devemos dividir o resultado por 2 !, obtendo assim 10 formas diferentes

de obter dois grupos com 3 pessoas cada.

Para resolver o item (b) seguimos os seguintes passos:

• Primeiramente calcularemos o número de formas possíveis para

dividir 6 pessoas em um grupo de 2 e outro grupo de 4; esta

quantidade vem dada por(62

)= 6!

4! 2!.

• Agora dividiremos as 4 pessoas restantes em um grupo de 2 e

outro grupo de 2; esta quantidade vem dada por(42

)= 4!

2! 2!.

Pelo princípio multiplicativo temos que existem(62

)(42

)= 6!

(2!)3possi-

bilidades de dividir 6 pessoas em 3 grupos com duas pessoas cada.

Igualmente ao caso anterior, aqui as permutações possíveis de cada 3

grupos formados são consideradas iguais; logo, devemos dividir este

último resultado por 3 !. Portanto, existem 15 formas diferentes de

dividir 6 pessoas em três grupos de 2 pessoas cada.

Exemplo 5.33. Se você possui 10 amigos, de quantas maneiras você

pode escolher dois ou mais deles para jantar?

Solução. Esquematizamos a solução da seguinte maneira:

• Primeiramente, vamos encontrar a quantidade de maneiras pelas

quais você pode jantar com 2 amigos; isto é feito de(102

)formas

diferentes.

Page 209: Resolução de problemas

192 5 Contagem

• Depois, vamos encontrar a quantidade de maneiras pelas quais

você pode jantar com 3 amigos; isto é feito de(103

)formas dife-

rentes.

• Em seguida, encontramos a quantidade de maneiras pelas quais

você pode jantar com 4 amigos; isto é feito de(104

)formas dife-

rentes.

• Em geral, o número de maneiras diferentes que você tem de

jantar com p amigos é dado por(10p

).

Pelo princípio aditivo, temos que a quantidade de formas diferentes

que você tem de jantar com 2 ou mais de seus amigos, é dada por

(10

2

)+

(10

3

)+ · · ·+

(10

9

)+

(10

10

)= 1013,

sendo este o número procurado.

Exemplo 5.34. De um grupo de 10 pessoas das quais 4 são mulheres,

quantas comissões de 5 pessoas podem ser formadas de modo que pelo

menos uma mulher faça parte?

Solução. Sendo que o grupo tem 10 pessoas e 4 destas são mulheres,

segue-se que no grupo temos 6 homens. Para formar um grupo de 5

pessoas com pelo menos uma mulher, temos as seguintes alternativas:

• Nosso grupo é composto por uma mulher e 4 homens; neste caso

poderemos formar(41

)(64

)= 60 comissões de 5 pessoas.

• Nosso grupo é composto por 2 mulheres e 3 homens; neste caso

poderemos formar(42

)(63

)= 120 comissões de 5 pessoas.

Page 210: Resolução de problemas

5.7 O Binômio de Newton 193

• Nosso grupo é composto por 3 mulheres e 2 homens; neste caso

poderemos formar(43

)(62

)= 60 comissões de 5 pessoas.

• Nosso grupo é composto por 4 mulheres e um homem; neste caso

poderemos formar(44

)(61

)= 6 comissões de 5 pessoas.

Pelo princípio aditivo temos que é possível formar 246 comissões de 5

pessoas de modo que pelo menos uma mulher faça parte.

5.7 O Binômio de Newton

Nesta seção, estudaremos uma fórmula que generaliza a conhecida

expressão

(a+ b)2 = a2 + 2ab+ b2.

Essa fórmula é conhecida como o binômio de Newton ou fórmula bino-

mial de Newton, devido ao Matemático Isaac Newton (1642-1727). A

fórmula binomial de Newton pode ser motivada pelas seguintes igual-

dades que são fáceis de vericar:

(a+ b)1 = a+ b =

(1

0

)a+

(1

1

)b,

(a+ b)2 = a2 + 2ab+ c2 =

(2

0

)a2 +

(2

1

)ab+

(2

2

)b2,

(a+ b)3 = a3 + 3a2b+ 3ab2 + c3 =

(3

0

)a3 +

(3

1

)a2b+

(3

2

)ab2 +

(3

3

)b3.

Os casos particulares acima podem ser estendidos para qualquer po-

tência inteira positiva de a+ b, ou seja, vale o seguinte resultado:

Page 211: Resolução de problemas

194 5 Contagem

Teorema 5.35 (Fórmula Binomial de Newton). Sejam a e b númerosreais e n ∈ N, então

(a+b)n =

(n

0

)an+

(n

1

)an−1b1+· · ·+

(n

i

)an−ibi+· · ·+

(n

n− 1

)a1bn−1+

(n

n

)bn.

Os números(ni

), 0 ≤ i ≤ n, são chamados também de coecientes bino-

miais.

Demonstração. Expandimos o binômio no produto de seus n fatores,

isto é,

(a+ b)n = (a+ b)(a+ b) · · · (a+ b)︸ ︷︷ ︸n−fatores

. (5.7)

Se desenvolveremos o produto destes n fatores iguais acima obtemos

uma soma nita de termos da forma a1a2 · · · an, onde cada aj, 1 ≤j ≤ n, toma valor a ou b. Notemos que em cada termo se o número

b aparece i vezes, então o número a aparecerá (n − i) vezes, ou seja,

quando cada termo for multiplicado deverá tomar valor igual a an−ibi,

para algum 1 ≤ i ≤ n. Por exemplo, os n termos

abb · · · b = abn, bab · · · b = abn, . . . , bbb · · · ba = abn

têm o mesmo valor . Assim, para calcular o coeciente do termo aibn−i

que aparece na equação (5.7), basta responder à seguinte pergunta: de

quantos modos podemos formar uma palavra com i letras a e (n− i)letras b? A resposta dessa pergunta foi estudada no Exemplo 5.30 e é

simplesmente(ni

). Logo, a expressão na equação (5.7) é

(a+ b)n =

(n

0

)an +

(n

1

)an−1b1 + · · ·+

(n

n− 1

)a1bn−1 +

(n

n

)bn,

o que prova o teorema.

Page 212: Resolução de problemas

5.8 Contagem e Probabilidades 195

A fórmula binomial de Newton nos dá algumas propriedades interes-

santes dos coecientes binomiais que resumimos na próxima proposi-

ção.

Proposição 5.36. Seja n ∈ N. As seguintes igualdades são válidas:

(a)(n0

)+(n1

)+ · · ·+

(ni

)+ · · ·+

(nn−1)

+(nn

)= 2n;

(b)(n0

)−(n1

)+ · · ·+ (−1)i

(ni

)+ · · ·+ (−1)n

(nn

)= 0.

Demonstração. Para a letra (a), basta tomar a = b = 1 e expanda

2n = (1 + 1)n no Binômio de Newton. Para a letra (b), tome a = 1

e b = −1 e expanda 0 = (1− 1)n no binômio de Newton, observando

que (−1)n é igual a 1 se n é par, e igual a 1 se n é ímpar.

5.8 Contagem e Probabilidades

Uma das aplicações interessantes da contagem de elementos de um

conjunto é quando desejamos estudar a probabilidade de eventos alea-

tórios. Por exemplo, se lançarmos um dado de seis faces, temos os

seguintes resultados possíveis:

Ω = 1, 2, . . . , 6.

Se desejamos saber qual é a chance de que ocorra um número

primo no lançamento, devemos contar quantos primos aparecem em

1, 2, 3, 4, 5, 6 e dividir por 6. Ou seja, a chance de ocorrer um número

primo num lançamento de um dado de seis faces é 3/6 = 0, 5.

Denimos a probabilidade de um subconjunto A ⊂ Ω como o nú-

mero

p(A) =|A||Ω| .

Page 213: Resolução de problemas

196 5 Contagem

Também chamamos o subconjunto Ω de todos os resultados possíveis

de espaço amostral e um subconjunto A de Ω de evento. Por exemplo,

podemos calcular a probabilidade de escolhermos um número par no

conjunto 1, 2, 3, . . . , 15. Neste caso, o conjunto Ω está claro e é igual a

Ω = 1, 2, 3, . . . , 15. O conjunto A é A = 2, 4, 6, . . . , 14. Logo,

p(A) =|A||Ω| =

7

15.

Assim, ca claro que a maior diculdade para calcular a proba-

bilidade de um evento é contar quantos elementos pertencem a este

evento e quantos elementos pertencem ao espaço amostral. A seguir,

veremos um exemplo mais elaborado onde aplicamos a noção de ar-

ranjo simples.

Exemplo 5.37. Calcular a probabilidade de que escolhendo um grupo

de 44 pessoas, existam pelo menos duas que fazem aniversário no

mesmo dia do ano.

Solução. Podemos reescrever isso do seguinte modo: num saco existem

bolas enumeradas com os números 1, 2, . . . , 365 (correspondentes aos

dias do ano). Retiramos a bola b1 e anotamos o número que apareceu.

Devolvemos a bola ao saco e efetuamos uma nova retirada, anotando

novamente o número que aparece. Repetindo este processo 44 vezes,

obtemos uma lista com 44 números. Assim, a pergunta se transforma

em: de quantos modos diferentes podemos escolher 44 bolas enume-

radas com os números 1, 2, 3, . . . , 365 com reposição, tal que existam

pelo menos duas bolas com o mesmo número?

A primeira coisa que devemos fazer é calcular o espaço amostral,

de todas as possibilidades possíveis de resultado. Como escolhemos

44 bolas enumeradas num saco, cada resultado possível é uma lista

Page 214: Resolução de problemas

5.9 Exercícios Propostos 197

(n1, n2, . . . , n44) com 44 números. Observe que, pelo princípio multi-

plicativo, |Ω| = 36544, pois temos 365 opções para escolher n1, 365

opções para escolher n2, etc.A segunda pergunta trata-se de saber quantos resultados são favo-

ráveis, ou seja, quantas são as escolhas tais que existam pelo menosduas bolas com o mesmo número. Para isso é mais fácil contar quantasescolhas existem tais que os 44 números são diferentes. Neste caso,devemos escolher uma ordenação de 44 números distintos entre 365.Isso corresponde à quantidade de arranjos de classe 44 num grupo de365 elementos. Assim, concluímos que a probabilidade de que esteevento ocorra é

p =36544 −A44

365

36544= 1−

365!(365!−44!)36544

.

Obter um valor aproximado para o número acima com o computador é

uma tarefa fácil nos dias atuais. Porém, aproximar expressões envolvendo

fatoriais (sem o uso do computador) é um fato conhecido há muito tempo

pela humanidade, através da famosa fórmula de Stirling.1 Com a ajuda

desta fórmula, obtemos que p é aproximadamente p ∼= 0.93, como havíamos

prometido no Capítulo 1.

Além dos exercícios abaixo, recomendamos a leitura de [9]. Lá, o

leitor encontrará material adicional sobre análise combinatória, bem

como uma ampla variedade de problemas.

5.9 Exercícios Propostos

1. De quantas maneiras podemos escolher três números distintos do

conjunto I50 = 1, 2, 3, . . . , 49, 50 de modo que sua soma seja

1Grosseiramente, a fórmula de Stirling diz que o quociente entre n! e√2πnnne−n está próximo de 1, para valores de n grandes.

Page 215: Resolução de problemas

198 5 Contagem

a) um múltiplo de 3?

b) um número par?

2. Considere o conjunto In = 1, 2, 3, . . . , n−1, n. Diga de quantosmodos é possível formar subconjuntos de k elementos nos quais

não haja números consecutivos?

3. Considere as letras da palavra PERMUTA. Quantos anagramas

de 4 letras podem ser formados, onde:

a) não há restrições quanto ao número de consoantes ou vogais?

b) o anagrama começa e termina por vogal?

c) a letra R aparece?

d) a letra T aparece e o anagrama termina por vogal?

4. Calcular a soma de todos os números de 5 algarismos distintos

formados com os algarismos 1, 3, 5, 7 e 9.

5. Quantos números podem ser formados pela multiplicação de al-

guns ou de todos os números 2, 2, 3, 3, 3, 5, 5, 6, 8, 9, 9?

6. Entre todos os números de sete dígitos, diga quantos possuem

exatamente três dígitos 9 e os quatro dígitos restantes todos

diferentes?

7. De quantas maneiras podemos distribuir 22 livros diferentes en-

tre 5 alunos se 2 deles recebem 5 livros cada e os outros 3 recebem

4 livros cada?

8. Quantos são os números naturais de sete dígitos nos quais o

dígito 4 gura exatamente 3 vezes e o dígito 8 gura exatamente

2 vezes?

Page 216: Resolução de problemas

5.9 Exercícios Propostos 199

9. De quantas maneiras uma comissão de 4 pessoas pode ser for-

mada, de um grupo de 6 homens e 6 mulheres, se a mesma é

composta de um número maior de homens do que de mulheres?

10. O comprimento de uma palavra é a quantidade de caracteres que

ela possui. Encontre a quantidade de palavras de comprimento

5 que podemos formar fazendo uso de 10 caracteres distintos, de

forma que não existam três caracteres consecutivos idênticos em

cada palavra.

11. Quantos números inteiros existem entre 1 e 10.000 que não são

divisíveis por 3, 5 e 7?

12. Quantas são as permutações da palavra PROPOR nas quais não

existem letras consecutivas iguais?

13. De quantos modos 6 casais podem sentar-se ao redor de uma

mesa circular de tal forma que marido e mulher não quem jun-

tos?

14. Quantas são as permutações das letras da palavra BRASIL em

que o B ocupa o primeiro lugar, ou o R ocupa o segundo lugar,

ou o L o sexto lugar?

15. De quantas formas podemos representar o número 15 como soma

de vários números naturais?

16. Quantos quadrados perfeitos existem entre 40.000 e 640.000 que

são múltiplos simultaneamente de 3, 4 e 5?

17. Oito amigos vão ao cinema assistir a um lme que custa um real.

Quatro deles possuem uma nota de um real e quatro possuem

Page 217: Resolução de problemas

200 5 Contagem

uma nota de dois reais. Sabendo-se que o caixa do cinema não

possui nenhum dinheiro, como eles podem organizar uma la

para pagar o lme permitindo o troco pelo caixa?

18. Se considerarmos todas as congurações do tabuleiro com duas

torres que não se atacam, como no Exemplo 5.2, sem distinguir

as torres, quantas congurações obteremos?

19. Continuando o problema anterior, generalize-o para 3, 4, 5, . . .

torres que não se atacam, encontrando também o número máxi-

mo de torres que podem ser colocadas no tabuleiro de modo que

duas delas não se ataquem.

20. Tente fazer o problema anterior para cavalos de xadrez.

21. Mostre que em toda sequência de n2 + 1 inteiros distintos possui

uma subsequência crescente de n + 1 elementos ou uma sub-

sequência decrescente de n+ 1 elementos.

22. Encontre o número de zeros que termina o número 2010!.

23. O jogo do 7 consiste em lançar dois dados e somar o número

obtido nas suas faces. Caso a soma seja 7, o jogador A ganha o

dois reais do jogador B. Caso a soma não seja 7, o jogador B

ganha um real de A. Pergunta-se: quem leva vantagem?

24. A função φ de Euler associa a cada número natural n o valor

φ(n) igual ao número de inteiros positivos menores ou iguais a

n relativamente primos com n. Ou seja,

φ(n) =∣∣1 ≤ m ≤ n; (m,n) = 1

∣∣.

Page 218: Resolução de problemas

5.9 Exercícios Propostos 201

Usando os princípios estudados, mostre que se n se decompõe

em fatores primos como n = p1α1p2α2 . . . pαkk , então

φ(n) = n

(1− 1

p1

)(1− 1

p2

). . .

(1− 1

pk

).

O leitor pode achar mais informações sobre a função φ de Euler

nos livros [11] ou ainda [10].

Page 219: Resolução de problemas

202 5 Contagem

Page 220: Resolução de problemas

6

Indução Matemática

Se as pessoas não a ham a Matemáti a simples é só por que aindanão per eberam o quanto a vida é ompli ada. John von NeumannImagine uma la com innitos dominós, um atrás do outro. Supo-

nha que eles estejam de tal modo distribuídos que, uma vez que um

dominó caia, o seu sucessor na la também cai. O que acontece quando

derrubamos o primeiro dominó?

Apesar da simplicidade da pergunta acima ela traz em sua essência

toda a ideia usada no método da indução nita. Muitas descobertas

em Matemática são feitas baseadas na realização de testes que nos

fornecem evidências empíricas. Tais evidências são estudadas para

efetivamente vericarmos se os resultados que elas insinuam são ver-

dadeiros. O método da indução nita constitui uma ferramenta muito

útil na hora de desvendar a veracidade de resultados provenientes deste

tipo de estudo. Esse método é uma das grandes armas do matemático

moderno e tem utilidade na solução de vários problemas, como iremos

ver ao longo deste capítulo.

203

Page 221: Resolução de problemas

204 6 Indução Matemática

6.1 Formulação Matemática

No início do século XX, o matemático Giuseppe Peano (1858-1932)

estabeleceu os axiomas necessários que nos permitem hoje descrever

com precisão o conjunto dos números naturais. O último dos seus

axiomas diz o seguinte: seja A um subconjunto de N (A ⊂ N). Se 1 ∈A e se, além disso, A contém todos os sucessores dos seus elementos,

então A = N.Este axioma é conhecido como axioma de indução e serve como

base do método de demonstração por indução, o qual é de grande

utilidade para estabelecer provas rigorosas em Matemática.

O princípio da boa ordenação dos naturais, enunciado no Capí-

tulo 3, e o axioma de indução não são independentes e sem nenhuma

conexão. De fato, eles são equivalentes, ou seja, se consideramos o

princípio da boa ordenação como sendo um postulado podemos dedu-

zir o axioma de indução e, reciprocamente, se consideramos o axioma

de indução como sendo um postulado podemos deduzir o princípio da

boa ordenação.

No resto do capítulo, p(n) representa uma armação em relação ao

natural n, podendo esta ser verdadeira ou falsa.

Teorema 6.1 (Princípio da Indução Finita). Considere n0 um in-

teiro não negativo. Suponhamos que, para cada inteiro n ≥ n0, seja

dada uma proposição p(n). Suponha que se pode vericar as seguintes

propriedades:

(a) p(n0) é verdadeira;

(b) se p(n) é verdadeira então p(n + 1) também é verdadeira, para

todo n ≥ n0.

Page 222: Resolução de problemas

6.1 Formulação Matemática 205

Então, p(n) é verdadeira para qualquer n ≥ n0.

A armação (a) é chamada de base da indução e a (b) de passo

indutivo. O fato de que p(n) é verdadeira no item (b) é chamado de

hipótese da indução.

Demonstração. Denamos o conjunto

V = m inteiros não negativos; m ≥ n0 e p(m) é verdadeira .

Notemos que V é não vazio, pois a condição (a) nos assegura que

n0 ∈ V . A prova do teorema é equivalente a mostrarmos que

V = n0, n0 + 1, n0 + 2, n0 + 3, · · · ,

ou equivalentemente, a provarmos que o conjunto

F = m inteiros não negativos; m ≥ n0 e p(m) é falsa

é vazio. Suponhamos que F é não vazio. Pelo principio da boa orde-

nação existe um menor elemento m0 ∈ F , onde p(m0) é falso. Obser-

vemos que,

• m0 ≥ n0 + 1. De fato, m0 ≥ n0, porém a possibilidade m0 = n0

contradiz a condição (a);

• m0 − 1 ∈ V . Com efeito, p(m0 − 1) é verdadeira pois, caso

contrário, m0−1 ∈ F e, além disso, m0−1 < m0, contradizendo

isto a minimalidade de m0.

Finalmente, como p(m0 − 1) é verdadeira, segue da condição (b) que

p(m0) também é verdadeira, o que é impossível pela denição de m0.

Portanto, o conjunto F é vazio, concluindo-se assim a prova.

Page 223: Resolução de problemas

206 6 Indução Matemática

Para um pouco mais sobre a relação entre os princípios de indução

e da boa ordenação, recomendamos o Apêndice A da referência [11].

Observação 6.2. Uma grande vantagem do princípio da indução -

nita é poder provar que uma quantidade innita de armações são

verdadeiras, simplesmente vericando que uma quantidade nita des-

tas armações são verdadeiras. Deixaremos clara a utilidade deste

método resolvendo alguns problemas na próxima seção.

6.2 Aplicações

Dentro da grande gama de problemas que podem ser abordados apli-

cando o método de indução podemos distinguir três importantes gru-

pos:

• demonstração de identidades;

• demonstração de desigualdades;

• demonstração de problemas de divisibilidade.

A seguir damos vários exemplos de como aplicar o método em

problemas referentes a cada um destes grupos.

6.2.1 Demonstrando Identidades

Começamos com os seguintes problemas clássicos:

(P1) Determinar uma fórmula para a soma dos n primeiros números

pares, isto é,

sp(n) := 2 + 4 + 6 + · · ·+ 2n.

Page 224: Resolução de problemas

6.2 Aplicações 207

(P2) Determinar uma fórmula para a soma dos n primeiros números

ímpares, isto é,

si(n) := 1 + 3 + 5 + · · ·+ 2n− 1.

Para induzir ambas as fórmulas, primeiro fazemos os cálculos para

vários valores de n, os quais apresentamos na seguinte tabela.

n 1 2 3 4 5 · · ·sp(n) 2 = 1 · 2 6 = 2 · 3 12 = 3 · 4 20 = 4 · 5 30 = 5 · 6 · · ·si(n) 1 = 12 4 = 22 9 = 32 16 = 42 25 = 52 · · ·

Os resultados na tabela sugerem que sp(n) = n(n + 1) e que

si(n) = n2. Entretanto, isto não constitui por si só uma prova ri-

gorosa destas fórmulas, pois para poder garantir a veracidade delas

utilizando a tabela teríamos que vericar cada valor de n natural,

sendo isto impossível. Provaremos agora que, de fato, as fórmulas

induzidas são válidas usando o método de indução nita.

Exemplo 6.3. Demonstre que para qualquer n ∈ N é válida a igual-

dade:

2 + · · ·+ 2n = n(n+ 1).

Solução. Denamos a proposição

p(n) : 2 + · · ·+ 2n = n(n+ 1)

e observemos que a mesma vale para n = 1 (base da indução); de fato

p(1) : 2 = 1(1 + 1).

Agora partimos para a prova do passo indutivo:

Page 225: Resolução de problemas

208 6 Indução Matemática

• Hipótese: suponhamos que p(k) é verdadeira para um certo k >

1, k ∈ N.

• Tese: devemos mostrar que p(k + 1) também é verdadeira.

Com efeito, como

2 + · · ·+ 2k = k(k + 1),

somando 2(k + 1) a ambos os lados desta igualdade, temos que

2 + · · ·+ 2k + 2(k + 1) = k(k + 1) + 2(k + 1)

= (k + 2)(k + 1).

Esta última igualdade arma que p(k + 1) também é verdadeira. O

Princípio de Indução nos garante que p(n) é verdadeira para qualquer

n ∈ N.

Exemplo 6.4. Demonstre que para qualquer n ∈ N é válida a igual-

dade:

1 + 3 + 5 + · · ·+ 2n− 1 = n2.

Solução. Aqui denimos a proposição:

p(n) : 1 + 3 + 5 + · · ·+ 2n− 1 = n2

e notamos que a mesma é válida se tomarmos, por exemplo, n = 1.

De fato,

p(1) : 1 = 2 · 1− 1.

Agora só resta provar o passo indutivo:

• Hipótese: suponhamos que p(k) seja verdadeira para um certo

k > 1, k ∈ N.

Page 226: Resolução de problemas

6.2 Aplicações 209

• Tese: devemos mostrar que p(k + 1) também é verdadeira.

Com efeito, como

1 + 3 + 5 + · · ·+ 2k − 1 = k2,

somando 2k + 1 a ambos os lados desta igualdade, temos que

1 + 3 + 5 + · · ·+ 2k − 1 + 2k + 1 = k2 + 2k + 1

= (k + 1)2.

O princípio de indução nos garante que p(n) é verdadeira para

qualquer n ∈ N.

Uma consequência imediata do Exemplo 6.3 é a fórmula para a

soma dos n primeiros números naturais, dada por

sn = 1 + 2 + 3 + · · ·+ n =n(n+ 1)

2. (6.1)

Com efeito, como

2 + 4 + · · ·+ 2n = n(n+ 1),

então dividindo por 2 ambos os membros da igualdade acima, obtemos

a equação (6.1).

Continuando com o mesmo raciocínio, é natural nos perguntarmos

se é possível obter uma fórmula para a soma dos n primeiros quadrados

perfeitos, ou seja, determinar qn onde:

qn = 12 + 22 + 32 + · · ·+ n2.

Para induzir a fórmula, consideramos os valores de sn e qn numa tabela:

Page 227: Resolução de problemas

210 6 Indução Matemática

n 1 2 3 4 5 6 · · ·sn 1 3 6 10 15 21 · · ·qn 1 5 14 30 55 91 · · ·

Aparentemente não existe nenhuma relação entre sn e qn. Mas, seconsiderarmos o quociente qn/sn, vejamos o que acontece:

n 1 2 3 4 5 6 · · ·qn/sn 3/3 5/3 7/3 9/3 11/3 13/3 · · ·

Isso nos sugere que vale a relação

qnsn

=2n+ 1

3,

logo nosso candidato para valor de qn é

qn =sn(2n+ 1)

3=n(n+ 1)(2n+ 1)

6.

Convidamos o leitor a provar a veracidade da equação acima utilizando o

Método da Indução no Exercício 1 no nal do capítulo.

6.2.2 Demonstrando Desigualdades

Apresentamos agora alguns exemplos de como usar indução para provar

desigualdades.

Exemplo 6.5. Prove que 3n−1 < 2n2para todo n ∈ N.

Solução. Denotamos por p(n) a propriedade: 3n−1 < 2n2. É claro que p(1)

é válida, pois 1 < 2. Agora supondo que P (n) é verdadeira temos que

3n = 3n−1 · 3 < 2n2 · 22n+1 = 2(n+1)2 ,

logo p(n+1) também vale. Observamos que na desigualdade acima usamos

o fato de que 3 < 22n+1 para qualquer n ∈ N.

Page 228: Resolução de problemas

6.2 Aplicações 211

Exemplo 6.6. Mostre que para todo número n ∈ N, n > 3, vale que 2n < n!

Demonstração. Para n = 4 a desigualdade é vericada, pois 24 = 16 < 4! =

24. Vamos assumir como hipótese de indução que a desigualdade é válida

para n ≥ 4. Então, precisamos mostrar que a mesma vale também para

n+ 1. De fato, por hipótese de indução:

2n < n! (6.2)

Como 2 < n + 1, podemos multiplicar o lado esquerdo da desigualdade

em (6.2) por 2 e o lado direito por n+1, sem alterar o sinal de desigualdade.

Logo, temos que:

2n.2 = 2n+1 < n!(n+ 1) = (n+ 1)!,

concluindo-se a demonstração.

Exemplo 6.7. Prove que, para todo n ∈ N,√

2 +

√2 +

√2 + · · ·+

√2

︸ ︷︷ ︸n−radicais

< 2.

Demonstração. Claramente a desigualdade vale para n = 1, pois√2 < 2.

Suponhamos que para certo n ∈ N a desigualdade acontece, então√

2 +

√2 +

√2 + · · ·+

√2

︸ ︷︷ ︸n−radicais

< 2.

Logo, adicionando 2 em ambos os lados desta desigualdade tem-se

2 +

2 +

√2 +

√2 + · · ·+

√2

︸ ︷︷ ︸n−radicais

< 2 + 2.

Page 229: Resolução de problemas

212 6 Indução Matemática

Tomando raiz quadrada em ambos os lados desta última desigualdade ob-

temos √√√√2 +

2 +

√2 +

√2 + · · ·+

√2

︸ ︷︷ ︸n+1−radicais

< 2,

como desejávamos.

6.2.3 Indução e Problemas de Divisibilidade

Agora damos alguns exemplos de problemas de divisibilidade que podem

ser mostrados utilizando o método da indução:

Exemplo 6.8. Mostre que para qualquer n ∈ N, n3+2n é sempre divisível

por 3.

Solução. Para n = 1 a armação é válida, pois 13+2·1 = 3, que obviamente

é divisível por 3.

Assumamos como hipótese indutiva que a armação vale para algum

k ∈ N, isto é,

Hipótese: k3 + 2k é divisível por 3.

Devemos mostrar que a armação também é verdadeira para k + 1, ou

seja, temos que provar que

Tese: (k + 1)3 + 2(k + 1) é divisível por 3.

Para provar isto último, usamos o fato de que

(k + 1)3 + 2(k + 1) = (k3 + 3k2 + 3k + 1) + (2k + 2);

Page 230: Resolução de problemas

6.2 Aplicações 213

agrupando adequadamente,

(k + 1)3 + 2(k + 1) = (k3 + 2k) + (3k2 + 3k + 3)

= (k3 + 2k)︸ ︷︷ ︸múltiplo de 3

+3(k2 + k + 1)︸ ︷︷ ︸múltiplo de 3

= múltiplo de 3,

concluindo assim a prova.

Exemplo 6.9. Mostre que a soma dos cubos de três números naturais con-

secutivos é divisível por 9.

Solução. Denamos a seguinte proposição:

p(n) : n3 + (n+ 1)3 + (n+ 2)3 é um múltiplo de nove.

Notemos que P (1) é válida, pois

13 + 23 + 33 = 1 + 8 + 27 = 36 = 9 · 4.

Precisamos provar agora o passo indutivo, isto é,

• Hipótese: P (k) é verdadeira para algum k ∈ N.

• Tese: P (k + 1) também é verdadeira.

Para provar isto, observamos que

(k+1)3 + (k+2)3 + (k+3)3 = (k+1)3 + (k+2)3 + (k3 +9k2 +27k+27).

Ordenando adequadamente, temos que o lado direito da última igualdade

se escreve como

k3 + (k + 1)3 + (k + 2)3 + (9k2 + 27k + 27)

= k3 + (k + 1)3 + (k + 2)3︸ ︷︷ ︸múltiplo de 9

+9(k2 + 3k + 3)︸ ︷︷ ︸múltiplo de 9

= múltiplo de 9,

completando assim nossa demonstração.

Page 231: Resolução de problemas

214 6 Indução Matemática

Muitas vezes, para conseguir mostrar que a hipótese p(n + 1) é verda-

deira, precisamos supor que p(k) é verdadeira para todo n0 ≤ k ≤ n. Isto

é a base do princípio forte da indução nita que enunciamos a seguir:

Teorema 6.10 (Princípio Forte da Indução Finita). Considere n0 um in-

teiro não negativo. Suponhamos que, para cada inteiro n ≥ n0 seja dada

uma proposição p(n) e que valem as propriedades

(a) p(n0) é verdadeira;

(b) se para cada inteiro não negativo k, com n0 ≤ k ≤ n, temos que p(k)

é verdadeira, então p(n+ 1) é também verdadeira.

Então, a proposição p(n) é verdadeira para qualquer n ≥ n0.

Utilizando o princípio forte da indução, vamos dar uma prova diferente

do teorema fundamental da aritmética da apresentada no Capítulo 3.

Exemplo 6.11 (Teorema Fundamental da Aritmética). Todo número na-

tural N maior que 1 pode ser escrito como um produto

N = p1 · p2 · p3 · · · pm, (6.3)

onde m ≥ 1 é um número natural e os pi, 1 ≤ i ≤ m são números primos.

Além disso, a fatoração em (6.3) é única se exigirmos que p1 ≤ p2 ≤ · · · ≤pm.

Solução. Para cada n ∈ N, n ≥ 2, denamos a proposição

p(n) : n é escrito de modo único como um produto de números primos.

Notemos que p(2) é verdadeira, pois 2 é um número primo.

Agora enunciemos o passo indutivo:

• Hipótese indutiva: p(k) é verdade para cada inteiro k tal que 2 ≤ k ≤n.

Page 232: Resolução de problemas

6.3 Indução na Geometria 215

• Tese: p(n+1) é verdade. Em outras palavras, temos que mostrar que

n+ 1 é escrito de modo único como um produto de números primos.

Faremos a prova dividindo em dois casos:

(a) Se n + 1 é um número primo, então p(n + 1) é verdade e isto acaba

nossa demonstração.

(b) Se n + 1 não é um número primo, então existem α, β ∈ N com 2 ≤α ≤ n e 2 ≤ β ≤ n tais que n+ 1 = α · β.Nossa hipótese indutiva é válida para α e β. Isto signica que α se

escreve de modo único como um produto de números primos e que β

se escreve de modo único um produto de números primos. Portanto,

n+ 1 = α · β se escreve como um produto de números primos.

Agora mostraremos que n+1 se escreve de modo único como produto

de primos. Assuma que

p1p2 . . . pk = q1q2 . . . qm = n+ 1, (6.4)

com p1 ≤ p2 ≤ · · · ≤ pk e q1 ≤ q2 ≤ · · · ≤ qm todos primos. Vamos

mostrar que necessariamente k = m e pi = qi.

De fato, como p1 é primo, ele divide algum qi. Logo, como qi é primo,

p1 = qi ≥ q1. Analogamente, existe um j tal que q1 = pj ≥ p1. Logo,p1 = q1. Cancelando p1 em ambos os lados da equação (6.4), temos

que (n + 1)/p1 = p2 . . . pk = q2 . . . qm ≤ n. Logo, por hipótese de

indução, k = m e p2 = q2, . . . , pm = qm, encerrando a demonstração.

6.3 Indução na Geometria

Tratamos aqui alguns exemplos que mostram a utilidade do método de

indução na resolução de problemas geométricos. Vamos começar estudando

Page 233: Resolução de problemas

216 6 Indução Matemática

duas propriedades importantes dos polígonos. A primeira delas trata da

soma dos ângulos internos de um polígono convexo de n lados (n-ágono).

Um polígono convexo é um polígono tal que qualquer segmento de reta

que liga dois de seus pontos está contido no interior dele. No caso de

polígonos, isto é equivalente ao fato de que todo segmento que liga dois

vértices ou é uma aresta ou está contido no interior do polígono.

Exemplo 6.12. Mostre que a soma dos ângulos internos de um polígono

convexo de n lados (n ≥ 3) é igual a (n− 2)π radianos.

Solução. No caso de n = 3 a propriedade acima é muito bem conhecida.

Desde Tales de Mileto e Euclides se conhecia que a soma dos ângulos internos

de um triângulo é π radianos. Façamos mais um caso, tomando n = 4. Neste

caso, podemos dividir um quadrilátero em dois triângulos, como mostra a

Figura 6.1 (a). Assim, a soma dos ângulos internos de um quadrilátero é

2π radianos.

A1 A2

A3

A4

(a) A1 A2

A3

A4

A5 (b)Figura 6.1: Dividindo polígonos

Para elucidar o processo de indução e não deixar dúvidas sobre o que

iremos fazer, vamos considerar mais um polígono, o pentágono (n = 5).

Neste caso, para mostrar que a soma dos seus ângulos internos é (5−2)π =

3π radianos, iremos dividir o pentágono A1A2A3A4A5 em um quadrilátero

A1A2A3A4 e um triângulo A1A4A5, como mostra a Figura 6.1 (b). Assim,

Page 234: Resolução de problemas

6.3 Indução na Geometria 217

a soma dos ângulos internos do pentágono A1A2A3A4A5 é igual à soma dos

ângulos internos do triângulo A1A4A5 (igual a π) mais a soma dos ângulos

internos do quadrilátero A1A2A3A4 (igual a 2π), ou seja, é igual a 3π.

Finalmente, vamos assumir como hipótese de indução que para um certo

n ≥ 3 mostramos que a soma dos ângulos internos do n-ágono é dada pela

expressão (n − 2)π. Precisamos mostrar que a soma dos ângulos internos

de um n + 1-ágono é [(n + 1) − 2]π = (n − 1)π. De fato, podemos repetir

o processo anterior. Vamos denominar de A1, A2, . . . , An, An+1 os vértices

consecutivos do (n+1)-ágono. Podemos dividi-lo no n-ágono A1A2 . . . An e

no triângulo A1An+1An. Logo, a soma dos ângulos internos do (n+1)-ágono

é (n− 2)π + π = (n− 1)π.

Exemplo 6.13. Mostre que o número de diagonais de um polígono convexo

de n-lados é igual a n(n−3)2

.

Solução. Observe que para n = 3 temos que existem 0 = 3.(3 − 3)/2 dia-

gonais num triângulo. Para n = 4, temos 2 = 4(4 − 3)/2 diagonais num

quadrilátero convexo (veja a Figura 6.2).

Vamos agora assumir como hipótese de indução que se n é um n-

ágono convexo então o seu número de diagonais é n(n− 3)/2 e vamos pro-

var que a fórmula vale para um (n + 1)-ágono convexo. De fato, denote

por A1, A2, . . . , An, An+1 os vértices consecutivos do n + 1-ágono. Pode-

mos decompô-lo como a união do n-ágono A1, A2, . . . , An e do triângulo

A1, An, An+1. Neste caso, para contarmos as diagonais do (n + 1)-ágono

devemos considerar os seguintes casos:

• Diagonais do n-ágono A1, A2, . . . , An; por hipótese de indução, o nú-

mero dessas diagonais é n(n− 3)/2.

• n− 2 diagonais que partem do vértice An+1 mais a diagonal A1An.

Assim, o número total de diagonais do (n+ 1)-ágono é

n(n− 3)

2+(n−2)+1 =

n2 − 3n+ 2n− 2

2=n2 − n− 2

2=

(n+ 1)(n− 2)

2,

Page 235: Resolução de problemas

218 6 Indução Matemática

como queríamos demonstrar.

A1 A2

A3

A4

(a) A1 A2

A3

A4

A5 (b)Figura 6.2: Diagonais de polígonos

Exemplo 6.14. Mostre que podemos cobrir os n2 pontos no reticulado a

seguir traçando 2n− 2 segmentos de reta sem tirar o lápis do papel.

︸ ︷︷ ︸n×n−pontos

Figura 6.3: O problema de bar n× n

Page 236: Resolução de problemas

6.3 Indução na Geometria 219

Solução. O caso n = 3 já foi enunciado no Problema 1.12 do Capítulo 1. A

gura a seguir mostra a solução, onde o caminho realizado com as 4 linhas

é o seguinte: A→ B → C → D → B.

• • •

• • •

• • •A

B

C

D

Figura 6.4: Solução do problema de bar 3× 3

Daremos a prova do problema acima por indução. Para isso, veja que

podemos resolver o caso n = 4 continuando a solução do caso n = 3. Como

paramos num dos vértices do quadrado 3×3, acrescentamos mais uma linha

e uma coluna para obter um reticulado 4×4. Assim, conseguimos cobrir os

16 pontos utilizando 4+ 2 = 6 linhas, sem tirar o lápis do papel e cobrindo

dois lados do quadrado, como mostram as linhas descontínuas na Figura

6.5.

• • •

• • •

• • •

••••A

B

C

D

Figura 6.5: Completando o reticulado

Page 237: Resolução de problemas

220 6 Indução Matemática

Finalmente, vamos assumir como hipótese de indução que podemos co-

brir n ≥ 2 um reticulado n×n com 2n− 2 linhas, sendo que a última delas

cobre um dos lados do reticulado. Acrescentando 2n+1 pontos como mostra

a Figura 6.5, obtemos um reticulado (n+1)× (n+1) que pode ser coberto

com 2n− 2 + 2 = 2(n+ 1)− 2 pontos, como queríamos demonstrar.

6.4 Miscelânea

Nesta seção discutiremos alguns exemplos interessantes de como podemos

aplicar o método da indução aos mais variados tipos de problemas. O

primeiro deles é uma generalização do Problema 1.8.

Exemplo 6.15 (A Moeda Falsa). Um rei muito rico possui 3n moedas de

ouro. Porém, uma destas moedas é falsa e seu peso é menor que o peso das

demais. Com uma balança de 2 pratos e sem nenhum peso, mostre que é

possível encontrar a moeda falsa com apenas n pesagens.

Solução. Para resolver este problema, vamos utilizar o Método da Indução.

De fato, se n = 1, procederemos da seguinte forma: pegamos duas moedas

quaisquer e colocamos na balança, deixando uma do lado de fora. Caso a

balança se equilibre, a moeda que está do lado de fora é necessariamente a

que tem menor peso. Caso a balança se desequilibre, a que tem menor peso

está na balança, no prato mais alto. O caso n = 2 foi feito no Problema

1.8.

Vamos agora assumir como hipótese de indução que dadas 3n moedas,

podemos achar a moeda mais leve com n pesagens. Vamos mostrar que

para 3n+1 moedas, é suciente n + 1 pesagens. De fato, dividiremos as

3n+1 moedas em 3 grupos, A,B e C com 3n moedas cada. Colocamos na

balança os grupos A e B. Caso os dois grupos se equilibrem, a moeda mais

leve está no grupo C. Caso o grupo A esteja mais leve, a moeda mais leve se

encontra no grupo A. De qualquer modo, com uma pesagem conseguimos

Page 238: Resolução de problemas

6.4 Miscelânea 221

determinar em qual grupo de 3n elementos a moeda mais leve se encontra.

Por hipótese de indução, precisamos de mais n pesagens para encontrar a

moeda mais leve, totalizando n+1 pesagens. Desaamos o leitor a mostrar

que não é possível realizar tal tarefa com menos de n pesagens.

Exemplo 6.16. Mostre que utilizando um balde com 5 litros de capacidade

e outro com 7 litros, é possível separar qualquer quantidade superior ou igual

a 24 litros.

Solução. Novamente, faremos a prova utilizando o Método da Indução.

Neste caso, começaremos o processo de indução a partir de 24. De fato,

podemos separar 24 litros utilizando duas vezes o balde de 7 e duas vezes o

balde de 5 litros. Note que o problema acima equivale a mostrar que

Todo número maior ou igual a 24 pode ser escrito da forma

7x + 5y, onde x e y são números inteiros maiores ou iguais a

zero.

Neste caso, escrevemos 24 como 24 = 2 · 7 + 2 · 5. Por hipótese de

indução, vamos supor que conseguimos escrever um número n ≥ 24 como

n = 7x+5y, com x e y números inteiros maiores ou iguais a zero. Devemos

mostrar que n+ 1 se escreve deste modo também. Para isso, vamos dividir

a análise em dois casos:

Caso 1: y ≤ 3

Logo, x ≥ 2 pois se isso não ocorresse, teríamos 7x + 5y ≤ 22 < 24, o

que é impossível. Assim, podemos escrever:

n+ 1 = 7x+ 5y + 1 = 7(x− 2) + 5(y + 3),

pois x− 2 ≥ 0.

Caso 2: y ≥ 4

Page 239: Resolução de problemas

222 6 Indução Matemática

Neste caso, y − 4 ≥ 0. Logo, podemos escrever:

n+ 1 = 7x+ 5y + 1 = 7(x+ 3) + 5(y − 4),

nalizando a nossa prova por indução.

6.4.1 Cuidados ao Usar o Princípio da Indução

Observação 6.17. Quando aplicamos o princípio da indução devemos to-

mar certos cuidados. A seguir damos um exemplo de como o método pode

ser aplicado de forma errada. Vamos mostrar a seguinte armação:

Armação: Num conjunto qualquer de n bolas, todas as bolas

possuem a mesma cor.

Observe que nossa proposição é claramente falsa. Mas, mesmo assim, vamos

dar uma prova por indução.

Para n = 1, nossa proposição é verdadeira pois em qualquer conjunto

com uma bola, todas as bolas têm a mesma cor, pois só existe uma bola. As-

suma por hipótese de indução que a proposição é verdadeira para n e prove-

mos que a proposição é verdadeira para n+1. Ora, seja A = b1, . . . , bn, bn+1o conjunto com n + 1 bolas referido. Considere os subconjuntos de B e C

de A com n elementos, construídos como:

B = b1, b2, . . . , bn e C = b2, . . . , bn+1

Observe que ambos os conjuntos têm n elementos. Assim, as bolas

b1, b2, . . . , bn do conjunto B têm a mesma cor. Do mesmo modo, as bo-

las do conjunto C têm a mesma cor. Em particular, a bola bn tem a mesma

cor da bola bn+1. Assim, todas as bolas têm a mesma cor. Ache o erro no

argumento! Se você não conseguir, leia a nota de rodapé. 1

1Uma dica da solução encontra-se no nal do capítulo.

Page 240: Resolução de problemas

6.5 Indução e Recorrências 223

6.5 Indução e Recorrências

Vamos começar esta seção discutindo um problema muito conhecido e inte-

ressante.

Exemplo 6.18 (As Torres de Hanói2). Diz uma antiga lenda que na origem

dos tempos, em um templo de Hanói, foram colocados 64 discos perfurados

de ouro puro e de diâmetros diferentes ao redor de uma de três hastes de

diamante. Muitos sacerdotes moviam os discos, respeitando as seguintes

regras: eles começam empilhados em ordem crescente de acordo com seu

tamanho (ver Figura 6.6). Os discos podem ser deslocados de uma coluna

para qualquer outra, sendo que nunca pode ser colocado um disco maior em

cima de um menor e a cada segundo os sacerdotes movem um disco.

Quando os sacerdotes transportassem todos os discos de uma coluna para

outra, o mundo se acabaria. Suponha que eles começaram esse processo no

ano 2000 e que a lenda é verdadeira, quanto tempo ainda resta para a Terra?

Figura 6.6: Torre de Hanói

Para responder esse problema, consideraremos o problema geral de des-

cobrir quantos movimentos são necessários para mover n anéis de uma haste

para outra. Argumentaremos do seguinte modo: observe que podemos mover

os discos para outra haste se n = 1 ou 2. Com efeito, se temos somente um

anel basta mover este para qualquer outra haste com um único movimento.

2Este jogo foi inventado, em 1882, pelo matemático Francês Édouard Lucas.

Page 241: Resolução de problemas

224 6 Indução Matemática

Se temos 2 anéis então movemos o menor deles para a segunda haste, o

maior para a terceira haste e, nalmente, o menor para a terceira haste,

realizando um total de 3 movimentos. Para calcular o caso geral, vamos

empregar um método chamado de método recursivo: o número ak+1 de mo-

vimentos necessários para mover k+1 anéis será expresso como uma função

de ak.

De fato, se temos k+1 anéis na primeira haste e sabemos mover k anéis

de uma haste para outra utilizando ak movimentos, então podemos mover

todos os k + 1 anéis para a segunda haste usando 2ak + 1 movimentos.

De fato, movemos todos eles, exceto o maior, para a terceira haste usando

ak movimentos. A seguir, colocamos o maior na segunda haste usando 1

movimento. Imediatamente, deslocamos todos os anéis da terceira haste

para a segunda haste usando mais ak movimentos. Logo, movemos todos os

k + 1 anéis utilizando 2ak + 1 movimentos. Em resumo:

ak+1 = 2ak + 1, (6.5)

onde ak é o número de movimentos necessários para mover k discos de uma

haste para outra. Vamos agora usar indução para provar que ak = 2k − 1.

Uma vez constatada a veracidade da armação para k = 1, 2, para

calcular ak, por hipótese de indução, vamos assumir que ak = 2k−1. Temos

pela equação (6.5):

ak+1 = 2ak + 1 = 2(2k − 1)− 1 = 2k+1 − 1.

como queríamos demonstrar.

Vamos aproveitar o Exemplo 6.18 para discutir algumas equações que

aparecem em muitas situações em Matemática: as equações de recorrência.

Em geral, uma equação de recorrência é uma equação envolvendo uma

certa quantidade de termos de sequência xn. Para ilustrar isso, observe

a equação (6.5). Aqui, estaremos interessados em um tipo particular de

equação de recorrência, as equações de recorrência lineares.

Page 242: Resolução de problemas

6.5 Indução e Recorrências 225

Denição 6.19. Uma equação de recorrência linear de grau k é uma ex-

pressão da forma:

xn+1 =rk−1xn + rk−2xn−1 + · · ·+ r0xn−k+1

x1 = a1, x2 = a2, . . . , xk = ak,(6.6)

onde r0, r1, . . . , rk−1 são números reais e r0 6= 0.

Por exemplo, são equações de recorrência lineares as seguintes equações

2xn − 3xn+1 = 0 e − 3xn +2

3xn+1 = 5xn+2

e não são equações de recorrência lineares as equações

2(xn)3 − 5xn+1 = 0 e − 3xn +

2

3xn+1 = 5xn+2 + 3.

Exemplo 6.20 (Sequência de Fibonacci). Um exemplo muito interessante

de equação de recorrência é a sequência conhecida por sequência de Fibo-

nacci, devido ao matemático italiano Leonardo di Pisa (1170-1250). Esta

sequência adquiriu muita fama devido a suas conexões com áreas das mais

variadas na cultura humana. Ela aparece em problemas de Biologia, Ar-

quitetura, Engenharia, Física, Química e muitos outras áreas da ciência e

arte.

Denimos a sequência de Fibonacci como sendo a sequência Fn que

satisfaz a seguinte equação de recorrência:

F1 = 1;

F2 = 1;

Fn = Fn−1 + Fn−2, se n ≥ 3.

Agora vamos utilizar indução para mostrar algumas de suas proprieda-

des.

Page 243: Resolução de problemas

226 6 Indução Matemática

Exemplo 6.21. Considere Fn a sequência de Fibonacci. Mostre que

Fn <

(7

4

)n.

Solução. Denamos a proposição p(n) := Fn <(74

)n. Para n = 1 temos

que F1 = 1 < 74 , de modo que p(1) é verdadeira. Suponhamos que

p(1), p(2), . . . , p(n),∀n ≥ 2,

sejam todas verdadeiras. Mostraremos que Fn+1 <(74

)n+1. Com efeito,

Fn+1 = Fn + Fn−1 <(74

)n+(74

)n−1

< 74

(74

)n−1+(74

)n−1

<(1 + 7

4

) (74

)n−1.

Como(1 + 7

4

)<(74

)2, segue-se que Fn+1 <

(74

)2 (74

)n−1. Portanto,

Fn+1 <(74

)n+1.

Exemplo 6.22. Dada a seguinte relação de recorrência

a0 = 8;

a1 = 10;

an = 4an−1 − 3an−2, ∀n ≥ 2.

Mostre que an = 7 + 3n, para todo n ∈ Z+.

Solução. Denamos a proposição P (n) : an = 7 + 3n. P (0) é verdadeira,

pois P (0) = 7 + 30 = 7 + 1 = 8. Suponhamos que P (k) é verdadeiro para

cada inteiro k tal que 1 ≤ k ≤ n. Vamos mostrar que P (k) é verdade para

Page 244: Resolução de problemas

6.5 Indução e Recorrências 227

k = n+ 1. Com efeito,

an+1 = 4an − 3an−1

= 4(7 + 3n)− 3(7 + 3n−1)

= 7 + 4× 3n − 3× 3n−1

= 7 + 3n−1(4× 3− 3

)

= 7 + 3n−1(9)= 7 + 3n−1 × 32

= 7 + 3n+1.

Vamos agora discutir o caso geral da equação de recorrência linear (6.6).

Para isso, vamos fazer algumas observações preliminares que deixaremos a

cargo do leitor:

• se an e bn são soluções da equação (6.6), então an + bn também é

solução;

• se an é solução da equação (6.6) e α é um número real, então αantambém é solução.

Com isto em mente, vamos descrever agora como obter todas as soluções

xn da equação (6.6) em função de n. Observe que dados os termos iniciais

a1, a2, . . . , ak a sequencia xn ca inteiramente determinada pela equação de

recorrência. O interessante aqui é determinar o termo xn+1 sem que seja

preciso o cálculo dos termos xn, xn−1, . . . , xn−k+1.

Vamos primeiro procurar o que se chama de solução particular da equa-

ção (6.6). Particular porque ela assume uma forma característica e porque

não assumiremos que as condições x1 = a1, . . . , xk = ak valham.

Vamos procurar soluções do tipo xn = λn, onde λ é um número real

positivo. Neste caso, temos que:

λn+1 = xn+1 =rk−1xn + rk−2xn−1 + · · ·+ r0xn−k+1

= rk−1λn + rk−2λ

n−1 + · · ·+ r0λn−k+1.

Page 245: Resolução de problemas

228 6 Indução Matemática

Passando os termos do lado direito da igualdade e colocando em evi-

dência o termo λn−k+1 temos:

λn−k+1(λk − rk−1λk−1 − rk−2λk−2 − · · · − r0

)= 0. (6.7)

Assim, como λk 6= 0, pois λ > 0, temos que

λk − rk−1λk−1 − rk−2λk−2 − · · · − r0 = 0. (6.8)

O polinômio

p(λ) = λk − rk−1λk−1 − rk−2λk−2 − · · · − r0

recebe o nome especial de polinômio característico da equação de recorrên-

cia (6.6). Acabamos de mostrar que qualquer raiz do polinômio caracterís-

tico gera uma solução particular da equação (6.6).

Vamos assumir que a equação (6.8) possui k raízes diferentes, digamos

λ1 > λ2 > · · · > λk. Então vale o seguinte teorema:

Teorema 6.23. Se escolhermos números reais c1, c2, . . . , ck, então

xn = c1λn1 + c2λ

n2 + · · ·+ ckλ

nk (6.9)

é uma solução da equação de recorrência, onde os termos iniciais ai para

i = 1, 2, . . . , k são:

ai = c1λi1 + c2λ

i2 + · · ·+ ckλ

ik.

Demonstração. Para mostrar o teorema, como x1 = a1, . . .xk = ak pela

denição dos ai's, basta mostrar que xn é uma solução.

Ora, o produto de uma solução por um número real também é uma

solução. Assim, como λni é uma solução para i = 1, 2, . . . , k e ci é um

Page 246: Resolução de problemas

6.6 Exercícios 229

número real, temos que ciλni é solução para i = 1, 2, . . . , k. Como já vimos

acima, a soma de soluções é também uma solução. Logo,

xn = c1λn1 + c2λ

n2 + · · ·+ ckλ

nk

é uma solução.

Neste ponto, voltamos a equação (6.6). Desde o princípio, dados os

números ai buscávamos a solução xn tal que x1 = a1, . . . , xk = ak. A

Equação (6.9) nos dá uma variedade de soluções, onde podemos escolher

os números ci como bem entendermos. Usando equações lineares, podemos

mostrar que sempre é possível escolher os números ci de modo que x1 =

a1, . . . , xk = ak. Isso encerra nossa busca. Para complementar esta seção,

recomendamos a leitura do Capítulo 3 de [4].

6.6 Exercícios

1. Se qn denota a soma qn = 12 + 22 + · · · + n2, prove que para todo

n ∈ Nqn =

n(n+ 1)(2n+ 1)

6.

2. Use o princípio da indução para provar as seguintes armações:

(a) 3n+1 + 2n+2 é divisível por 7 para todo n ∈ N;

(b) a soma dos cubos de três números naturais consecutivos é divi-

sível por 9;

(c) 7 + 77 + 777 + · · ·+ 777 . . . 7︸ ︷︷ ︸n−vezes

= 7(10n+1 − 9n− 10)/81;

(d) (n+ 1)(n+ 2) . . . (n+ n) = 2n · 1 · 3 · 5 · · · (2n− 1).

3. Use o princípio da indução para provar as seguintes desigualdades:

Page 247: Resolução de problemas

230 6 Indução Matemática

(a) 2n−1(an + bn) > (a + b)n, ∀n ∈ N, com a, b ∈ R, a + b > 0 e

a 6= b;

(b)1√1+

1√2+

1√3+ · · ·+ 1√

n>√n, para todo n ∈ N;

(c)1

n+ 1+

1

n+ 2+

1

n+ 3+ · · ·+ 1

2n>

13

24, para todo n ∈ N.

4. Mostre a seguinte identidade trigonométrica

cosx+2 cos 2x+ · · ·+n cosnx =(n+ 1) cosnx− n cos(n+ 1)x− 1

4 sin2 x2.

5. Um torneio de xadrez tem n jogadores. Cada jogador joga uma única

partida com cada um dos outros jogadores. Calcule o número total

de partidas realizadas no torneio.

6. Demonstre que para qualquer n ∈ N é válida a igualdade

13 + 23 + 33 + · · ·+ n3 =

[n(n+ 1)

2

]2.

7. Demonstre que para qualquer n ∈ N é valida desigualdade

an =

(1 +

1

n

)n< 3.

8. Prove que, para todo n ∈ N e a > 0,√

a+

√a+

√a+ · · ·+√a

︸ ︷︷ ︸n−radicais

<1 +√4a+ 1

2.

9. Mostre que para qualquer número natural n ≥ 0, 11n+2 + 122n+1 é

sempre divisível por 133.

10. Mostre que para todo n ∈ Z+ temos que 32n+1 +2n+2 é um múltiplo

de 7.

Page 248: Resolução de problemas

6.6 Exercícios 231

11. Mostre que para todo n ∈ Z+ temos que 32n+2+26n+1 é um múltiplo

de 11.

12. Considere Fn a sequência de Fibonacci . Mostre que

Fn =1√5

(1 +√5

2

)n− 1√

5

(1−√5

2

)n.

13. Mostre as seguintes propriedades a respeito da sequência de Fibonacci

Fn:

(a)n∑

i=1

Fi = Fn+2 − 1; (b)n∑

i=1

F2i−1 = F2n;

(c)n∑

i=1

F2i = F2n+1 − 1; (d) Fn−1Fn+1 − F 2n = (−1)n.

14. De quantas formas diferentes podemos cobrir um tabuleiro de 2 ×n com peças de dominós que cobrem exatamente duas celas do ta-

buleiro?

15. Calcular o número de regiões em que o plano é dividido por n retas

distintas em cada uma das seguintes situações:

(a) as n retas são concorrentes;

(b) não existem duas retas paralelas nem três retas concorrentes.3

16. Dizemos que uma gura é enquadrável com régua e compasso, se a

partir dela é possível, utilizando apenas régua e compasso, construir

um quadrado de mesma área. Prove que:

3Até onde sabemos, este problema é conhecido como a pizza de Steiner, o qual

foi resolvido, em 1826, pelo notável geômetra Jacob Steiner (1796-1863).

Page 249: Resolução de problemas

232 6 Indução Matemática

(a) um triângulo é sempre enquadrável;

(b) um polígono qualquer é enquadrável.

Sugestão para o item (b): Utilize indução dividindo a gura em tri-

ângulos.

17. Dê uma resposta à situação á Observação 6.17.

Sugestão: Observe a validade do argumento quando o conjunto A tem

2 elementos. Veja que B e C não se intersectam. Ou seja, o passo

indutivo falha de n = 1 para n = 2.

Page 250: Resolução de problemas

Referências Bibliográcas

[1] AIGNER, M. e ZIEGLER, G. (2002). As Provas estão no

Livro. Edgard Blücher.

[2] GARCIA, A. e LEQUAIN, I. (2003). Elementos de Álgebra.

Projeto Euclides, IMPA.

[3] LIMA, E. L.; CARVALHO, P. C. P.; WAGNER, E. e MOR-

GADO, A.C. (2004). A Matemática do Ensino Médio. Volume

1. Sociedade Brasileira de Matemática.

[4] LIMA, E.L.; CARVALHO, P. C. P.; WAGNER, E. e MOR-

GADO, A.C. (2004). A Matemática do Ensino Médio. Volume

2. Sociedade Brasileira de Matemática.

[5] LIMA,E.L.; CARVALHO,P. C. P.; WAGNER,E. e MOR-

GADO,A.C. (2004). A Matemática do Ensino Médio. Volume

3. Sociedade Brasileira de Matemática.

[6] LIMA, E.L.; CARVALHO, P. C. P.; WAGNER,E. e MOR-

GADO, A.C. (2001). Temas e Problemas. Sociedade Brasileira

de Matemática.

[7] LIMA, E.L. (2001). Álgebra Linear. Sociedade Brasileira de

Matemática.

285

Page 251: Resolução de problemas

286 REFERÊNCIAS BIBLIOGRÁFICAS

[8] MORAIS FILHO, D. C. (2007). Um Convite à Matemática.

EDUFCG.

[9] MORGADO, A.; CARVALHO, J.; CARVALHO, P.; FER-

NANDEZ, P. (1991). Análise Combinatória e Probabilidade .

Sociedade Brasileira de Matemática.

[10] RIBENBOIM, P. (2001). Números Primos: Mistérios e Re-

cordes. Sociedade Brasileira de Matemática.

[11] SANTOS, J. P. O. (1993) Introdução à Teoria dos Números.

IMPA.

[12] SANTOS, J. P. O.; MELLO, M. P. e MURARI, I. T. C.

(2006). Introdução à Análise Combinatória. Editora Unicamp.

[13] SOARES, M. G. (2005). Cálculo em uma Variável Complexa.

Sociedade Brasileira de Matemática.

Page 252: Resolução de problemas

Mestrado Profissional

em Matemática em Rede Nacional

Iniciação à Matemática

Autores:

Krerley Oliveira Adán J. Corcho

Unidade IV:

Capítulos VII e VIII

Page 253: Resolução de problemas

7

Desigualdades

Existem duas formas de fazer ótima Matemáti a. A primeira é sermais esperto que todo mundo. A segunda é ser mais estúpido quetodo mundo mas persistente. Raoul BottNeste capítulo estudaremos algumas desigualdades clássicas que

são usadas frequentemente na resolução de problemas matemáticos,

sendo estas aplicadas em contextos que variam desde o nível mais

simples até o mais complexo.

Uma vez que uma inequação em uma ou mais variáveis é resolvida,

o resultado dá lugar a uma desigualdade que é válida para um certo

conjunto de valores. Alguns exemplos simples de desigualdades são os

seguintes:

(a) x ≤ |x|, para qualquer −1 < x < 1;

(b) x2 < x, se x < 1;

(c) (x− y)2 ≥ 0, para quaisquer x e y reais;

(d) xy

+ yx≥ 2, para quaisquer x, y > 0.

233

Page 254: Resolução de problemas

234 7 Desigualdades

7.1 Desigualdade Triangular

A desigualdade triangular arma o seguinte

Teorema 7.1 (Desigualdade Triangular). Dado um triângulo ABC o

comprimento de um dos lados é sempre inferior à soma dos compri-

mentos dos outros dois lados, ou seja,

AB < AC + CB, AC < AB +BC e BC < BA+ AC.

A B

C

Figura 7.1: Desigualdade Triangular

Em outras palavras, a desigualdade triangular é a formulação ma-

temática da ideia intuitiva de que o caminho reto é mais curto entre

os pontos A e B.

Em analogia com a geometria plana temos uma versão da desigual-

dade triangular para números reais, que provamos a seguir.

Proposição 7.2. Sejam a e b números reais quaisquer, então

|a+ b| ≤ |a|+ |b|.

Demonstração. Se a + b ≥ 0, então |a + b| = a + b ≤ |a| + |b|. Caso

contrário, se a+ b < 0, então |a+ b| = −a− b ≤ |a|+ |b|.

Page 255: Resolução de problemas

7.1 Desigualdade Triangular 235

Corolário 7.3. As seguintes desigualdades valem

|a− b| ≤ |a|+ |b| (7.1)

|a− b| ≥ |a| − |b|, (7.2)

|a− b| ≥∣∣|a| − |b|∣∣ (7.3)

Demonstração. Para a primeira, escrevemos |a − b| = |a + (−b)| ≤|a| + | − b| = |a| + |b|. A segunda desigualdade decorre de |a| =

|b+ (a− b)| ≤ |b|+ |a− b|. A última desigualdade é consequência da

segunda, trocando os papéis de a e b.

A

B

C

D

O•

•P

Figura 7.2: Problema da central de energia

Exemplo 7.4. Quatro cidades rurais, A, B, C e D, estão situadas

geogracamente formando um quadrilátero convexo. Deseja-se cons-

truir uma central de distribuição de energia para as quatro cidades de

modo que a soma total das distâncias da central a cada uma das quatro

cidades seja a mínima possível. Onde deverá ser construída a central?

Solução. Mostraremos que a central de energia deverá ser colocada

no ponto O de intersecção das diagonais do polígono ABCD. Com

Page 256: Resolução de problemas

236 7 Desigualdades

efeito, considerando um ponto P , diferente de O, (veja Figura 7.2) a

desigualdade triangular nos garante que

OA+OC = AC < PA+ PC

e

OB +OD = BP < PB + PD,

de onde se segue que

OA+OC +OB +OD < PA+ PC + PB + PD,

como esperávamos.

Exemplo 7.5. Duas torres de alturas h1 e h2, respectivamente, estão

separadas a uma distância d. As torres são amarradas por uma corda

APB que vai do topo A da primeira torre para um ponto P no chão,

entre as torres, e então até o topo B da segunda torre, como na Figura

7.3. Qual a posição do ponto P que nos dá o comprimento mínimo da

corda a ser utilizada?

A

B

P

Figura 7.3: Problema das Torres

Page 257: Resolução de problemas

7.1 Desigualdade Triangular 237

Solução. Imaginemos que a superfície do chão é um espelho e que re-

etimos o ponto através deste, obtendo assim o ponto B′ como mostra

a Figura 7.4.

A

B

PC D

B′

P ′

Figura 7.4: Solução geométrica do problema das torres

Consideremos o segmento AB′ que intercepta o chão no ponto P

e para nossa surpresa vericaremos que este é o ponto que nos dá o

comprimento mínimo das cordas. Com efeito, suponhamos que existe

outro P ′ situado entre as torres que nos dá um comprimento menor

para a corda, então da Figura 7.4 é fácil ver que os triângulos BPD

e B′PD são congruentes, assim como os triângulos BP ′D e B′P ′D

também são congruentes. Logo, as seguintes igualdades seguem dire-

tamente das congruências:

BP = B′P e BP ′ = B′P ′.

Agora, usando a desigualdade triangular no triângulo AB′P ′ e as igual-

dades acima, temos que

AP ′ + P ′B = AP ′ + P ′B′ ≥ AB′ = AP + PB′ = AP + PB,

Page 258: Resolução de problemas

238 7 Desigualdades

chegando assim à conclusão de que AP + PB nos oferece o compri-

mento mínimo desejado.

Agora calcularemos a que distância está P da base D. Lembremos

que AC = h1, BD = h2 e CD = d e observamos que

tang(]BPD) =h2PD

=h1

d− PD.

Daí tem-se PD =dh2

h1 + h2.

7.2 Desigualdade das Médias

Denição 7.6. Sejam a1, a2, . . . , an−1 e an números reais positivos.

As quantidades

mh(a1, a2, . . . , an) =n

1/a1 + 1/a2 + · · ·+ 1/an, (7.4)

mg(a1, a2, . . . , an) = n√a1a2 · · · an, (7.5)

ma(a1, a2, . . . , an) =a1 + a2 + · · ·+ an

n, (7.6)

mq(a1, a2, . . . , an) =

√a21 + a22 + · · ·+ a2n

n(7.7)

são chamadas, respectivamente, de média harmônica, média geomé-

trica, média aritmética e média quadrática dos números ai, i = 1, 2, . . . , n.

A seguir provaremos alguns resultados que estabelecem relações de

desigualdades entre as médias denidas acima.

Page 259: Resolução de problemas

7.2 Desigualdade das Médias 239

Proposição 7.7 (Desigualdade das Médias Aritmética e Quadrática).

Dados a1, a2, . . . , an números reais positivos tem-se

a1 + a2 + · · ·+ ann

≤√a21 + a22 + · · ·+ a2n

n,

ou seja, ma(a1, a2, . . . , an) ≤ mq(a1, a2, . . . , an). Além disso, a igual-

dade vale se, e somente se, a1 = a2 = · · · = an.

Demonstração. Usando a igualdade∑1≤i<j≤n

(ai − aj)2 = (n− 1)n∑i=1

a2i − 2∑

1≤i<j≤naiaj (7.8)

concluímos que,

2∑

1≤i<j≤naiaj ≤ (n− 1)

n∑i=1

a2i , (7.9)

dado que o termo da esquerda em (7.8) é não negativo. Somando em

ambos os membros de (7.9) a quantidaden∑i=1

a2i obtemos

( n∑i=1

ai

)2≤ n

n∑i=1

a2i ,

donde, dividindo por n2 e tomando a raiz quadrada, segue-se a desi-

gualdade desejada. Por último, observamos que a igualdade em (7.9)

é atingida se, e somente se,∑

1≤i<j≤n(ai − aj)2 = 0, o que é verdade se,

e somente se, a1 = a2 = · · · = an.

Proposição 7.8 (Desigualdade das médias Geométrica e Aritmética).

Dados a1, a2, . . . , an números reais positivos tem-se

n√a1a2 · · · an ≤

a1 + a2 + · · ·+ ann

,

Page 260: Resolução de problemas

240 7 Desigualdades

ou seja, mg(a1, a2, . . . , an) ≤ ma(a1, a2, . . . , an). Além disso, a igual-

dade vale se, e somente se, a1 = a2 = · · · = an.

Demonstração. A prova desta desigualdade é mais técnica e exige um

pouco mais de esforço. Dividiremos a mesma em dois passos.

Passo 1. A desigualdade vale para n = 2m.

Procederemos por indução. Para n = 2 a desigualdade vale. De

fato,

(√a1 −

√a2)

2 = a1 + a2 − 2√a1a2 ≥ 0.

Assim, a1 + a2 ≥ 2√a1a2 e conseqüentemente a1+a2

2≥ √a1a2.

Agora provamos que se a desigualdade vale para n = k, então

também vale para n = 2k. Com efeito,

a1 + · · ·+ a2k2k

=a1+···+ak

k+ ak+1+···+a2k

k

2(1)

≥k√a1 · · · ak + k

√ak+1 · · · a2k

2(2)

≥√

k√a1 · · · ak k

√ak+1 · · · a2k

= 2k√a1 · · · a2k,

onde em (1) e (2) usamos a validade da desigualdade em para n = k

e para n = 2, respectivamente. Logo, como já provamos a validade

para n = 2, é claro que vale também para n = 4, 8, . . . , 2m, . . . , como

esperávamos.

Passo 2. Dado m inteiro positivo, então a desigualdade vale para

todo n < 2m.

Para vericar isto, denimos o número

L = n√a1 · · · an,

Page 261: Resolução de problemas

7.2 Desigualdade das Médias 241

e como a desigualdade vale para n = 2m, temos então que

a1 + · · ·+ an + L+ · · ·+ L︸ ︷︷ ︸2m−n vezes

2m≥ 2m

√a1 · · · an · L2m−n

=2m√Ln · L2m−n = L.

Portanto,a1 + · · ·+ an + (2m − n)L

2m≥ L,

logo

a1 + · · ·+ an ≥ 2mL− (2m − n)L = nL,

obtendo assim que

a1 + · · ·+ an ≥ nL = n n√a1 · · · an,

o que nos dá a desigualdade desejada.

Como para qualquer inteiro positivo n sempre existe um inteiro

positivo m tal que n < 2m, a desigualdade ca provada para todo n.

A prova de que a igualdade só ocorre quando a1 = a2 = · · · = an

pode também ser feita por indução e deixamos a cargo do leitor.

Proposição 7.9 (Desigualdade das Médias Harmônica e Geométrica).

Dados a1, a2, . . . , an números reais positivos tem-sen

1/a1 + 1/a2 + · · ·+ 1/an≤ n√a1a2 · · · an,

ou seja, mh(a1, a2, . . . , an) ≤ mg(a1, a2, . . . , an). Além disso, a igual-

dade vale se, e somente se, a1 = a2 = · · · = an.

Demonstração. Usando a Proposição 7.8 com os números ai substituí-

dos por 1/ai (i = 1, 2 . . . , n) vale que( n∏i=1

1

ai

)1/n= mg(1/a1, . . . , 1/an) ≤ ma(1/a1, . . . , 1/an) =

1

n

n∑i=1

1

ai.

Page 262: Resolução de problemas

242 7 Desigualdades

Invertendo esta última desigualdade, obtemos então

mh(a1, a2, . . . , an) ≤ mg(a1, a2, . . . , an),

concluindo-se assim a prova. Notemos que as igualdades só ocorrem

se 1/a1 = 1/a2 = · · · = 1/an equivalem as igualdades a1 = a2 = · · · =an.

O próximo resultado resume as relações provadas, nas proposições

7.7, 7.8 e 7.9, para as médias mh, mg, ma e mq.

Teorema 7.10 (Desigualdade das Médias). Para toda coleção de nú-

meros reais positivos a1, a2, . . . , an−1 e an se vericam as seguintes

desigualdades:

min(a1, . . . , an) ≤ mh(a1, a2, . . . , an)

≤ mg(a1, a2, . . . , an)

≤ ma(a1, a2, . . . , an)

≤ mq(a1, a2, . . . , an) ≤ max(a1, . . . , an).

(7.10)

Além disso, em cada caso a igualdade ocorre se, e somente se, a1 =

a2 = · · · = an.

Exemplo 7.11. Num triângulo retângulo a altura relativa à hipote-

nusa é sempre menor ou igual que a metade da hipotenusa. Além

disso, a igualdade só ocorre quando o triângulo retângulo é isósceles

(ou seja, seus catetos são iguais).

Solução. Usando a Figura 7.5, temos que a hipotenusa c é dada por

c = x+ y e usando o teorema das alturas para um triângulo retângulo

Page 263: Resolução de problemas

7.2 Desigualdade das Médias 243

temos que h2 = xy, logo h =√xy. A desigualdade entre as médias

geométrica e aritmética nos dá que

h =√xy ≤ x+ y

2=c

2,

como queríamos. Além disso, a altura é a metade da hipotenusa se, e

x yc

ha b

Figura 7.5: Interpretação geométrica da desigualdade das médias geo-

métrica e aritmética

somente se, a igualdade entre as médias ocorre, ou seja, quando x = y.

Então, os catetos a e b do triângulo são iguais, sendo este isósceles.

Exemplo 7.12 (Desigualdade Isoperimétrica para Triângulos). O pe-

rímetro de um triângulo de lados a, b e c é a soma p = a + b + c.

Entre todos os triângulos com perímetro xado p o de maior área é o

triângulo equilátero.

Solução. Usando a Fórmula de Herón temos que a área de um triân-

gulo com perímetro p é dada pela expressão

A =√

p2(p2− a)(p

2− b)(p

2− c),

onde a, b e c são os lados do triângulo.

Usando agora a desigualdade mg ≤ ma temos que,

A ≤√

p2

( p2−a+p

2−b+p

2−c

3

)3

=p2

12√

3.

Page 264: Resolução de problemas

244 7 Desigualdades

Logo a maior área possível é p2

12√3, a qual é atingida quando

p2− a = p

2− b = p

2− c⇔ a = b = c,

ou seja, quando o triângulo é equilátero. Notemos que neste caso,p2

12√3

= a2√3

4.

Exemplo 7.13 (Desigualdade Isoperimétrica para Paralelepípedos).

Entre todos os paralelepípedos com área lateral xada A o de maior

volume é o cubo (ou seja, o paralelepípedo com todos seus lados iguais).

a

b

c

Figura 7.6: A área lateral de um paralelepípedo de lados a, b e c é dada

por AL = 2(ab+ bc+ ac).

Solução. Denotando por a, b e c as medidas das arestas do paralele-

pípedo sabemos que é a soma das áreas de todas as faces do paralele-

pípedo, ou seja,

AL = 2(ab+ ac+ bc).

Sendo V o volume do paralelepípedo e usando a desigualdade entre as

médias aritmética e geométrica temos que

V 2 = ab · ac · bc ≤(ab+ ac+ bc

3

)3

=

(AL6

)3

. (7.11)

Assim, o maior volume possível é V =

√(AL

6

)3, obtido quando ab =

ac = bc, consequentemente a = b = c.

Page 265: Resolução de problemas

7.3 Desigualdade de Cauchy-Schwarz 245

7.3 Desigualdade de Cauchy-Schwarz

Teorema 7.14 (Desigualdade de Cauchy-Schwarz). Dados a1, . . . ,

an e b1, . . . , bn números reais tem-se

|a1b1 + · · ·+ anbn| ≤√x21 + · · ·+ a2n

√b21 + · · ·+ b2n (7.12)

Além disso, a igualdade só ocorre se existir um número real α, tal que

a1 = αb1, . . . , an = αbn ou b1 = αa1, . . . , bn = αan.

Demonstração. Usando a identidade de Lagrange:

n∑i=1

a2i

n∑i=1

b2i =( n∑i=1

aibi

)2+

∑1≤i<j≤n

(aibj − ajbi)2

temos que ( n∑i=1

aibi

)2≤

n∑i=1

a2i

n∑i=1

b2i ,

de onde se obtém diretamente a desigualdade de Cauchy-Schwarz.

Além disso, a igualdade ocorre se, e somente se,∑1≤i<j≤n

(aibj − ajbi)2 = 0⇐⇒ aibj − ajbi = 0, 1 ≤ i < j ≤ n,

o que é verdade se, e somente se, existe α tal que ai = αbi ou bi = αai,

com i = 1, 2, . . . , n.

Exemplo 7.15. Entre todos os triângulos retângulos de catetos a e b

e hipotenusa c xada, o que tem maior soma dos catetos s = a + b é

o triângulo isósceles.

Solução. Usando a desigualdade de Cauchy-Schwarz temos que

a+ b = a · 1 + b · 1 ≤√a2 + b2

√12 + 12 = c

√2

Page 266: Resolução de problemas

246 7 Desigualdades

e este máximo é atingido quando a = λ · 1 e b = λ · 1 ou 1 = λ · a e

1 = λ · b. Em qualquer caso devemos ter a = b.

Exemplo 7.16 (Desigualdade de Minkowski). Dados ai, bi com 1 ≤i ≤ n, números reais, tem-se√√√√ n∑

i=1

(ai + bi)2 ≤

√√√√ n∑i=1

a2i +

√√√√ n∑i=1

b2i .

Solução. Partimos da seguinte igualdade:

n∑i=1

(ai + bi)2 =

n∑i=1

a2i +n∑i=1

b2i + 2n∑i=1

aibi. (7.13)

Aplicando a desigualdade de Cauchy-Schwarz no lado direito de (7.13)

temos que

n∑i=1

(ai + bi)2 ≤

n∑i=1

a2i +n∑i=1

b2i + 2

√√√√ n∑i=1

a2i

√√√√ n∑i=1

b2i

=

√√√√ n∑i=1

a2i +

√√√√ n∑i=1

b2i

2

.

(7.14)

Tomando raiz quadrada em ambos os membros de (7.14) obtemos a

desigualdade de Minkowski.

7.4 Desigualdade de Jensen

A Desigualdade de Jensen está estreitamente relacionada com o con-

ceito de convexidade, o qual explicamos a seguir.

Page 267: Resolução de problemas

7.4 Desigualdade de Jensen 247

Denição 7.17. Uma função f : [α, β] → R é dita convexa se para

quaisquer a, b ∈ [α, β] e para todo λ ∈ [0, 1] satisfaz

f(λa+ (1− λ)b

)≤ λf(a) + (1− λ)f(b).

x

y

a b

(a, f(a))

(b, f(b))

y = f(x)

Figura 7.7: Gráco de uma função convexa

Geometricamente, a denição de convexidade signica que para

cada par de pontos a e b escolhidos no intervalo [α, β] o gráco da

função encontra-se abaixo do segmento de reta secante que junta os

pontos (a, f(a)) e (b, f(b)), como mostra a Figura 7.7.

Exemplo 7.18. A função f(x) = x2 é convexa em qualquer intervalo

[α, β].

Solução. Sejam a, b ∈ [α, β] e suponhamos, sem perda de generalidade,

Page 268: Resolução de problemas

248 7 Desigualdades

que a < b. Então, para todo λ ∈ [0, 1] valem as desigualdades:

(λa+ (1− λ)b)2 = λ2a2 + (1− λ)2b2 + 2λ(1− λ)ab

(1)

≤ λ2a2 + (1− λ)2b2 + λ(1− λ)(a2 + b2) (7.15)

= a2[λ2 + λ(1− λ)] + b2[(1− λ)2 + λ(1− λ)]

= λa2 + (1− λ)b2,

onde na passagem (1) usamos a desigualdade ab ≤ a2+b2

2.

Exemplo 7.19. A função f(x) = 1/x é convexa em qualquer intervalo

[α, β] com α positivo.

Solução. Sendo a, b ∈ [α, β] com a < b, para todo λ ∈ [0, 1] tem-se

1 = (λ+ (1− λ))2

= λ2 + 2λ(1− λ) + (1− λ)2

(1)

≤ λ2 +(ab

+b

a

)λ(1− λ) + (1− λ)2 (7.16)

= λ2 +a

bλ(1− λ) +

b

aλ(1− λ) + (1− λ)2

= λa(λa

+(1− λ)

b

)+(1− λ)b

(λa

+(1− λ)

b

)=(λa+ (1− λ)b

)(λa

+(1− λ)

b

)onde na passagem (1) usamos que a/b + b/a ≥ 2 para quaisquer nú-

meros positivos a e b. De (7.16) segue-se que

1

λa+ (1− λ)b≤ λ

1

a+ (1− λ)

1

b,

mostrando isto a convexidade da função 1x.

Page 269: Resolução de problemas

7.4 Desigualdade de Jensen 249

Observemos que, usando a desigualdade entre as médias aritmética

e quadrática obtemos(a1 + a2 + · · ·+ an

n

)2

≤ a21 + a22 + · · ·+ a2nn

,

em outras palavras

(ma(a1, a2, . . . , an))2 ≤ ma(a21, a

22, . . . , a

2n). (7.17)

Por outro lado, a desigualdade entre as médias harmônica e aritmética

nos garantem que

1

ma(a1, a2, . . . , an)≤ ma(1/a1, 1/a2 . . . , 1/an). (7.18)

O seguinte resultado garante que as propriedades (7.17) e (7.18), satis-

feitas pelas funções convexas x2 e 1x, são válidas para qualquer função

convexa.

Teorema 7.20 (Desigualdade de Jensen). Seja f : [α, β] → R uma

função convexa e sejam λi ∈ [0, 1] (i = 1, . . . , n) tais quen∑i=0

λi = 1.

Então, para quaisquer ai ∈ [α, β] (i = 1, . . . , n) vale

f(λ1a1 + · · ·+ λnan) ≤ λ1f(a1) + · · ·+ λnf(an). (7.19)

Observação 7.21. Observemos que, quando λ1 = λ2 = · · · = λn =

1/n, a desigualdade de Jensen nos diz que

f(a1 + a2 + · · ·+ an

n

)≤ f(a1) + f(a2) + · · ·+ f(an)

n,

ou seja, f(ma(a1, . . . , an)) ≤ ma(f(a1), . . . , f(an)).

Page 270: Resolução de problemas

250 7 Desigualdades

Demonstração. Faremos a prova por indução. Para n = 2 a validade

decorre diretamente da denição. Suponhamos que dado n natural

(7.19) vale, então temos que provar a validade de

f(n+1∑j=1

λjaj

)≤

n+1∑j=i

λjf(aj). (7.20)

Notemos quen+1∑j=1

λjaj =n∑j=1

λjaj +(

1−n∑j=1

λj

)an+1

= αn∑j=1

λjαaj + (1− α)an+1,

(7.21)

onde α =n∑j=1

λj. Assim, usando quen∑j=i

λjα

= 1 e a hipótese de indução,

obtemos

f(n+1∑j=1

λjaj

)≤ αf

( n∑j=1

λjαaj

)+(1− α)f(an+1)

≤ αn∑j=1

λjαf(aj) + (1− α)f(an+1)

=n+1∑j=1

λjf(aj),

(7.22)

como queríamos provar.

7.5 Exercícios

1. Provar que em todo triângulo a soma dos comprimentos das

medianas é menor que o perímetro do triângulo e maior que o

semiperímetro deste.

Page 271: Resolução de problemas

7.5 Exercícios 251

2. Os centros de três círculos que não se intersectam estão sobre

uma reta. Prove que se um quarto círculo toca de forma tangente

os três círculos, então o raio deste é maior que pelo menos um

dos raios dos três círculos dados.

3. Dado n inteiro positivo, provar quen∑j=1

1

j≥ 2n

n+ 1.

4. A soma de três números positivos é 6. Provar que a soma de

seus quadrados não é menor que 12.

5. Determinar as dimensões do paralelepípedo de menor diagonal

possível, sabendo que a soma dos comprimentos de todas suas

arestas é 12.

6. Encontrar todas as soluções positivas do sistema de equações

não lineares x21 + · · ·+ x210 = 1

1x21

+ · · ·+ 1x210

= 100.

7. Demonstrar que, se a1, a2, . . . , an são números positivos tais que

a1a2 · · · an = 1

então

(1 + a1)(1 + a2) · · · (1 + an) ≥ 2n.

8. Prove que a média geométrica é super-aditiva, isto é, para nú-

meros não negativos ai e bi, 1 ≤ i ≤ n, tem-se

n

√√√√ n∏i=1

ai + n

√√√√ n∏i=1

bi ≤ n

√√√√ n∏i=1

(ai + bi).

Page 272: Resolução de problemas

252 7 Desigualdades

Além disso, estude em que condições ocorre a igualdade.

Sugestão: Use a desigualdade entre as médias geométrica e aritmética.

9. Usar o método de indução para provar a desigualdade de Cauchy-

Schwarz.

10. Para todo λ realn∑i=1

(ai +λbi)2 ≥ 0. Use este fato para dar outra

prova da desigualdade de Cauchy-Schwarz.

11. Use a desigualdade de Cauchy-Schwarz para dar uma prova alter-

nativa da desigualdade entre as médias aritmética e quadrática

(ma ≤ mq).

12. Prove quen∑i=1

aibi ≤1

2

n∑i=1

a2i +n∑i=1

b2i

.

13. Prove que a4 + b4 + c4 ≥ abc(a+ b+ c).

14. Prove que se a ≥ 0, b ≥ 0 e c ≥ 0, então

(a+ b)(a+ c)(b+ c) ≥ 8abc.

15. Prove a desigualdade de Bernoulli: (1 + x)n > 1 + nx, para

qualquer x positivo e n inteiro positivo.

16. Prove que se a, b, c e d são inteiros positivos, então:

(a+ b+ c+ d)

(1

a+

1

b+

1

c+

1

d

)≥ 16.

17. Prove que se a ≥ 0, b ≥ 0 e c ≥ 0, então

(ab+ bc+ ca) ≥ a√bc+ b

√ac+ c

√ab.

Page 273: Resolução de problemas

7.5 Exercícios 253

18. Prove que se x ≥ 0, então 3x3 − 6x2 + 4 ≥ 0.

Sugestão: Use a desigualdade entre as médias aritmética e geométrica.

19. Prove que se x ≥ 0, então 2x+ 3/8 ≥ 4√x.

20. Sejam C1 e C2 dois círculos concêntricos de raios r1 e r2, res-

pectivamente, com r1 < r2. Sobre o círculo C1 se marcam dois

pontos P1 e P2 diametralmente opostos. Deseja-se encontrar o

ponto P sobre o círculo C2 que maximiza a soma

d(P ) = PP1 + PP2.

Page 274: Resolução de problemas

254 7 Desigualdades

Page 275: Resolução de problemas

8

Polinômios

A oisa mais bela que podemos ontemplar é o mistério. Isto é afonte da verdadeira arte e iên ia. Albert Einstein8.1 Operações com Polinômios

A necessidade de estudar equações polinomiais aparece em problemas

práticos da humanidade desde épocas muito remotas. Indícios arque-

ológicos indicam que os babilônicos já tinha o domínio de técnicas de

resolução de algumas equações do primeiro grau e do segundo grau,

apresentadas em forma de problemas cotidianos. Contudo, o grande

avanço teórico no estudo das equações polinomiais só se iniciou com o

Renascimento na Europa. No início do século XVI, Vièti introduziu o

uso de letras para representar quantidades desconhecidas.

Na mesma época, um outro grande desao estava perturbando as

mentes matemáticas de toda a Europa, em especial as da Itália. A

solução explícita utilizando as operações elementares (soma, subtra-

ção, multiplicação, divisão, radiciação e potenciação) da equação do

255

Page 276: Resolução de problemas

256 8 Polinômios

terceiro grau não era conhecida e muitos dos melhores matemáticos

da época trabalharam neste problema, destacando-se entre eles Ni-

colo Fontana, o Tartaglia (gago, em italiano). A história da solução

desta equação está repleta de intrigas, disputas e acusações, envol-

vendo Tartaglia e Cardano. Hoje os historiadores atribuem a Tarta-

glia a primazia na descoberta da solução da equação do terceiro grau

como conhecemos. É desta época também a solução da equação do

quarto grau, atribuída a Ludovico Ferrari.

Entretanto, apesar dos muitos esforços empreendidos na direção de

encontrar a solução geral da equação do quinto grau, mais de 200 anos

se passaram sem nenhum sucesso. Até que em 1824, o matemático

norueguês Niels Abel mostrou que é impossível resolver as equações de

grau cinco em sua forma geral. Ou seja, nem todas as equações de grau

cinco podem ser resolvidas com as operações elementares. Mais ainda,

em 1830 o matemático francês Evariste Galois descobriu um método

que determina quando uma equação de grau qualquer é resolúvel com

as operações elementares, encerrando um belíssimo capítulo do estudo

das equações polinomiais e da Matemática.

Neste capítulo iremos estudar um pouco mais formalmente os poli-

nômios e suas propriedades.

Denição 8.1. Um polinômio na variável x é uma expressão do tipo

p(x) = anxn + an−1x

n−1 + · · ·+ a1x+ a0

onde a0, a1, . . . , an são números. Se an 6= 0, dizemos que n é o grau

do polinômio e a0, a1, . . . , an são seus coecientes. O coeciente an é

chamado de coeciente líder do polinômio.

Observação 8.2. Não se dene o grau do polinômio nulo, que tem

todos os coecientes iguais a zero.

Page 277: Resolução de problemas

8.1 Operações com Polinômios 257

Por exemplo,

• p(x) = 3x− 1 é um polinômio de grau 1;

• q(x) = 4x3 + 7x+ 1 é um polinômio de grau 3;

• t(x) =π

2x4 é um monômio de grau 4;

• v(x) = −π2x4 + 5x2 + 1 é um polinômio de grau 4;

• u(x) = 7 é um polinômio de grau 0.

Uma equação polinomial de grau n, ou simplesmente uma equação

de grau n, é uma sentença p(x) = 0, onde p(x) é um polinômio de

grau n com coecientes reais. Por exemplo, 2x−1 = 0 é uma equação

do primeiro grau, enquanto −x5 + 4x3 + 5x− 1 = 0 é uma equação de

grau 5. Note que nem todos os coecientes precisam ser diferentes de

zero.

Para obtermos o valor do polinômio p(x) = anxn + an−1xn−1 +

· · ·+a1x+a0 no número real r, devemos substituir x por r para obter

o número real

p(r) = anrn + an−1r

n−1 + · · ·+ a1r + a0.

Por exemplo, o valor do polinômio p(x) = 4x3 − 7x+ 1 em 2 é p(2) =

4 · 23 − 7 · 2 + 1 = 19.

Dizemos que um número real r é uma raiz para a equação

anxn + an−1x

n−1 + · · ·+ a1x+ a0 = 0

se o valor de p(x) = anxn + an−1xn−1 + · · ·+ a1x+ a0 em r é zero, ou

seja, se r verica

anrn + an−1r

n−1 + · · ·+ a1r + a0 = 0.

Page 278: Resolução de problemas

258 8 Polinômios

Por exemplo, 5 é raiz da equação:

2x− 10 = 0.

Uma das vantagens dos polinômios sobre outros objetos matemá-

ticos é que podemos denir as operações de soma de polinômios e

multiplicação de polinômios. Com estas operações, o conjunto dos po-

linômios possui muitas propriedades similares à dos números inteiros,

tornando prático o seu uso.

Vamos denir agora o que signica a soma de dois polinômios.

Para isso, vamos começar somando dois monômios e depois estender

nossa denição para polinômios em geral.

Para somar dois monômios de mesmo grau p(x) = akxk e q(x) =

bkxk somamos seus coecientes, obtendo o polinômio t(x) = p(x) +

q(x) = (ak+bk)xk. Em geral, para somar o polinômio p(x) = a0+a1x+

a2x2 + · · ·+ anx

n com o polinômio q(x) = b0 + b1x+ · · ·+ bmxm, onde

n ≤ m devemos somar todos os monômios de mesmo grau, obtendo o

polinômio:

t(x) = p(x) + q(x) = c0 + c1x+ · · ·+ cmxm

onde, ci = ai + bi para 0 ≤ i ≤ n e ci = bi para i > n.

Por exemplo, sendo

• p(x) = 3x− 1,

• q(x) = 4x3 + 7x+ 1,

• t(x) = π2x4,

• v(x) = −π2x4 + 5x2 + 1

Page 279: Resolução de problemas

8.1 Operações com Polinômios 259

temos que

• p(x) + q(x) = 4x3 + (3 + 7)x− 1 + 1 = 4x3 + 10x,

• v(x) + t(x) = (π2− π

2)x4 + 5x2 + 1 = 5x2 + 1.

A seguir, enumeramos algumas propriedades simples e importantes

da soma de polinômios que decorrem da denição dada e das propri-

edades análogas válidas para os números reais.

1. Associatividade. Dados polinômios p(x), q(x) e t(x), vale

(p(x) + q(x)) + t(x) = p(x) + (q(x) + t(x))

2. Elemento neutro. Se 0 denota o polinômio nulo e p(x) é um

polinômio qualquer, então

0 + p(x) = p(x).

3. Elemento simétrico. Se p(x) = a0 + a1x + · · · + anxn é um

polinômio, então o polinômio q(x) = −a0 − a1x − · · · − anxn

satisfaz:

p(x) + q(x) = 0.

4. Comutatividade. Se p(x) e q(x) são polinômios, então

p(x) + q(x) = q(x) + p(x).

Note que os números inteiros possuem propriedades similares para

a operação de soma de números inteiros. Vamos agora denir o produto

de dois polinômios. Para isso, vamos primeiramente denir o produto

de dois monômios, como já zemos no caso de soma de polinômios.

Page 280: Resolução de problemas

260 8 Polinômios

Se n,m são números naturais, denimos o produto dos monômios

p(x) = anxn e q(x) = bmx

m como:

p(x)q(x) = anbmxn+m.

Tendo isto em mente, para efetuarmos o produto do polinômio de

grau n, p(x) = a0 + a1x + a2x2 + · · · + anx

n pelo polinômio q(x) =

b0 + b1x+ · · ·+ bmxm de grau m, com n ≤ m, devemos:

• Completamos a escrita de p(x) e de q(x) até o termo n + m

colocando ak = 0 para k > n e bk = 0 para k > m;

• Denimos

t(x) = p(x)q(x) = c0 + c1x+ · · ·+ cn+mxn+m

onde, ci = a0bi +a1bi−1 + · · ·+ai−1b1 +aib0 para 0 ≤ i ≤ n+m.

Apesar de parecer complicada, a denição não é tão difícil de ser

aplicada. Para tentar visualizar o processo de multiplicação de dois

polinômios vamos pensar que os monômios são seres alienígenas vin-

dos do distante planeta de Algebrum e possuam mãos. Quando dois

monômios se encontram, invariavelmente eles apertam as mãos e desse

aperto aparece o produto desses monômios.

Assim, para multiplicar os polinômios p(x) e q(x), que são forma-

dos por dois grupos de monômios, devemos escolher o primeiro monô-

mio de p(x) e fazê-lo apertar a mão de cada um dos monômios de

q(x), somando os monômios obtidos. Após isso, tomamos o segundo

monômio de p(x) e fazemos ele apertar a mão de cada um dos monô-

mios de q(x), somando os monômios obtidos aos monômios anteriores.

Repetimos o processo até o último monômio de p(x).

Page 281: Resolução de problemas

8.1 Operações com Polinômios 261

Deste modo, se p(x) = x2 + 2x − 3 e q(x) = −x2 + 5x + 1, para

obter p(x)q(x) fazemos:

p(x)q(x) = −x4 + 5x3 + x2 − 2x3 + 10x2 + 2x+ 3x2 − 15x− 3

= −x4 + 3x3 + 14x2 − 13x− 3.

Observe que com a denição de multiplicação de polinômios dada

acima, o coeciente c0 é igual a a0b0. Do mesmo modo, o coeciente

do termo xn+m é cn+m = anbm. Como p(x) tem grau n (isto é, an 6= 0)

e q(x) tem grau m (bm 6= 0), o coeciente cn+m = anbm 6= 0. Logo,

o polinômio p(x)q(x) tem grau n + m. Com isso, demonstramos o

seguinte fato:

Proposição 8.3. Se o polinômio p(x) tem grau n e o polinômio q(x)

tem grau m, então o polinômio p(x)q(x) tem grau n+m.

Um caso particular interessante é quando multiplicamos um núme-

ro c, que podemos considerar como sendo um polinômio de grau zero

q(x) = c, por um polinômio p(x) = a0 + a1x+ · · ·+ anxn. Neste caso,

nós obtemos o polinômio

cp(x) = ca0 + ca1x+ · · ·+ canxn.

Do mesmo modo em que podemos vericar as propriedades da

soma de polinômios a partir das propriedades similares dos números

reais, podemos também vericar as propriedades abaixo sobre a mul-

tiplicação de polinômios. Deixamos essa vericação como exercício.

1. Associatividade. Dados polinômios p(x), q(x) e t(x), vale

(p(x)q(x))t(x) = p(x)(q(x)t(x))

Page 282: Resolução de problemas

262 8 Polinômios

2. Elemento neutro. Se 1 denota o polinômio constante e p(x) é

um polinômio qualquer, então

1p(x) = p(x).

3. Comutatividade. Se p(x) e q(x) são polinômios, então

p(x)q(x) = q(x)p(x).

4. Distributividade. Se p(x), q(x) e t(x) são polinômios, então

(p(x) + q(x))t(x) = q(x)t(x) + p(x)t(x).

Note que, assim como nos inteiros, a propriedade de existência de

elementos inversos para a multiplicação de polinômios não vale. De

fato, podemos vericar que se p(x) é um polinômio de grau n maior ou

igual a um, então não existe um polinômio q(x) tal que p(x)q(x) = 1.

De fato, suponha por absurdo, que exista q(x) um polinômio com grau

m ≥ 0 tal que

p(x)q(x) = 1.

Então, utilizando a Proposição 8.3 temos que o grau de p(x)q(x) é

n + m que é maior ou igual que um. Como o grau do polinômio

constante 1 é zero, temos que a igualdade acima não pode valer, onde

chegamos a um absurdo.

Em resumo, os únicos polinômios que podem ter inversos com res-

peito à operação de multiplicação são os polinômios constantes não

nulos. Esta é mais uma das semelhanças entre os inteiros e os polinô-

mios.

Page 283: Resolução de problemas

8.2 Algoritmo de Euclides 263

8.2 Algoritmo de Euclides

Diremos que um polinômio a(x) divide o polinômio b(x) se existir q(x)

tal que b(x) = q(x)a(x).

Por exemplo, o polinômio a(x) = x2 + x + 1 divide o polinômio

x3 − 1 pois

(x− 1)(x2 + x+ 1) = x3 − 1.

Devido à Proposição 8.3, se o polinômio a(x) divide o polinômio

não nulo b(x), então o grau de a(x) é menor ou igual ao grau de b(x).

Agora, vamos enunciar um fato que vale para os inteiros e que vale

também para os polinômios e que será de grande utilidade. Pedimos

que o leitor releia o algoritmo de Euclides, estudado no Capítulo 3.

No conjunto dos polinômios, ainda vale

Teorema 8.4 (Algoritmo de Euclides). Sejam a(x) e b(x) dois polinô-

mios com coecientes reais, b(x) 6= 0. Então, existem polinômios com

coecientes reais q(x) e r(x), com r(x) = 0 ou grau de r(x) menor

que o grau de b(x) tais que:

a(x) = b(x)q(x) + r(x).

Além disso, q(x) e r(x) estão determinados de modo único.

Demonstração. Vamos mostrar primeiro a unicidade. De fato, assuma

que

a(x) = b(x)q1(x) + r1(x) = b(x)q2(x) + r2(x),

com r1 e r2 de graus menores que o grau de b. Assim,

b(q1 − q2) = r2 − r1.

Page 284: Resolução de problemas

264 8 Polinômios

Consequentemente, q1 = q2, já que caso contrário, o polinômio b(q1 −q2) teria grau pelo menos igual ao grau de b e o polinômio r2− r1 temgrau menor que o grau de b.

Vamos agora mostrar a existência. Os passos da prova são idênticos

a prova do algoritmo de Euclides para números inteiros, demonstrado

no Capítulo 3. De fato, a ideia é reduzir o grau do dividendo até

que ele se torne menor que o do divisor e a divisão se torne imediata.

Note que se a tem grau menor que b, então tomamos o resto com

sendo r = a e o quociente como sendo q = 0. Suponhamos que

a(x) = anxn+ · · ·+a1x+a0 tenha grau n e b(x) = bmx

m+ · · ·+b1x+b0

tenha grau m e que n > m. Dena

c1(x) = a(x)− anbmxn−mb(x).

Observe que o grau de c1 é no máximo n− 1. Se c1 puder se dividido

por b, digamos com c1(x) = b(x)q(x) + r(x), com grau de r(x) menor

que o grau de b(x), então

a(x) = b(x)anbmxn−m + c1(x) = b(x)(

anbmxn−m + q(x)) + r(x).

Logo, reduzimos o problema de dividir o polinômio a(x) por b(x) pelo

problema de dividir o polinômio c1(x) por b(x), com c1(x) de grau

menor que a(x). Repetimos o processo, utilizando c1 no lugar de a(x),

obtendo o polinômio c2(x) de grau menor que o de c1(x). Como a cada

passo reduzimos o grau do dividendo em pelo menos uma unidade, ao

m de no máximo n −m passos, obteremos um polinômio com grau

menor que o grau de b(x), que é claramente divisível por b(x). Proce-

dendo como antes, achamos q(x) e r(x) tais que a(x) = b(x)q(x)+r(x)

e r(x) com grau menor que o grau de b(x).

Page 285: Resolução de problemas

8.2 Algoritmo de Euclides 265

Por exemplo, se a(x) = 10x3 − 3x + 2 e b(x) = x2 + 1, tomando

q(x) = 10x e r(x) = −13x+ 2 temos que

10x3 − 3x+ 2 = (x2 + 1)10x+ (−13x+ 2).

Note que o grau de r(x) = −13x + 2 é menor que o grau de b(x) =

x2 + 1.

Se na expressão do polinômio p(x) decidimos substituir a variável

x por um número real s, estaremos avaliando o polinômio p(x) em s

e denotamos este número por p(s).

Por exemplo, se p(x) = x2 + 3x + 1, então substituindo x por 2,

temos que

p(2) = 22 + 3 · 2 + 1 = 11

e fazendo x = −3

p(−3) = (−3)2 + 3 · (−3) + 1 = 1.

Quando p(s) = 0 dizemos que s anula o polinômio não nulo p(x),

ou ainda, que s é uma raiz do polinômio p(x).

Por exemplo, para p(x) = x3 − 8, temos que 2 é uma raiz de p(x)

já que p(2) = 23 − 8 = 0.

Um fato muito importante que é consequência do algoritmo de

Euclides é o seguinte teorema:

Teorema 8.5. Se s é uma raiz do polinômio p(x), então o polinômio

x− s divide p(x). Reciprocamente, se x− s divide p(x), então s é raiz

de p(x).

Demonstração. Primeiramente, assuma que x − s divida p(x). Neste

caso, existe um polinômio q(x) tal que p(x) = q(x)(x− s). Avaliando

Page 286: Resolução de problemas

266 8 Polinômios

o polinômio p(x) em s, temos que:

p(s) = q(s)(s− s) = q(s) · 0 = 0.

Logo s é uma raiz de p(x).

Para provar que se s é uma raiz de p(x) então x − s divide p(x),

vamos utilizar o algoritmo da divisão, com a(x) = p(x) e b(x) = x− s.Neste caso, temos que existem q(x) e r(x) de modo que r(x) = 0 ou o

grau de r(x) é menor que o grau de x− s e além disso vale

p(x) = q(x)(x− s) + r(x).

Observe que, com as condições do resto r(x), podemos escrever que

r(x) = c ∈ R. Então, p(x) = q(x)(x−s)+c e 0 = p(s) = q(s)·0+c = c.

Portanto, r(x) = 0 e p(x) = q(x)(x− s), isto é, x− s divide p(x).

A proposição anterior nos permite determinar o número máximo

de raízes reais de um polinômio não nulo. De fato, vamos mostrar.

Proposição 8.6. O número máximo de raízes reais do polinômio não

nulo p(x) = anxn + an−1xn−1 + · · ·+ a1x+ a0 é n.

Demonstração. Digamos que s0 < s1 < s2 < · · · < sk sejam raízes

distintas do polinômio p(x). Observe que podemos utilizar a Propo-

sição 8.5 para garantir que existe um polinômio não nulo q1(x) tal

que

p(x) = q1(x)(x− s0).

Assim, pela Proposição 8.3, o grau de q1(x) deve ser igual a n − 1.

Note que p(si) = q1(si)(si−s0). Como para todo i = 1, 2, . . . , k temos

que si > s0 com p(si) = 0, temos que, necessariamente, q1(si) = 0.

Assim, em particular, temos que q1(s1) = 0. Logo, podemos aplicar

Page 287: Resolução de problemas

8.2 Algoritmo de Euclides 267

a proposição novamente para obter que existe um polinômio não-nulo

q2(x) tal que

q1(x) = q2(x)(x− s1).

Assim, como o grau de q1(x) é n − 1, pela Proposição 8.3, o grau de

q2(x) deve ser igual a n− 2.

Novamente, temos que q1(si) = q2(si)(si − s1), si > s1 e p(si) = 0

para todo i = 2, . . . , k. Disto segue que, necessariamente, q2(si) = 0,

se i = 2, 3, . . . , k. Assim, temos que q2(s2) = 0.

Logo, podemos repetir esse argumento para obter um polinômio

q3(x) de grau n−3, de modo que s3, s4, . . . , sk são raízes de q3(x). Re-

petindo o argumento, encontramos uma sequência q1(x), q2(x), q3(x), . . .

com graus no máximo n − 1, n − 2, n − 3, . . . o que nos leva a con-

cluir que não podemos repetir esse argumento mais que n vezes, já

que os graus dos polinômios q1(x), q2(x), q3(x), . . . estão diminuindo.

Ou seja, não podemos ter mais que n raízes para o polinômio p(x), o

que conclui a prova.

Alertamos que, apesar da Proposição 8.6 nos garantir que existem

no máximo n raízes reais de um polinômio de grau n não nulo, existem

polinômios que não possuem raízes reais. Por exemplo, p(x) = x2 + 1

não possui raízes rais, já que x2 ≥ 0 para todo número real x.

Uma consequência da Proposição 8.6 é a seguinte:

Proposição 8.7. Se dois polinômios p(x) e q(x) de grau n avaliados

em n+ 1 números r1, r2, . . . , rn+1 coincidem, isto é, p(ri) = q(ri) para

i = 1, 2, 3, . . . , n+ 1, então p(x) e q(x) são iguais.

Demonstração. Considere o polinômio t(x) = p(x) − q(x). Observe

que se t(x) é não-nulo, o grau de t(x) é no máximo n, já que p(x)

Page 288: Resolução de problemas

268 8 Polinômios

e q(x) têm graus iguais a n. Observe ainda que t(ri) = 0, já que

p(ri) = q(ri) e

t(ri) = p(ri)− q(ri) = 0.

Logo, t(x) tem grau no máximo n e mais de n raízes, contradizendo a

Proposição 8.6.

No Exercício 23 faremos uma aplicação interessante dessa propo-

sição, propondo que você prove que dados números reais a1, a2, . . . ,

an+1 e r1, r2, . . . , rn+1, então existe um único polinômio de grau n tal

que p(ri) = ai.

8.3 Sempre Existem Raízes de um Polinômio?

Pode parecer frustrante o fato de que um polinômio com coecientes

reais pode não possuir raízes reais. Por exemplo, quando tentamos

aplicar a fórmula de Bhaskara à equação x2 + 1 = 0, encontramos

∆ = −4 e, consequentemente, se fosse possível escrever as soluções,

elas se escreveriam como

x1 =

√−4

2

e

x2 = −√−4

2

É claro que as expressões acima não têm sentido no conjunto dos

números reais, pois não existe número cujo quadrado seja −4, ou seja,

não é possível extrair a raiz quadrada de −4. Isso tirou o sono de

várias gerações de matemáticos. Desde Herón de Alexandria há dois

mil anos atrás, os matemáticos encontram expressões como a do tipo

acima, envolvendo raízes de números negativos.

Page 289: Resolução de problemas

8.3 Sempre Existem Raízes de um Polinômio? 269

A primeira reação da comunidade matemática foi rejeitar esses

números complexos e simplesmente desconsiderar raízes de números

negativos. Porém, já no século XVI, Cardano se deu conta de que os

números complexos surgem naturalmente quando desejamos resolver

uma equação do terceiro ou quarto grau, mas relutava quanto ao seu

uso, dizendo que esses números eram tão sutis, quanto inúteis.

No século seguinte, motivado pela sugestão de Albert Girard que

uma equação de grau n possui n raízes, Reneé Descartes observou que

os números reais eram insucientes para representar todas essas raízes

e utilizou o termo imaginárias para as raízes que não são reais.

A notação tradicional i =√−1 só veio a ser introduzida um século

mais tarde, com Leonard Euler, que também é o pai do termo número

complexo. Euler e o matemático francês Jean D'Alambert zeram apli-

cações dos números complexos a problemas práticos, como projeção

de mapas e hidrodinâmica. Euler e Lagrange, grandes matemáticos

da história da humanidade, tentaram mostrar a armação de Girard,

de que uma equação de grau n possui n raízes, mas sem sucesso. A

primeira prova correta de tal teorema só apareceu no nal do século

XVIII com os trabalhos de Gauss.

8.3.1 Números Complexos e Raízes de Polinômios

O conjunto dos números complexos, denotado pela letra C, é o con-

junto das expressões

C = x+ iy;x, y ∈ R,

onde i satisfaz i2 = −1. Costuma-se denotar i por√−1. Destacamos

que i é meramente um símbolo que nos ajudará a denir as operações

de soma e de multiplicação de números complexos. Essas operações

Page 290: Resolução de problemas

270 8 Polinômios

terão as mesmas propriedades que as operações de números reais, como

associatividade, comutatividade, elemento neutro, etc. Por exemplo,

são números complexos 2− 3i, 3 + i e −3i.

Vamos denir a soma e multiplicação de números complexos. Da-

dos dois números complexos a+ bi e c+ di denimos a soma como:

(a+ bi) + (c+ di) = (a+ b) + (c+ d)i

e denimos a multiplicação como

(a+ bi)(c+ di) = (ac− bd) + (bc+ ad)i

Por exemplo, se tomamos os números 2− 3i e 3 + 4i então

(2− 3i) + (3 + 4i) = 5 + i

e

(2− 3i)(3 + 4i) = (2 · 3− (−3 · 4)) + (−3 · 3 + 2 · 4)i = 18− i.

Aqui nós estamos considerando 0 + 3i = 3i e 3 + 0 · i = 3. Isso

nos permite colocar os números reais dentro do conjunto dos números

complexos, considerando cada número real r como sendo um número

complexo da forma r + 0 · i.Fica para o leitor a vericação de que valem as propriedades de

associatividade, comutatividade, etc. O elemento neutro da soma é o

elemento 0 + 0 · i que simplesmente denotaremos por 0. Do mesmo

modo, o elemento neutro da multiplicação é 1+0 ·i, que será denotadopor 1. O leitor curioso pode achar mais informações sobre números

complexos e soluções de equações algébricas em [5] ou [13].

Page 291: Resolução de problemas

8.3 Sempre Existem Raízes de um Polinômio? 271

Assim, dado um número complexo z faz sentido avaliar o polinômio

(de coecientes complexos ou reais) p(x) = anxn + an−1xn−1 + · · · +

a1x+ a0 em z, obtendo o número complexo

p(z) = anzn + an−1z

n−1 + · · ·+ a1z + a0.

Por exemplo, se p(x) = x2 + 4, então 2i e −2i são raízes deste

polinômio, já que:

p(2i) = (2i)2 + 4 = −4 + 4 = 0.

e

p(−2i) = (−2i)2 + 4 = 4i2 + 4 = −4 + 4 = 0.

Note que p(x) não possui nenhuma raiz real, mas possui duas raízes

complexas. Como já mencionamos, a grande vantagem em utilizar os

números complexos em vez dos números reais é que, dado um polinô-

mio qualquer com coecientes complexos, ele sempre tem uma raiz

complexa. Isso foi o assunto da tese de doutorado do Príncipe da

Matemática, Johann Carl Friedrich Gauss (1777-1855).

Teorema 8.8 (Teorema Fundamental da Álgebra). Todo polinômio

não constante com coecientes complexos de grau n possui exatamente

n raízes complexas, contadas com multiplicidade.

Uma demonstração do Teorema Fundamental da Álgebra foge do

objetivo deste livro. Podem ser dadas várias demonstrações diferen-

tes desse teorema, utilizando diversas teorias matemáticas avançadas.

Uma demonstração desse teorema pode ser achada em [13].

Page 292: Resolução de problemas

272 8 Polinômios

8.4 Exercícios

1. Calcule o quociente e o resto da divisão de p(x) por q(x) para

os polinômios p(x) e q(x) dados:

(a) p(x) = 3x3 − 2x+ 1 e q(x) = −7x− 1;

(b) p(x) = x5 − 1 e q(x) = x− 1;

(c) p(x) = 3x5 − 2x3 + 1 e q(x) = x2 + x+ 1

2. Encontre os valores de A e B de forma que

x+ 1

x2 − x =A

x+

B

x− 1.

3. Se os polinômios x2−x+4 e (x−a)2+(x+b) são iguais, encontre

a+ b.

4. Quais os valores de a e b que tornam iguais os polinômios

P1(x) = x2 − x− 6 e P2(x) = (x+ a)2 − b?

5. A divisão de P (x) por x4 + 1 tem quociente x + 2 e resto 1.

Encontre o polinômio P (x).

6. Qual o resto da divisão do polinômio x100 por x+ 1?

7. Determine o resto da divisão do polinômio p(x) pelo polinômio

g(x) = x, onde p(x) = (x− 1)(x− 2) . . . (x− n) + b .

8. Mostre que xn − 1 é divisível por x− 1 para todo n ≥ 1.

9. Faça os seguintes itens:

(a) encontre o quociente da divisão de xn+1 − 1 por x− 1;

Page 293: Resolução de problemas

8.4 Exercícios 273

(b) utilize a divisão anterior para calcular a soma 1 + x+ x2 +

x3 + · · · + xn dos n primeiros termos de uma progressão

geométrica de razão x.

10. Determine o valor de a para que o polinômio P (x) seja divisível

por x− a, onde P (x) = x3 + (1− a)x2 + (1 + a)x− 1.

11. Mostre que o polinômio P (x) = x100 − 2x50 + 1 é divisível por

x2 − 1.

12. Mostre que o resto r(x) da divisão do polinômio p(x) por x− sé r(x) = p(s).

Dado o polinômio p(x) = anxn + an−1xn−1 + · · · + a1x + a0

denimos a derivada de p(x) como sendo o polinômio:

p′(x) = nanxn−1 + (n− 1)an−1x

n−2 + · · ·+ 2a2x+ a1.

Por exemplo, a derivada do polinômio x5 é o polinômio 5x4 e a

derivada do polinômio x3+5x2+2x−1 é o polinômio 3x2+10x+2.

13. Usando as informações do Exercício 12, calcule:

(a) a derivada dos polinômios:

(i) x+ 1;

(ii) x4 + 3;

(ii) 1 + x+ x2 + x3 + · · ·+ xn.

(b) Sabendo que p(0) = 1, calcule também o polinômio p(x)

cuja derivada é

(i) x4.

(ii) −x2 + 1.

Page 294: Resolução de problemas

274 8 Polinômios

(ii) x3 + 2x2 + 3.

(c) Prove que se p(x) e q(x) são polinômios, então

(i) (p+ q)′(x) = p′(x) + q′(x)

(ii) (pq)′(x) = p′(x)q(x) + p(x)q′(x)

Sugestão: Faça primeiro para monômios.

Denimos uma raiz múltipla de um polinômio p(x) como sendo

uma raiz a tal que (x − a)2 divide p(x). Caso a seja uma raiz

que não é raiz múltipla, dizemos que ela é raiz simples.

14. Mostre que a é raiz múltipla de um polinômio p(x) se, e somente

se, a é raiz de p(x) e de p′(x).

Sugestão: Use o exercício anterior.

15. Para quais valores de n ∈ N tem-se que

(a) 1 + x2 + x4 + . . .+ x2n−2 é divisível por 1 + x+ . . .+ xn−1?

(b) 1 + x3 + x6 + . . .+ x3n−3 é divisível por 1 + x+ . . .+ xn−1?

(c) Generalize.

16. (a) Resolva a equação 20x3 − 30x2 + 12x − 1 = 0, sabendo-se

que1

2é uma de suas raízes.

(b) Uma raiz da equação x3−(2a+1)x2+a(a+2)x−a(a+1) = 0

é a+ 1, ache as outras duas.

17. Ache os possíveis valores de a ∈ Z para que o polinômio

a2x4 + 4x3 + 4ax+ 7

seja divisível por x+ 1.

Page 295: Resolução de problemas

8.4 Exercícios 275

Um polinômio com coecientes reais não constante p(x) é dito ir-

redutível se p(x) = a(x)b(x), então a(x) ou b(x) são polinômios

constantes. Quando p(x) não for irredutível, diremos simples-

mente que ele é redutível. Os polinômios irredutíveis desempe-

nham papel análogo no conjunto dos polinômios ao dos números

primos em Z.

18. Prove que todo polinômio de grau 1 é irredutível.

19. Prove que se f(x) é um polinômio de grau ≥ 2 e possui uma raiz

real, então f(x) é redutível.

20. Mostre que todo polinômio f(x) de grau ímpar ≥ 3 é redutível.

Um polinômio com coecientes inteiros não constante p(x) é dito

irredutível sobre Q se p(x) = a(x)b(x) com a(x) e b(x) polinômios

com coecientes racionais, então a(x) ou b(x) são polinômios

constantes.

Um teorema importante que descreve uma condição para um

polinômio ser irredutível sobre Q é o conhecido critério de Ei-

senstein, que diz:

Teorema 8.9 (Critério de Eisenstein). Seja f(x) = a0 + a1x +

· · ·+ anxn um polinômio com coecientes inteiros. Suponha que

exista um primo p tal que:

(a) p - an;

(b) p | a0, p | a1, . . . , p | an−1;(c) p2 - a0.

Então, f(x) é irredutível sobre Q.

Page 296: Resolução de problemas

276 8 Polinômios

Para uma prova desse resultado veja o livro [2]. Faça os seguintes

problemas:

21. Mostre que os seguintes polinômios f(x) são irredutíveis sobre

Q.

Sugestão: Use o critério de Eisenstein.

(a) f(x) = x4 + 2x3 + 2x2 + 2x+ 2;

(b) f(x) = x6 + 15;

(c) f(x) = x4 + 10x3 + 20x2 + 30x+ 22.

22. Determine quais dos polinômios abaixo são irredutíveis sobre Q.

Sugestão: Use o critério de Eisenstein.

(a) x3 − x+ 1

(b) x3 + 2x+ 10

(c) x4 − x+ 1

O problema a seguir trata do polinômio de interpolação de La-

grange.

23. Demonstre a proposição a seguir:

Polinômio de Interpolação de Lagrange. Sejam ai, bi em

R, i = 1, 2, . . . , n, com os ai′s dois a dois distintos e os bi′s nem

todos nulos. Considere os polinômios

pi(x) = bi(x− ai) · · · (x− ai−1)(x− ai+1) · · · (x− an)

(ai − a1) · · · (ai − ai−1)(ai − ai+1) · · · (ai − an)

Page 297: Resolução de problemas

8.4 Exercícios 277

para i = 1, 2, . . . , n. Então, o polinômio

p(x) =n∑i=1

pi(x)

é o único polinômio de grau menor que n, tal que p(ai) = bi,

para todos i = 1, 2, . . . , n.

24. Determine o polinômio p(x) de grau 7 tal que

p(1) = p(2) = · · · = p(7) = 8 e p(0) = 1.

Page 298: Resolução de problemas

278 8 Polinômios

Page 299: Resolução de problemas

A

Apêndice: Funções

Estamos acostumados a expressões cotidianas que retratam uma

relação entre grandezas, como por exemplo, o quanto João ganha é

função do que ele trabalha, ou ainda a distância que percorremos é

uma função da velocidade e do tempo que viajamos. Essas e outras

expressões ilustram a noção de função como uma relação entre grande-

zas de dois conjuntos dados. Matematicamente, a noção de função foi

melhor entendida muito recentemente, com os avanços teóricos ocorri-

dos no nal do século XIX e início do século XX. Entretanto, o seu uso

como instrumento e os estudos para tornar sua denição um objeto

claro são bem antigos e datam pelo menos desde o início do cálculo

diferencial, onde a noção de função era por vezes entendida como sua

expressão analítica. O entendimento dessa noção foi crucial para o

avanço da Matemática e é importante que o estudante de Matemática

tenha claro seu signicado.

Para iniciar a discussão um pouco mais formalmente da noção

de função, vamos denir intuitivamente uma função como um objeto

matemático composto de três ingredientes: um conjunto não vazio A,

279

Page 300: Resolução de problemas

280 A Apêndice: Funções

chamado de domínio da função, um conjunto não vazio B, chamado

de contradomínio da função e uma correspondência, que associa a

cada elemento do primeiro conjunto um único elemento do segundo

conjunto. O trio domínio, contradomínio e correspondência damos o

nome de função. Para simplicar o seu uso, foi criada uma notação

que empacota todos os três ingredientes. Denotamos uma função por

f : A→ B

x→ f(x)

para indicar que A é o domínio, B é o contradomínio e que se x é

um elemento de A então a ele associaremos o elemento f(x) de B.

É importante não confundir uma função com sua expressão analítica,

quando esta é dada. Para caracterizar uma função, precisamos dar

seus três ingredientes: domínio, contradomínio e correspondência, e

não somente a correspondência y = f(x).

Exemplo A.1. Seja a função f denida de modo que o seu domínio

é o conjunto dos números naturais e o contradomínio é o conjunto dos

números naturais, e a correspondência é tal que a cada número natural

n associamos o seu quadrado n2. Observe que podemos denotar isso

compactamente por:f : N→ N

n→ n2

Veja também que se dermos simplesmente a expressão analítica

x → x2 ou y = x2 para nossa função, ela não estaria caracterizada,

pois não saberíamos qual é o domínio e o seu contradomínio.

Em alguns casos onde o domínio e o contradomínio estão xados e

claros para o interlocutor, podemos nos referir a uma função simples-

mente invocando sua correspondência y = f(x).

Page 301: Resolução de problemas

281

Exemplo A.2. Considere o domínio como sendo o conjunto P for-

mado pelas pessoas do Brasil e o segundo conjunto como sendo o con-

junto L das letras do alfabeto. A correspondência será a seguinte: a

cada pessoa do Brasil, associaremos a primeira letra do seu nome.

Assim, uma pessoa chamada Mário, será associada à letra M. Em

notação de função:f : P → L

x→ f(x)

onde f(x) é a primeira letra do nome de x.

Exemplo A.3. Considere o domínio S como sendo o conjunto dos

pontos de uma sala de aula e o contradomínio como sendo os números

reais. A cada ponto x da sala de aula associamos sua temperatura t(x)

em um dado momento, medida por um termômetro instalado na sala.

Observe que t : S → R assim denida é uma função, pois cada ponto

possui uma única temperatura bem denida no instante xado, que é

um número real. Por outro lado, se trocarmos os papéis do domínio

e contradomínio e a cada número real associamos o ponto da sala que

tem aquela temperatura, não teremos uma função, pois pode haver

mais de um ponto com a dada temperatura ou ainda uma temperatura

que não é atingida por nenhum ponto da sala.

Exemplo A.4. Vamos agora dar outro exemplo em que não temos

uma função, isto é, cuja a nossa aparente correspondência não é de

fato uma correspondência, pois não associa a cada elemento x do do-

mínio um único elemento f(x) do contradomínio. Para tanto, xe o

domínio como sendo o conjunto dos números reais no intervalo [0, 1]

e como contradomínio o conjunto Σ denido pelas sequências de ele-

mentos no conjunto 0, 1, 2, . . . , 9. Ou seja,

Page 302: Resolução de problemas

282 A Apêndice: Funções

Σ =

(a1, a2, a3, . . . ); ai ∈ 0, 1, 2, 3, . . . , 9.

Cada elemento x ∈ [0, 1] possui uma expansão decimal x = 0, x1x2x3 . . . .

Dena f : [0, 1]→ Σ colocando f(x) = (x1, x2, x3, . . . ).

A princípio, parece que f denida desse modo é uma função. Po-

rém, olhando de perto vemos que o número 0, 1 possui mais de uma

representação na base decimal, pois 0, 1 = 0, 09999 . . . . Portanto, f

não está bem denida, isto é, f não associa a cada elemento de [0, 1]

um único elemento de Σ.

Denição A.5. Dada uma função f denida por

f : A→ B

x→ f(x)

o conjunto imagem de f é o subconjunto f(A) do contradomínio B

formado pelos pontos y do contradomínio tais que existe algum ponto

x no domínio A tal que y = f(x). Ou seja

f(A) = y ∈ B; existe x ∈ A tal que y = f(x).

A imagem de um ponto x ∈ A é o ponto f(x). Denimos também

a restrição de f a um subconjunto A′ de seu domínio é a nova função

denida considerando-se o domínio como sendo o conjunto A′ e os

demais elementos os mesmos. Denotamos essa nova função por f |A′

ou aindaf |A′ : A′ → B

x→ f(x)

No Exemplo A.2 poderíamos trocar o domínio por um de seus

subconjuntos não vazios. Por exemplo, poderíamos considerar o sub-

conjunto A de P formado pelas pessoas do Brasil que nasceram em

Alagoas.

Page 303: Resolução de problemas

283

Denição via Relações

Um modo mais formal de denir função é usar a noção de relação

entre dois conjuntos A e B. Uma relação entre A e B é simplesmente

um subconjunto R do produto cartesiano A×B. Uma função é uma

relação R entre A e B que satisfaz duas condições:

• R é unívoca: dados x1, x2 ∈ A e y ∈ B tais que (x1, y) ∈ R e

(x2, y) ∈ R então x1 = x2;

• R é total: dado x ∈ A existe y ∈ B tal que (x, y) ∈ R. de modo

que dado x ∈ A, existe um único y ∈ B tal que (x, y) ∈ R.

Funções Injetoras, Sobrejetoras e Bijetoras

Denição A.6. Uma função f : A→ B é dita injetora (ou injetiva)

se a seguinte propriedade vale:

Dados x, y ∈ A tais que f(x) = f(y), então x = y.

Outro modo equivalente de formular tal propriedade é usando sua

forma contrarrecíproca:

Se x, y ∈ A são tais que x 6= y, então f(x) 6= f(y).

Exemplo A.7. Por exemplo, a função f : R→ R dada por f(x) = x2

não é injetora, pois f(−1) = (−1)2 = 12 = f(1).

Por outro lado, se g : [0,+∞)→ R é dada por g(x) = x2, então g

é injetora, pois dados dois números não negativos a e b tais que g(a) =

g(b), isto é, a2 = b2, então a2 − b2 = 0, de onde (a − b)(a + b) = 0,

restando as possibilidades a = b ou a = −b. Como a e b são positivos,

temos que a = b.

Page 304: Resolução de problemas

284 A Apêndice: Funções

Denição A.8. Uma função f : A → B é dita sobrejetora (ou so-

brejetiva) se a seguinte propriedade vale:

Dado y ∈ B existe x ∈ A, tal que f(x) = y.

Exemplo A.9. Por exemplo, a função f : R → R dada por f(x) =

x2 do exemplo anterior não é sobrejetora, pois não existe nenhum

número real x tal que f(x) = −1, por exemplo. Por outro lado, se

considerarmos g(x) : R → [0,+∞) dada por g(x) = x2, então g é

sobrejetora, pois dado qualquer número não negativo b, podemos tomar

a como sendo a =√b de modo que g(a) = a2 = b.

Denição A.10. Uma função é dita bijetora (ou ainda bijetiva) se

ela é injetora e sobrejetora.

Por exemplo, a função f : R→ R dada por f(x) = x3 é uma função

bijetora, pois é injetora e sobrejetora, já que dado y ∈ R, existe um

único x ∈ R tal que y = x3.

Quando f : A → B é bijetora, então dado qualquer elemento y ∈B, existe um elemento x ∈ A tal que f(x) = y (pois f é sobrejetora) e

esse elemento é único (pois f é injetora). Em outros termos, podemos

denir uma nova função: g : B → A associando a cada elemento y ∈ Bo único elemento x em A tal que f(x) = y. Em outras palavras,

g(y) = x, se e somente se, f(x) = y.

g é chamada de função inversa de f .

Quando existe uma bijeção f entre dois conjuntos A e B, dizemos

que A e B têm a mesma quantidade de elementos ou cardinalidade.

Para mais informações sobre funções, recomendamos a leitura de [3].

Page 305: Resolução de problemas

Referências Bibliográcas

[1] AIGNER, M. e ZIEGLER, G. (2002). As Provas estão

no Livro. Edgard Blücher.

[2] GARCIA, A. e LEQUAIN, I. (2003). Elementos de Ál-

gebra. Projeto Euclides, IMPA.

[3] LIMA, E. L.; CARVALHO, P. C. P.; WAGNER, E. e

MORGADO, A.C. (2004). A Matemática do Ensino Mé-

dio. Volume 1. Sociedade Brasileira de Matemática.

[4] LIMA, E.L.; CARVALHO, P. C. P.; WAGNER, E. e

MORGADO, A.C. (2004). A Matemática do Ensino Mé-

dio. Volume 2. Sociedade Brasileira de Matemática.

[5] LIMA,E.L.; CARVALHO,P. C. P.; WAGNER,E. e

MORGADO,A.C. (2004). A Matemática do Ensino Mé-

dio. Volume 3. Sociedade Brasileira de Matemática.

[6] LIMA, E.L.; CARVALHO, P. C. P.; WAGNER,E. e

MORGADO, A.C. (2001). Temas e Problemas. Socie-

dade Brasileira de Matemática.

[7] LIMA, E.L. (2001). Álgebra Linear. Sociedade Brasileira

de Matemática.

285

Page 306: Resolução de problemas

286 REFERÊNCIAS BIBLIOGRÁFICAS

[8] MORAIS FILHO, D. C. (2007). Um Convite à Matemá-

tica. EDUFCG.

[9] MORGADO, A.; CARVALHO, J.; CARVALHO, P.;

FERNANDEZ, P. (1991). Análise Combinatória e Pro-

babilidade . Sociedade Brasileira de Matemática.

[10] RIBENBOIM, P. (2001). Números Primos: Mistérios e

Recordes. Sociedade Brasileira de Matemática.

[11] SANTOS, J. P. O. (1993) Introdução à Teoria dos Nú-

meros. IMPA.

[12] SANTOS, J. P. O.; MELLO, M. P. e MURARI, I. T.

C. (2006). Introdução à Análise Combinatória. Editora

Unicamp.

[13] SOARES, M. G. (2005). Cálculo em uma Variável Com-

plexa. Sociedade Brasileira de Matemática.